Sokolov - Quantum Mehcanics
Sokolov - Quantum Mehcanics
Sokolov
T. M. Loskutov
I. M. Ternov
Quantum meEhaniE5
• A. A. SOKOLOV
• Y. M. LOSKUTOV
• I. M. TERNOV
Moscow State University
O rig in a lly p u b li s h e d in R u s s i a as
K v a n to v a y a M ekhanika
by A. A. S okolov, Y. M. L o s k u to v , and I. M. T e rn o v
T e x tb o o k P u b li s h i n g H o u se of th e Ministry
of E d u c a tio n of R S F S R , M oscow, 1962
Preface to the English Edition
PREFACE xiii
INTRODUCTION xv
PROBLEMS 511
Preface
A. A. Sokolov
Introduction
' A s m a g n e t i c m o n o p o l e s do n o t e x i s t in n a t u r e , C o u l o m b ’s l a w in m a g n e t o s t a t i c s i s
v e r i f i e d b y m e a n s of m a g n e t i c d i p o l e s .
xvi INTRODUCTION
where the primed coordinates refer to the moving system , and the
unprimed coordinates to the stationary system . We find that
accelerations and forces are identical in the two frames of refer
ence, and therefore the equations of mechanics (in which the
velocity does not appear) are invariant under this transformation.
If the Galilean-Newtonian transformation is applied to the Maxwell-
Lorentz equations, they assume different forms in different inertial
system s, because the equations contain the velocity of propagation
of electromagnetic waves which, added vectorially, has different
values in different inertial system s. The original Michelson-Morley
and other numerous experiments showed, however, that the speed of
light is the same in any direction in all inertial coordinate system s.
As a result, Einstein generalized the Newtonian principle of rela
tivity in a way that led directly to the so-called Lorentz trans
formations
-vt
y'= y, z z,
v~f
t— X
C V
t'= c•
V I-?2
The classical laws of electrodynamics are invariant under this
transformation. Since the equations of Newtonian mechanics,
however, are not invariant under the Lorentz transformations,
they had to be replaced by relativistic equations in which the
m ass m of a moving particle was related to its velocity v and
its rest m ass m„by the relationship
where £ and H are the electric and magnetic field intensities, re
spectively, p is the charge density (for example, of the electron),
and v is its velocity.
4 NON R E L A T I V I S T I C QUANTUM MECHANICS
2 e-w'
3 c3
( 1. 2 )
to obtain the following wave equation, which holds for the compo
nents of both vectors E and //:
(1.3)
H=k" x E, (1.4)
where k" — k k is a unit vector.
T H E QUANTUM THEORY OF LIGHT 5
where
2- 3 ). M
yB-i i , (1.9)
where A and ® are the vector and scalar potentials of the electro
magnetic field. These potentials are related by the Lorentz condi
tion
y . A -1 f ci ot
^ = 0. (1.10)
E = — V ® — 1c ^dt , '
( 1 . 11 )
H = VxA.
Pi dX _ , _ e .
(1. 12 )
Y 1—p3T ( l'
6 NONRELATIVISTIC QUANTUM MECHANICS
(1.13)
Substituting (1.12) and (1.13) into (1.8) and taking into account (1.10)
and (1.11), we obtain Eq. (1.6) for the motion of an electron. Thus,
our choice of the Lagrangian is justified.
Since we know X, we may also determine the Hamiltonian H:
H = l p A - x = y = + ^ 1 ^ + ^ = ^ + *
camav2
1—pa m:c* 1—pa *
H’f ^nonrel. ■ P‘
- f e<l>= (1.15)
— 2m0
(1.16)
by the relation
du
plu—'rfZT' (1.17)
In th e l i t e r a t u r e , t h e f u n c t i o n p ^ i s s o m e t i m e s u s e d for t h e s p e c t r a l d e n s i t y . The
f u n c t i o n p ^ i s r e l a t e d to p v by t h e e q u a t i o n
Pc0 - pv •
277
sin ce = 277 V.
8 NONRELATIVISTIC QUANTUM MECHANICS
CO
( 1 . 18 )
+ E* (U9)
Here e and m0 are the charge and rest m ass of the oscillator, <a is its natural frequency
of oscillations, and Ex is the x component of the electric field intensity of the black-
body radiation.
Representing Ex in the form of a Fourier series
CD
£*= 2 e« e ,'n“0'. (L20)
n=—co
where Exn is the amplitude of an individual oscillation of the field with frequency
m0 ( 1. 22 )
x =
_1 „ 2 eHmc.)1
( " “ ») + ' 3 m 0c»
The average energy of the oscillator, which, according to the virial theorem, is twice
the average kinetic energy, is given by
e
m0 (1.23)
E = m0x 2 = m0
1
—
n= _oo
* / v.
CO ^ ( n c o o ) - +
. 1• 23 « m>f)C
“o)a
,
oo CO
{ y / / " - ‘y V f f
’ n* —r t' »
n =» — co n, n ' =j= — oo
TH E QUANTUM THEORY OF LIGHT 9
2
gloj0<(«—n') __ J_ P e/<o0( («—.a') ^ _ 0 1or n = n,
T J \0 for n' y i n, (1.24)
—"2
where x = 2ji/ m0i Eq. (1.23) can be reduced to the form
(-o), ^ | £ , n|
m0 Jt2 = 2 m0 2 ^ra(noip)
i ««».)*- “T + { I w„cs
This equation has a very sharp maximum in the neighborhood of frequency a , and
hence the total energy of the oscillator will actually depend only on those terms of the
series for which na>0 «= <■>. Consequently, in the above equation, the square modulus of the
amplitude | Exn |2 can be replaced by| Ex |, where n0 — — , and at the same time the sum
can be changed to an integral. According to (1.21),2
d i a n - - ai 0d f l = a>0 = — , (1.25)
nQ
since d n = I. Therefore, we obtain
l* W !d<ji„
£= 2
m0 ( 2 e2 “
>;!l3 (1.26)
’ " 1 “ \3 m0 c3J
2 ua 1 ■*■'’ 0 1
(1.27)
On the other hand, the energy density u, which is related to the electromagnetic field
of the radiation by Eq. (1.16), can also be expressed in terms of | £ |2. Since the radia
tion is isotropic we have, on the basis of Eq. (1.16),
“ = L (£2 + "*) = 4
“ + &y + Ei), (1.28)
^ T h e r e a d e r s h o u l d n o t f e e l u n e a s y a b o u t o ur e q u a t i n g t h e d i f f e r e n t i a l d<on to t h e f i n it e
q u a n t i t y <Oq = 2tt/ t, for i t i s a l w a y s p o s s i b l e to m ak e t h e p e r i o d T s u f f i c i e n t l y l a r g e s o t h a t
th e r e l a t i o n s h i p Cl)q oj w i l l b e s a t i s f i e d . M a t h e m a t i c a l l y , t h i s c o r r e s p o n d s to a t r a n s i
tion from F o u r i e r s e r i e s to F o u r i e r i n t e g r a l .
10 N O N R E I —A T I V I S T I C QUANTUM MECHANICS
Using the expansion (1.20) and the rule (1.24) for averaging over t , we obtain
! in .
u = l E* = k 2 (1.29)
Hence, taking into account Eq. (1.18) together with the relationship
, dm- dm-
in - — a = n0 — - , (1.30)
m0 m
foro>n = to (n = n0)t we obtain
3n» I Exn„ I’
P<0 --- 4
2 h ( U l)
d.32)
(1.34)
= O
~n\ m = kT.
P» = (1-36)
F i g . l . l . R a d i a t i o n s p e c t r u m of a n i d e a l b l a c k body .
T he h e a v y d o tted line i n d ic a te s the R a y le ig h - J e a n s
2
c u r v e po) = pQX , a n d t h e s o l i d l i n e t h e P l a n c k c u r v e
Pa> —pox ^ / ( eX ~ 1)» w h i c h i s t h e s a m e a s t h e e x p e r i
m e n t a l c u r v e . H e r e p 0 = ( k T ) 3/ n 2h 2c 3, (o = oj0x, a n d
<u0 = kT/H.
C. PLANCK’S EQUATION
£ = ns, (1.38)
where n — 0 , 1 , 2 __
In order to determine the average value of the energy, we must
replace the integral (1.34) by the sum
00
(1.39)
t = fiu>. (1.41)
na) 3
Pm fnu >(1.42)
7i’>c, (eW — I)
1 kV
f his.
THE QUANTUM THEORY OF LIGHT 13
a - 4 3 )
r e3 dk 7t4
] 7 -1 15’
u = a7’4, (1.45)
15 cW
where
^ U s u a l l y it i s n o t t h e d e n s i t y u w h i c h i s m e a s u r e d , b u t t h e e n e r g y a T 4 w h i c h i s r a d i a t e d
p e r s e c o n d p e r s q u a r e c e n t i m e t e r o f t h e b l a c k b o d y ’s s u r f a c e w i th in a s o l i d a n g l e o f 277
c C c
(U = 277). In t h i s c a s e , t h e Stefa n -B o ltzm a n n c onstant i s O ~ 277 ---- a = — a = 5.67 • 10
8 77 4
e r g •cm ^ •d e g 4
14 NONRELATIVISTIC QUANTUM MECHANICS
w = J P\dk
«= jpo.du), (1.47)
o a
2Ttc 16it3cft
Px-- X2 Pm-- 2 r.cH (1.48)
X5(eftrx._i)
2Itch
*rxr
—5 2 rcrfl = 0.
h T l
e m ax .
£/ = 5(1 — e~v),
i / ^ 5 ( l — e-») = 4.965.
s= ( £ 2 + m d*x = <Px,(1.50)
«= j£ W x . (1.51a)
it = k" (1.52)
£= /ko). (1.53)
n = k'>-
C
= k'>-K = flk, (1.54)
2~b° / 2 it
where h — 2-h, and k = — is the wave vector! k = — is the wave
number).
On the basis of these concepts, in 1905, Einstein constructed a
qunnfitative theory of the photoelectric effect, which had been dis
covered by Hertz in 1887. What is observed in the photoelectric
effect is the following: the potential difference required for a
spark to jump between two small charged spheres is reduced if the
THE QUANTUM THEORY OF LIGHT 17
= (1.55)
Hk' — hm'lc represent the momentum of the photon before and after
scattering. We rewrite Eq. (1.56) in the form
c8
u> — = — m 0),
(1.56a)
* -* = " •
2 rn
lo -IthC
- - = 2 .4 - 10"‘“ cm.
t l ’ oC
-y- ~ 10 1 = lO°/0.
Wavelength
F i g . 1.4. S p e c t r a l d i s t r i b u t i o n o f x - r a y s in
t h e C o m p to n e f f e c t b e f o r e (u p p e r c u r v e )
a n d a f t e r ( lo w e r c u r v e ) s c a t t e r i n g .
Only t h e i n t e n s i t y o f t h e m a x im a d e p e n d s on th e t y p e o f s c a t t e r i n g s u b s t a n c e . A s the
a to m ic w e i g h t o f t h e s c a t t e r i n g s u b s t a n c e i n c r e a s e s , t h e i n t e n s i t y o f t h e u n s h i f t e d m a x i
mum i n c r e a s e s , a n d t h a t of t h e s h i f t e d maxim um d e c r e a s e s .
C hapter 2
(2 . 1 )
In s p e c t r o s c o p y , i t is c u s t o m a r y to w r i t e B a l m e r ' s fo rm u la in t h e form
w h e r e t h e R y d b e rg c o n s t a n t for h y d r o g e n is R sp 1 0 9 , 6 7 7 . 6 c m T h e v a l u e o f th e R y d b e r g
c o n s t a n t in Kq. (2 1) is r e l a t e d to / f sp by t h e e q u a t i o n
A 2 nc
THE BOHR QUANTUM THEORY 21
which are called spectral terms. For the hydrogen atom, these
terms are given by
Tv - T_
Hence
a n d t h u s the R i t z c o m b i n a t i o n p r i n c i p l e l e a d s d i r e c t l y to E q . (2 .1 a).
22 N O N R EL A TIV ISTIC QUANTUM M E CH A N IC S
E's2= P
P 3- rr3= e-^ *
Hence
E = % r- <2-3)
Therefore, if an electron with charge e = — e0 and m ass m0 is
placed at a distance x from the center of the atom, it experiences
a quasi-elastic attractive force towards the center
x — A cos K / + <p0),
THE BOHR QUANTUM THEORY 23
where
O>0
el
m0Rl’
where n = 1, 2, 3,...
The decisive blow to the Thomson model was dealt by the
experiments of Rutherford, who showed that the positive charge is
not distributed throughout the entire volume of the atom, but is
concentrated virtually at one point. Nevertheless, the Thomson
potential inside a nucleus of finite dimensions, with a charge Ze03
uniformly distributed through the volume, is
(2. 6 )
V2 2 Rl
If t h e c h a r g e of t h e n u c l e u s i s Z e g , t h e e l e c t r i c f i e l d i n t e n s i t y i n s i d e t h e n u c l e u s i s
Z e0 <9$ r
E = — r = ------------,
f?0 dr r
from w h ic h , u s i n g t h e b o u n d a r y c o n d i t i o n t h a t a t r = R q t h e p o t e n t i a l i s t h e s a m e a s for a
point charge
Z e0
$ 1"= R q
Ro
w e o b t a i n E q. (2.6).
24 NONRELATIVISTIC QUANTUM MECHANICS
In e a r l i e r e x p e r i m e n t s , R u th e r f o rd a n d h i s c o - w o r k e r s h a d e s t a b l i s h e d t h a t a l p h a
p a r t i c l e s h a v e th e s a m e m a s s a s t h e h e l i u m ato m , a n d a p o s i t i v e c h a r g e w h ic h i s t w i c e
the m a g n i t u d e of th e e l e c t r o n c h a r g e . It i s now known th at a l p h a p a r t i c l e s a r e th e n u c l e i
of he I ium atom s.
THE BOHR QUANTUM THEORY 25
where 7. is the gravitational constant and has the same form as the
potential energy of the Coulomb attraction between an electron and
a nucleus
5If t h e f i n i t e n e s s o f th e n u c l e a r m a s s Mnuc *s t a k e n i n t o a c c o u n t , th e n u c l e u s h a s a
c e r t a i n r e c o i l ( l ik e t h a t of th e a l p h a p a r t i c l e ) a s a r e s u l t of th e i n t e r a c t i o n . In t h i s c a s e ,
all th e c a l c u l a t i o n s m u s t b e p er fo r m ed in th e c e n t e r - o f - m a s s s y s t e m and, in t h e r e s u l t s
o b t a i n e d for t h e c a s e ,\7nuc it i s n e c e s s a r y to r e p l a c e th e m a s s of t h e a l p h a p a r t i c l e
Mg by t h e r e d u c e d m a s s
M0 »'nUc
re d
« 0 + Mnuc
E = const, (2.7)
(r x v ) = const, (2 . 8 )
F i g . 2.2 . D i a g r a m for t h e d e r i v a t i o n o f R u t h e r
f o r d ’s fo rm u la for t h e c r o s s s e c t i o n of e l a s t i c
s c a t t e r i n g o f a l p h a p a r t i c l e s by n u c l e i .
Y ('* + r y ) + <2-12)
i = M0 H . (2.13)
(2.15)
Vob (2.16)
It 1= r2“
v0bu\
and
(2.17)
2Ze\ (2.19)
u M(,vlb2 = 0 .
Hence
u = A cos y-\-B sin 7 ------------ . (2.20)
( 2 . 21 )
lim r = lim —= 0 0 ,
■-* r. ^
and
( 2 . 22 )
limrsin 7 = lim =
28 NON R E L A T I V I S T 1C Q U A N T U M M E CH A N IC S
A 2Zej (2.23)
MoVjb1 ’
« = j s in c P- - ^ g | r (l + cos(p). (2.25)
~C>-0) %
I‘ i^. 2. *1. D e p e n d e n c e of th e s c a t t e r i n g
a n g l e 0 on th e i m p a c t p a r a m e t e r b, w h e r e
b0 ^2 ^ 3 ^oo-
Equation (2.26) can be checked experimentally by photographing
the tracks of alpha particles in a Wilson cloud chamber. From the
maximum scattering angle, it is possible to compute the minimum
THE BOHR QUANTUM THEORY 29
dN = N-2*\bdb\. (2.28)
The ratio dN/N has the dimensions of area and is called the differ
ential cross section. It is usually denoted by dj. Taking the deriva-
tive of cot2y in Eq. (2.29), we obtain the Rutherford formula for
elastic scattering of alpha particles by a Coulomb center:
* = ( w ) ’- ^ r - (2'30)
sin 2
the lim its of applicability of the Coulomb law. Blackett took a large
number of photographs of tracks of particles in a Wilson cloud
chamber and calculated the frequencies with which the various
scattering angles occur. From analysis of the experimental data, he
established that the number of particles observed at large scatter
ing angles [according to (2.26), large angles correspond to small
values of the parameter b ] is markedly sm aller than the number
yielded by the formula (2.30). On this basis, Blackett concluded
that, in air, for example. Coulomb’s law is valid down to dis
tances of the order of 3-KP12 cm. At sm aller distances, there is a
deviation from Coulomb’s law. Indeed, at cm, the
interaction between the alpha particle and the nucleus appears to
take the form of a strong mutual attraction. Further experimental
investigations have confirmed the existence of a characteristic
attractive force at distances less than 10“ 12 cm. This attraction
drops off rapidly with increasing distance from the nucleus.
The Rutherford formula (2.30) can be used to find the number
Z from an experimental determination of dN and N. This was
undertaken by Chadwick, another of Rutherford’s students. Chadwick
showed that the value of Z is very clo se to the atomic number,
which gives the element’s position in Mendeleyev’s periodic table.
The existence of this phenomenon was rigorously proved at a later
date.
Thus the experiments of Rutherford and his colleagues definitely
established the planetary model of the atom. These experiments
proved that the positive charge of the electron is concentrated in
a nucleus with dimensions of 1 0 ' 1 3 — 1 0 ~ 12 cm, and that, inside the
atom. Coulomb forces keep the electrons moving in orbits with
a radius of the order of 1 0 ” 8 cm.
T = —- - 'r )>
V = — y.el
r
THE BOHR Q U ANT UM THEORY 31
+ — , (2.32)
d dX ,
dt dtp 0
(2.33)
d dX
dt Pr dr 0 .
Here
ujr, o,
and
pr= * X - = m0? (2.34)
is obtained from the condition that the time t does not occur ex
plicitly in the Lagrangian.
We shall consider only the sim plest case, that of circular orbits,
for which r —. 0. Accordingly, pr = m j vanishes, and from (2.33) we
have
dX
-F = m r. 2f - 7Ze\i = 0_ , (2.37)
6In t h e f o l lo w in g c h a p t e r s , t h e m a s s o f t h e e l e c t r o n in n o n r e l a t i v i s t i c t h e o r y s h a l l be
d e n o t e d b y mo, s i n c e m w i l l b e u s e d for t h e m a g n e t i c q u an tu m nu m b er.
32 N O N R ELA TIV ISTIC QUANTUM M E CH A N IC S
Hence
? = --> ■ (2.38)
pv = ml)r^ = -^ , (2.41)
or
? = 2nm0rs • (2.42)
From Eqs. (2.38) and (2.42), we can obtain expressions for r and
$ in terms of the adiabatic invariant:
_ l1 (2.43)
r 4K*m0Zel ’
(2.44)
L — — 2^ . (2.45)
dE W m o Z ’ei (2.46)
v°— dl — /»
THE BOHR QUANTUM THEORY 33
This relation holds not only for the case under consideration, but
also for any periodic or quasi-periodic motion . 7 Moreover, a
system performing any periodic motion can generally emit radia
tion not only at the first harmonic k = 1 , but also at the harmonics
k = 2, 3, 4 , . . . . The following expression is, therefore, obtained for
the classical frequency of radiation:
dE 2 e*w*
dt 3 c3
until it falls into the nucleus. This is not, however, what actually
takes place, and atoms can exist in a non-radiating state for an ar
bitrarily long time. Moreover, according to the classical theory,
the frequency of the radiation should be the same as the frequency
of mechanical oscillation (the fundamental frequency u>= uj0 = 2 itv0)
or, alternatively, an integral multiple of this frequency (one of the
harmonics %= mu0, where n = 2 , 3 , 4 , . . . ) . Once again, this predic
tion does not explain Balmer’s experimentally established formula
(2 . 1 ) for the lines in the spectrum of radiation.
7We d e f i n e p e r i o d i c m o tio n a s a m o t i o n in w h i c h a p o i n t r e t u r n s to i t s i n i t i a l p o s i t i o n
a f t e r a c e r t a i n p e r i o d o f tim e. A s e x a m p l e s o f t h i s , w e c a n t a k e h a r m o n i c m o tio n
x = a c o s (tit
or m otion a b o u t an e l l i p s e
x = a c o s (i)\t ,
y = b c o s Cl)2 f »
w h e r e th e f r e q u e n c y c0 \ i s i n c o m m e n s u r a b l e w ith co2 -
34 N O N R EL A TIV ISTIC QUANTUM M E CH A N IC S
where the quantum number n can assume only integral values, that
is, n = 1, 2, 3 , . . . (in classical mechanics, the adiabatic invariant I
could assume any constant value).
Second, Bohr hypothesized that, when an electron p asses from
a stationary state with energy En (the initial state) to a state with
an energy £„-<£„ (the final state), the atom radiates a quantum of
energy h'< = hu> (the frequency postulate), whose angular frequency
COis
(2.48)
, = ( n - n ’) ^ - = k ^ - . (2.48a)
therefore, the collisions are elastic and the current does not change.
If however, £ 3 s 4.9 ev, the electrons in the beam may give up a
part of their energy (namely, 4.9 ev) to the atoms, and therefore,
the current will change. If the electrons’ energy lies in the range
14.7 ev ^ > £> 9.8 ev, the transfer of energy to the atoms can occur
twice; 4.9 ev is given up in the first collision and 4.9 ev in the
second.
We shall now use the first and second Bohr postulates [Eqs.
(2.47) and (2.48)] to construct a theory of the hydrogen-like atom.
In the equation for the radius of the orbit (2.43) and the equation for
the energy (2.45), let us substitute for / its quantized value, which
from Eq. (2.47) is
/ = 2-xnh,
We thus have
n2 ft2 (2.49)
n m„Zel ’
m 0Z 2e \
2nJH2 (2.50)
where
g0 = —
TTIqBq «=>0.529-10"8*locm (2.53)
m0eJ
R 2ft8
(2.55)
8 A c c o r d i n g lo t h e c o r r e s p o n d e n c e p r i n c i p l e , in t h e l i m i t , a l l t h e r e s u l t s o f a p r e v i o u s
t h e o r y ( h e r e , c l a s s i c a l e l e c t r o d y n a m i c s ) s h o u l d f o l l o w from a n e w t h e o r y ( h e r e , t h e D o h r
th eo ry ).
For in stan ce, fo r h ■* 0 (o r a t t h e lim it of la rg e q u an tu m n u m b e rs ), th e r e s u l t s of q u a n
tu m m ech an ics sh ould approach the c l a s s i c a l results. In e x a c t l y the s a m e w ay, w hen
fV > 0, th e r e s u l t s o f t h e r e l a t i v i s t i c t h e o r y s h o u l d a p p r o a c h t h e n o n r e l a l i v i s t i c r e s u l t s ,
a n d so on. I 'h u s , th e c o r r e s p o n d e n c e p r in c ip le e n a b l e s u s to c h e c k th e e x t e n s i o n o f a
t h e o r y b y r e q u i r i n g it l o r e d u c e to t h e c l a s s i c a l p i c t u r e a t t h e l i m i t .
In t h e i n i t i a l s t a g e s o f a n e w t h e o r y , w h e n it c a n n o t y e t b e u s e d Lo i n v e s t i g a t e c e r t a i n
phenom ena, th e c o r r e s p o n d e n c e p rin c ip le m ay be u s e d fo r r e a s o n a b l e e x t e n s i o n s o f th e
p r e d i c t i o n s o f t h e o l d t h e o r y lo the.’ n e w t h e o r y . T h u s , f o r e x a m p l e , t h e B o h r t h e o r y c o u l d
b e u s e d to c a l c u l a t e t h e f r e q u e n c y o f t h e r a d i a t i o n , b u t n o t i t s i n t e n s i t y . B o h r w a s a b l e
lo d e t e r m i n e t h e i n t e n s i t y fr o m t h e c o r r e s p o n d e n c e p r i n c i p l e , a c c o r d i n g to w h i c h t h e o n l y
c h a n g e s of the q u an tu m n u m b er n that w e re a llo w e d (th e s e l e c t i o n r u le s \ n n —n ) w ere
th o s e that c o i n c i d e d w ith th e c l a s s i c a l l y a llo w e d h a r m o n ic s of th e ra d ia tio n . T h e quantum
th eo ry of ra d ia tio n of light w a s c o m p le te d only w ith th e d e v e lo p m e n t of q uantum e le c tr o -
d y n am ic s.
THE BOHR Q U ANT UM THEORY 37
Problem 2.1. Using the Bohr theory, quantize a hydrogen-like atom for the case of
elliptic orbits (that is, find the spectrum of the energy levels). Show that, In the non-
relativistic approximation, the coordinates r and f have the same frequency of variation;
that is, the motion is periodic. Show that the formula for the frequency of the radiation
of an atom remains the same as in the case of circular orbits.
What new feature is introduced in the theory by considering elliptic orbits?
Solution. Using condition (2.40) and noting that, in this problem, the generalized
momenta corresponding to the <pand r coordinates are
. •1 f n n , 2m0Zel P%
p f = m0r ‘<f = const and p r = m ar = \ 2m0E 4----------- p -,
we find
max _____
/» = j = /, = 2 J prdr = 2 * { - p v + Zel
0 'min
(2.56)
min
Hence the energy E is
c- 4jc2m0Z2e$ _ n
E= ~ m r + L r < 0-
dE dE
a' = Wr and “?= c>7;
coincide with one another. Using the Bohr quantization rule (2.47), we replace the adiabatic
invariants lr and by nrh and/i^A, respectively, obtaining
Wo Z ‘ g}
Th 3 /ta ’
38 N O N R EL A TIV ISTIC QUANTUM M E CH A N IC S
Problem 2.2. Find the classical equation of motion and the trajectory of an electron
about the nucleus according to relativistic theory. Show that, in this case, the r and tp
coordinates change with different frequencies; that is, the motion is quasi-periodic.
Determine the angle through which the perihelion of the electron Is shifted during "one”
revolution. Obtain a formula for the energy levels and find their splitting. Compare the
results with the corresponding nonrelativistic problem 2.1.
Solution. Using the relativistic Lagrangian function
X = - m0c2 V l - y + ,(2.57)
where
P2 = 5 = C
W 2+
„ o , . Po Zel
/; = c \ rS----- / const,
r v
where
7 c‘p$ ’
f <?P\
9 Z e\E ’
£
It is apparent from Eq. (2.58) that the motion is quasi-periodic. For the shift i f of
the perihelion, we have from (2.58)
Atp =
2 it (1 —i) *-Z2el
/¥= 2 *pr
where
E2
A = ml c2 1----
B^ L ejJL I l el
c‘ c2
From this it is evident that the frequencies and cor = will be different.
'9 K
Using the Bohr quantization rule (2.47), we transform Eq. (2.59) to
where a =
€“ 1
is the fine-structure constant.
ch 137
Expanding the formula (2.60) into a series in a2 and restricting ourselves to quantities
of the order of a4, we have
,, _ R h Z J l. a2Z 2 ( n 3 VI
n,n? n2 [ n2 \n 4 )J‘ (2.61)
Since n9 varies from 1 to n, it follows from Eq. (2.61) that the energy levels, which
are determined by the principal quantum number n = n, + nv, are split into n closely
spaced sublevels (this close spacing is a consequence of the smallness of a-).
40 N O N R EL A TIV ISTIC QUANTUM M E CH A N IC S
Determine the values of s at which stable states of the system are possible.
Answer, s < 2.
Hint. Use the expression for the effective potential energy
„ Pi -4
eff 2 m<,ra rs
and make use of the fact that stable motion is possible if Pgff has a minimum.
Problem 2.4. Find the scattering cross section for nonrelativistic electrons by nuclei
(Coulomb point charges). Compare the result with the scattering cross section for alpha
particles.
Answer.
di - Ze\ ,2 dQ
4i'0 1 .. »*
an 7
Problem 2.5. Show that in the relativistic scattering, unlike the nonrelativistic case,
electrons can be captured by nuclei. Obtain the total capture cross section.
Solution. According to nonrelativistic theory, the trajectory of electrons in the
Coulomb field of a nucleus is given by the equation
r ~ Tor ----<j>>’
1 + £*7 COS
where
_P\_ + DhZ-el
m„Zef, ’
At 0, this equation describes hyperbolic motion, and therefore the capture of elec
trons Is impossible.
In relativistic mechanics, the equation of the trajectory has the form of Eq. (2.58).
If
Z-el > I in this equation, y becomes imaginary. Therefore, r —• Ofor <p— co; that is.
__ _ T.Z-e},
“cap ~ r‘" m a x ~ cip-i ■
C hapter 3
A. DE BROGLIE WAVES
E— — jt = M = yAr°. (3.1)
_m„c-_ n _ iiiqV
(3.2)
vr^ w ’ p ~ ’
should also exhibit wave-like properties. The corresponding fre
quency and wave number were defined by equations sim ilar to
Einstein’s:
£ = hu> and p = hk. (3.3)
T h i s h y p o t h e s i s w a s m a d e by d e B r o g l i e w i t h a tw o f o ld p u r p o s e : f i rs t , to p r o v i d e a
p h y s i c a l b a s i s for t h e B o h r q u a n t i z a t i o n ; s e c o n d , to e x p l a i n t h e f i r s t e x p e r i m e n t s on e l e c
tr o n d i f f r a c t i o n ( s e e below).
2
F ro m no w on, w e s h a l l l e a v e i t to t h e r e a d e r to d i s t i n g u i s h b e t w e e n t h e r e l a t i v i s t i c
e n e r g y (w h ich i n c l u d e s t h e r e s t m a s s e n e r g y ) a n d t h e n o n r e l a t i v i s t i c e n erg y . Only in
c a s e s in w h ic h th e two e n e r g i e s a p p e a r in t h e s a m e e q u a t i o n w ill t h e y b e d i s t i n g u i s h e d
by s o m e k i n d o f i n d e x , for e x a m p l e ,
E = E f m0 c 2 .
42 N O N R EL A TIV ISTIC QUANTUM M E C H A N IC S
X=■?“-=
k *p. ' (3.4)
2f = n. (3.6)
m0v ’
m0v3 _ <vt>
' 2 — 300’
(3.8)
, h h Y \ 50 1 - 2 ■1 0 “ 7 /Q
X= — = -v - — —TF7— cm. (3.9)
m°v Y m0e0<I> V 'l'
Fig. 3 .2 . D i a g r a m o f t h e D a v i s s o n - G e r m e r
e x p e r i m e n t s on e l e c t r o n d i f f r a c t i o n .
T o r e d u c e th e a b s o r p t i o n of e l e c t r o n s , a r e l a t i v e l y thin foil ( t h i c k n e s s of th e o rd e r of
10 cm) w a s u s e d .
WAVE PROPERTIES OF PARTICLES 45
s = 2d sin 9 , (3.11)
where 9 is the angle between the ray and the lattice plane. Since
the single crystals of the foil are oriented at random, a ray can
leave the foil at any angle with
respect to the original direction
(see Fig. 3.5). Among the crystals,
there will be some that are oriented
at just the right angle to satisfy
Bragg’s law
2d sin ©= n\ = n - - - - - , (3.12)
y <i>
F i g . 3.4. R e f l e c t i o n from a th r e e -
dim ensional cry sta l lattice.
where n is an integer. Whenever
this is the case, a diffraction maxi
mum occurs and a bright spot Q is found on the screen. Since the
experimental apparatus is cylindrically symmetric, the bright spots
form diffraction rings, whose radiuses Rn can be found from the
relation (see Fig. 3.5)
ta n 2 ? = f-, (3.13)
where L is the distance from the screen to the polycrystalline foil.
Since the angle 9 is very sm all in these experiments (9 ^ s i n © ^
tan 9 ), Eqs. (3.12) and (3.13) yield
L_
Rn= n d 1.2 • I 0 ~ 7 (3.14)
F i g . 3 .5 . D i f f r a c t i o n of e l e c t r o n w a v e s i n a
p o ly c ry sta llin e s u b s ta n c e (the T a rtak o v sk iy -
T hom son experim ents).
Et — p* — const; (3.16)
d x E cs
dt p v
(3.17)
Ax _ E
At p
F i g . 3 .6 . T h e p r o p a g a t i o n o f a m o n o c h r o m a t ic w a v e .
D u rin g t h e tim e A t, t h e w a v e a s a w h o l e m o v e s a d i s
t a n c e Ax. T h e p h a s e v e l o c i t y o f t h e w a v e i s ------.
At
48 NONRELATIVISTIC QUANTUM MECHANICS
0 , P' < P —
IV (3.18)
A^ = \ rA
^P' P ~ % =/; + 2 "
0 , ?>P+f-
p+ 4 p
(£ 7 - p ’x )
6 dp’. (3.19)
In this equation, let us change from the variable //to the variable
p" r= // — / / — -fd cs p" j=s , dp’ = dp’ ^ and expand C' in a series
about the central point // = /;:
r f ■) i •/
Id = c /' -t ■wifi + !>■% + 7""‘ $ + ■ • • • <3-20>
WAVE P R O P E R T IE S OF P A R T IC L E S 49
(3.22)
(3.24)
which shows that the group velocity u of the wave packet as a whole
is exactly equal to the velocity v of the particle itself.
From the expression for the amplitude fl(£) = i4-^p of the wave
packet at time t — 0 (when the amplitude corresponds to ?= x; see
Fig. 3.7), it is readily seen that the maximum value of this
amplitude B(0) = A lies at the point x = Q. At all other points cor
responding to the relative maxima the amplitude is sm aller. In
particular, considering different values of the argument we have
50 NON R E L A T I V I S T I C QUANTUM M E CH A N IC S
F i g . 3 .7 . F o rm of t h e w a v e p a c k e t a t t = 0.
Since the center of mass of the wave packet (that is, its principal
maximum) travels with the velocity of the particle (u = v), the wave
packet describes the localization of the particle. In particular,
it follows from other hypotheses that have been made that the
position of the particle is characterized by the square modulus of
the amplitude of the x wave, namely,
IB I4 = **>!'■
p p
As a result, the wave packet gradually spreads out with time. The
“ spreading” time is determined by the time interval in which the
initially disregarded part of the phase of <|> in (3.21) becomes com
mensurate with it. According to (3.20), the neglected part of the
phase is proportional top' 4 <~(Ap)4. Thus, from (3.20) we can obtain
a measure of the time At which elapses between the initial forma
tion of the wave packet and its distortion:
(V) 2 -d2E
T—— 7C.
2n dp2
Or, the time interval starting with the formation of the wave packet
and ending when the distortion of the latter can no longer be
neglected is
At
2th (3.26)
d E _ _ p_
dp m0 ’
cEE_
dp1 m0 ‘
A t~ ^ {A x )\ (3.27)
A p * ~ x s'nt?' (3.29)
, - d%- and p v
r ~ dr df *
pili„x
<f(x) =
0
is not monochromatic for finite values of /. Find the range of wave numbers Ak = k — ka
over which the amplitudes of the individual harmonics may be regarded as nonzero.
Solution. Let us represent <p(x) in the form of a Fourier integral
OO
J e - k*dx.
—CO
Substituting the expression for tp(x), we find
s i n - (k — ka) I
/I (k) = / jt (k — A’o) I
"spread” of the wave vector due to the finite width of the wave packet. Setting Aft = ,
and / — Ax, we again obtain the uncertainty relation:
Ap Ax ~ ft.
Problem 3.3. Find the mean velocity and the "spreading” time of the wave packet
m
? (T, t) = A (ft) e~ ‘ M - *x) dk (3.30)
Solution. Substituting the expression for A (ft) Into Eq. (3.30) and Integrating, we find
for t = 0
+ i (ftp*—a>0()
qYn +
?(■*. 0 =
2/flp /
where
C
D0
m0 c2 ftft2
n r + 2 w7 -
Hence we have for the probability density
<?a(jy—
71^ 2[l + (l £ ) 1
l?C*, 0 l2;
/■+ (£ )'
where t»0 = - — is the velocity of the center of mass of the wave packet. From this formula
mo
It is seen that the maximum of the wave packet, that is, the point x = v 0t, moves with the
velocity Vo of the particle.
56 N O N R EL A TIV ISTIC QUANTUM M E C H A N IC S
If we substitute q ~ into this formula, the "spreading” time of the wave packet is
expressed by a relationship which coincides with Eq, (3.27).
The amplitudes with the same order of magnitude correspond to frequencies lying in the
range
d to = I CO W0 I ' •y.
It is this equation which represents the line broadening since, in effect, only the
amplitudes corresponding to these frequencies differ from zero.
Since the damping coefficient 7 is connected with the mean life of the damped oscilla
tions by the relation Af ~ 1/ 7, the above equation yields the familiar optical relationship
connecting the broadening of the spectral lines and the mean life of the atom:
At Aw^ 1 .
Hence, since Am = AEjht we obtain what is known as the fourth uncertainty relation
AF.At ~ h.
C hapter 4
where u>= 2 :tv is the angular frequency, arid the spatial part (/*) of
the wave function satisfies the equation
We then have
+ = (4.5)
we have
= (/*)]. (4.7)
Once we have found the space-dependent part <Ji(r) of the wave func
tion from (4.8), we can use Eq. (4.2), which is valid for all mono
chromatic waves, to obtain the complete wave function, which
depends on both (spatial and time) coordinates. Substituting U>= Y ’
we have 1
i ~ t
(4.9a)
(4.10)
E'jt .. (4.11)
60 N O N R EL A TIV ISTIC QUANTUM M E C H A N IC S
V ^ n+ - ^ ( £ n- m n= 0, (4.8a)
Since
— V-B,
where
«= — W * .,
^ V- B d 'x = J‘ BndS = 0,
j 6*.6,(rfb: = 0. (4.17)
If, however, n’— n (or £„ = £„■), this integral does not go to zero
and we may impose the requirement that it be equal to unity (the
normalization condition):
(4.18)
I
V'o for —oo<\\:<;0 (region I)
0 for 0 < . * < / (region II) (4.19)
where
62 NON R E L A T I V I S T I C QUANTUM M E CH A N IC S
(4.22)
k = ± -V 2 m ,(E -V ,).
F i g . 4 .1 . T h e m o tio n o f a p a r t i c l e in a
p o t e n t i a l w ell.
where
If the energy can assume any value without restriction, the wave
function inside the potential barrier (0<^£<Fo) will contain both
an exponentially increasing part and an exponentially decreasing
part (see Fig. 4.2). Therefore, we must choose only those values
of £ for which exponentially increasing solutions do not exist inside
Exponentially
F i g . 4 .2 . Wave f u n c t i o n for a g i v e n v a l u e o f E .
T h e e n e r g y l e v e l i s t a k e n to b e t h e a b s c i s s a of
t h e w a v e fu n c tio n .
£ m= B e xl.
when E < V q, t h e n u m b er of u n k n o w n c o e f f i c i e n t s in t h e w a v e f u n c t i o n i s s m a l l e r
t h a n t h e n u m b e r o f i m p o s e d c o n d i t i o n s . A c c o r d i n g l y , s o l u t i o n s are p o s s i b l e on ly for c e r t a i n
v a lu e s of E and a d is c r e te spectrum i s obtain ed .
4
In j o i n i n g t h e s o l u t i o n s , we m u s t m a t c h t h e a c t u a l w a v e f u n c t i o n s ( a n d a l s o t h e i r
firs t d e r i v a t i v e s ) a t t h e a p p r o p r i a t e p o in t.
64 NON R E L A T I V I S T I C QUANTUM M E CH A N IC S
sin kl = 0 , (4.28)
from which
kl = -n, (4.29)
V=0 /=°°
F i g . 4 .3 . P a r t i c l e in a p o t e n t i a l w e l l with
in fin ite ly high w alls .
to the wave functions for positive n, taken with the opposite sign.
Since lA = E, we obtain the following equation for the energy
spectrum (the eigenvalues):
En (4.30)
'2 inj*
which gives
'in= j . (4.32)
F, — --r- , , / 2 . —.v
(4.34)
2 mj* ' ^ ) / -ln / ’
Ei = 4 £!, 0 o= ^ —sin 2 ~ (4.35)
d ‘- l
k-'i = 0 , (4.37)
(lx-
66 NON R E L A T I V I S T I C QUANTUM M E CH A N IC S
where
. -i_ _ 2 maE _ _ p 2
ti‘ ~ li! ■
(4.38)
It is seen that the first term Ae~'(',“~kx) describes the motion of the
particle in the positive direction of the x axis, and the second term
Be the motion in the negative direction. If we restrict our
selves to the traveling wave which is propagated in the direction of
positive x, we have
^ = Aeikx. (4.41)
k=^. (4.43)
where /i= 0 , rh 1, zb2, zL3 ........ From Eqs. (4.38) and (4.43), the
e; crgy levels are
^ <p*(J>dx — 1 . (4.45)
(4.46)
* , 1f ------ T ~ ^ dx =
^ nd x = T \ e
1/2
I -
we find
(4.52)
where
(4.54)
if(z) = L 2 e‘*a2,
where
(4.57)
and
(4.58)
where
= + + (4.59)
THE T IM E -IN D E PE N D E N T SCH RO D IN G ER WAVE E Q U A TIO N 69
i/{t) = L ie-(nHEt-p-r)t (4 .6 2 )
where
p = hk, E = £ - o. (4 .6 3 )
C n ^ ^ K x ) F (x ) d x . (4.65)
In order to bring <]>„(x) under the Integral sign, we change here the variable of integra
tion x to x' and substitute the expression for Cn into the expression (4.64):
This expansion is a generalized Fourier series. We can obtain the ordinary Fourier
series from it by substituting the harmonic functions (4.47) for (*).
We note that the sum over n in Eq. (4.66) is a 8 function:
2 *5 M = 6(*—*'). (4*67)
^fn'Kn- = fn.
n'
It follows that the 8 function differs from zero only in a very narrow region ~ x , just
as the S symbol differs from zero only for n' = n. By setting F (x1) = 1 in (4.66), we obtain
one of the fundamental properties of the 6 function:
§ 6 (x — x')dx' = l. (4.68)
It is interesting to note that the relations (4.66) and (4.68) are completely independent of
the particular set of orthonormal functions used to construct the 8 function. In the simplest
case, the 8 function can be constructed from the
< 1 3(X-X'(X) orthonormal functions (4.47), which are used In
I expansion into a Fourier series:
t( x - x ')= j2 je L . (4.69)
The range of integration over ft should be taken from — oo to + oo. We note that the 8
function belongs to the class of functions that are known as "improper functions.” This is
manifested in the impossibility of direct evaluation of the Integral (4.70), since it diverges
at x = x'. Therefore, if we wish to obtain a representation of the 8 function, we have to
"smear out” somewhat the integrand of (4.70). For Instance, the 8 function may be
"smeared out" as follows:
A graph of the smeared 8 function is given in Fig. 4.4. At the limit a —. 0, the function
must have the following properties:
THE T IM E -IN D E P E N D E N T SCH R O D IN G ER WAVE E Q U A TIO N 71
t (x — x 1) = oo for x = x',
As we approach the limit a -* 0, the area between the 8 function and the x axis remains
unchanged:
J " .•+(■,■-*)• =L
The three-dimensional 8 function can be similarly defined as
§ i(r)d * x = l. (4.75)
The integrals (4.74) and (4.75) are three-dimensional. The former extends over the
entire wave-number space (h), and the latter, over the entire coordinate space (r).
It is very convenient to express the density of a point charge with the help of a three-
dimensional 8function:
P (r) = e6 (r). (4.76)
V2<I>= -4 n e 6 (r ) = - - — 2 ^ d 3ke‘*r ,
sin kr
1 I" (4.77)
Thus, the three-dimensional 8 function enables us to describe the singularity at the point
r —>0 of the Laplacian operator applied to 1jr:
In d e t e r m i n i n g th e p o t e n t i a l w e m a d e u s e of t h e r u l e for d i v i d i n g a n e x p o n e n t i a l
fu n c tio n by t h e L a p l a c i a n o p e r a t o r:
fV 2r v ftr = —
- k2 '
T he v alid ity of th is rule i s im m ediately obvious, sin c e
v 2( v 2r v ft' = v 2 — = cikr
-k 2
72 N O N RELA TIV I S T IC Q U ANT UM M E CH A N IC S
We shall not discuss the properties of the 6 function any further and shall proceed to
use it to normalize the continuous spectrum for one-dimensional free motion. From
(4.41), the wave functions for this case are of the form
p _ iS ix
p ) = A e ft , 't'* (P:) = Ae - s • (4.79)
The normalization of these functions is related to the 5 function in the following way:
— oo
= Ai2r.M (p —p') = h (p — p'). (4.80)
A = -^= r,
Therefore,
I ' j. *
• (4-81)
Let us rompare the ordinary normalization of wave functions with the Bfunction
normalization. We can write the orthonormality condition forfunctlons normalized by the
ordinary method as follows:
Analyzing the solutions that we have obtained for a particle in a potential well and for
a free particle, we arrive to the following conclusions. If the condition V E is satisfied
at all points of space at infinity, the energy spectrum Is discrete. If, on the other hand,
there are regions at infinity In which V <; /:, the energy spectrum is continuous. While
this conclusion was obtained on the basis of solutions for a rectangular potential well, it
is quite general as long as the potential energy is a continuous function of x, y and z.
C hapter 5
(5.1)
(5.2)
(5.3)
6 ( t ) = , ^ C ne - m)F^ n. (5.4)
n
To prove this, we substitute (5.4) into (5.3), remembering that Cn
are constant coefficients and that >]>, satisfies the equation
^ „ + ^ ( £ » - m » = 0, (5.5)
E=m (5.7)
$( t ) = e * En‘ t™.
w e have
E* (/) = En<|;(/).
( )p**rMors wi l l h e d e n o t e d by r o m a n c h a r a c t e r s .
T HE T I M E - D E P E N D E N T SCHRODINGER WAVE EQUATION 75
P= i i dr ’ (5.8)
which takes its name from the fact that in the case of a free particle,
its eigenvalue is identical to classical momentum. Indeed, operating
with p on the wave function of a free particle [see Eq. (4.62)], we
have
Thus, to carry out the formal transition from the classical theory
to quantum mechanics, it is necessary to replace the energy E and
momentum p in the classical equation for the law of conservation
of energy 2
2If t h e e l e c t r o n i s in b o t h an e l e c t r i c f i e l d a n d a m a g n e t i c f i e l d , w h i c h i s c h a r a c t e r i z e d
by t h e v e c t o r p o t e n t i a l A, t h e n , u s i n g t h e c l a s s i c a l e x p r e s s i o n ( 1 . 1 5 ) for t h e H a m i l t o n i a n ,
t h e t i m e - i n d e p e n d e n t S c h r o d i n g e r e q u a t i o n c a n b e w r i t t e n i n t h e form
( P~ c ' 1)
Ip = o. (5 .9 a )
0
76 NONRELATIVISTIC QUANTUM MECHANICS
(E „ -H ) ^ = 0. (5.12)
-a r + v -y = o , (5.13)
which involves the charge density p and the current density j. This
equation basically represents a general form of the law of con
servation of charge. To show this, we multiply (5.13) by d'x and
integrate over all space
We shall now use the wave theory to find an expression for the
charge and current densities. For this purpose, we take the
Schrodinger equation (5.3), writing it in the form-
T l - - 2 'i- V * M O + ^ « O = 0. (5.17)
t (o - - I r ( o = o . (5.i8)
Multiplying Eq. (5.17) by ^*(0> and Eq. (5.18) by '}(/), and adding
the two equations, we obtain
P= e**(0'M0- (5-20)
From (5.19) and (5.20), we find the current density
/= { * (0 V ** (0 - r ( 0 V * (/)}. (5.21)
i( /) = e " » h‘^,
P= W (5-22)
and the current density
(5.23)
well (see Fig. 4.3) is zero (/= 0 ). This is quite natural, since
oscillations described by real wave functions are actually standing
waves, and standing waves cannot give rise to a particle flux.
The case of motion of a free particle is somewhat different.
According to (4.60), the wave function for.this case describes a
traveling wave:
ty= L-*/*eiPr"1. (5.24)
Substituting this expression for <[i into (5.22) and (5.23), we obtain
the following expressions for the charge density and the current
density, respectively:
p— = L~3e,
j = 1 ^ j P = *v -
Hence, when the charge is distributed with uniform probability
over the entire volume, its density is equal to the charge divided
by the volume, as was to be expected. Moreover, the relationship
between the current density and the charge density remains the
same as in classical electrodynamics.
Let us now explain the physical meaning of the coefficients Cn
which appear in the solution ^(/) of the time-dependent wave equation
[see (5.4)]. For this purpose, we substitute the expression for <p(0
into Eq. (5.16), which serves to define the conservation of total
charge. Then, using (5.20), we obtain
^ ^ nd*x = 8 „„.,
we have
1n C"C" n (5.25)
2 i
In the second case, the motions of particles, say, in the states« = /i|
and n = n.it cannot be considered independently of one another,
because whenever two states i|»Bl and are both possible, the total
wave function is a superposition of the individual states, namely,
E P% V= T + V .
2 m0 (5.27)
where
S = <
\ 2Tdt. (5.29)
6
dS = 2Tdt.
From the equations
dS OS OS
d S ~ *<“ + % dx- 'r ^ d,J dz, (5.30)
27V// = //i„ (.U - j- ,jl -l- i '2) dt = pxdx - ■pydij + /y/z,
THE TIM E-DEPENDENT SCHRODINGER WAVE EQUATION 81
we find that
p=V S. (5.31)
S (t) = S — Et
— [v S (0 ] 2 + ^ + ^ = 0. (5.33)
S(t) = — E t + p r . (5.34)
The relationship between the wave function and the action func
tion will hold in general form whenever we make the transition
from the Schrodinger equation to the Hamilton-Jacobi equation. We
shall consider further only the stationary case, for whichl
l. $
<b= Aeh * (5.36)
82
NON RELATIVISTIC QUANTUM MECHANICS
= (5’39)
We note from Eq. (5.38) that Eq. (5.36) leads to the same relation
between the momentum operator p and the action function 5 as in
the classical theory if we replace the momentum operator by the
classical momentum p. Substituting (5.39) into (5.37), we obtain
i ( VS)' + | / - £ - ^ = 0, (5.40)
(VS)*>A|v*S|
jr\V -p \< 1.
p = V2mn(E— V),
THE T IM E -D E P E N D E N T SC H R O D IN G ER WAVE E Q U A TIO N 83
yquan = — J lL V*S, ( 5 . 43 )
that E — V(xlt) at the boundary (x = .v0) of the two regions. For the
one-dimensional case, the original equation (5.40) becomes
Taking both sides of the equation, we equate terms that are inde
pendent of h and, sim ilarly, terms that are directly proportional
to h (here it is necessary to bear in mind that the quantity S, is
proportional to tl). We thus obtain
S i= p \ 2S0S'l = ihS'0\ (5.47)
Hence, it follows that
J h i s a p p r o x i m a t e m eth o d of s o l u t i o n i s b a s i c a l l y th e s a m e a s th e p e r t u r b a t i o n m e th o d ,
w hich is a l s o s u c c e s s f u l l y u s e d in t h e s o l u t i o n of t h e S c h r o d i n g e r e q u a t i o n ( s e e C h a p
ter 1 1 below ;.
THE T IM E -D E P E N D E N T SCH R O D IN G ER WAVE E Q U A TIO N 85
In exactly the same way, for the second region (x^>x0), in which
p1<^0, we obtain
.V X
where
\p\ = V2ml>(V - E ).
The wave functions (5.50) and (5.51) are the desired approximate
solutions. From these equations it is seen that, when £ > V, the
wave function is cosinusoidal or sinusoidal, as in the case of a
potential well [see Eq. (4.21)] or a free particle [see Eq. (4.47)],
whereas, w h e n F ^ E , it changes exponentially, as in the case of
transmission through a potential barrier [see Eq. (4.23)].
Comparing the solutions for V = const with the solutions for the
case where the potential energy is a function of .v, we see that the
transition from one case to the other is simply equivalent to re
placing the area of the rectangular barrier contained between the x
axis and an axis indicating the constant quantity * = = LeJ
by the area enclosed between the x axis and the curve for V = V (x).
Schematically this transition can be represented in the following
manner:
cos{z - n
i (5-54)
Therefore, if we set
x„
then
I A* "I/~^Z r ,
Vjf<jr0 — -y=^ y (5.56)
r. (a 0 — x) j 2 Y a
V jc= .a-0 - o ---- A ' j / ^
' 'Ah ■ 3 (5.58)
THE T IM E -D E PE N D E N T SCH RO D IN G ER WAVE E Q U A T IO N 87
Introducing the new variable z = ^O° (x0 — x)3/a (the argument of the
2
If the B essel function is to satisfy Eq. (5.59) and if, at the same
time, the wave function is to obey the asymptotic equation (5.53),
we must put n = ± 4 u- . Thus, instead of the approximate solution
(5.50), we have
|/> | c r . r { c / i /3 ( — J | / ? | d x ) +
X 0
( 7 ^ ^ IP Idx)}> (5.61)
Xq
where / i/a and /_ i/s are the B essel functions of an imaginary argu
ment.
In order to connect the two solutions, we must find the asymptotic
forms of Eqs. (5.60) and (5.61) for the region x — x„. The appro
priate values of z andp are determined from (5.57). For the B essel
function, it is enough to take only the first term of its expansion:
A V « (3a)1/a __ B V *
ft1 /2 r(i/3) (* 0 — *) + r i ‘/ s ( 3 a ) 1/» r ( 2 / 3 ) ’
c /IT ( S a ) 1^ D VK (5.63)
tjf=.«-o+o— fi1 /2 r (i/3) (X— x„) -+ f t l / 2 ( 3 a ) l / “ 1’ ( 2 / 3 ) '
NON R E L A T I V IS T IC QUANTUM M E CH A N IC S
88
D = B, C = — A.
5T h e B e s s e l fu n c tio n of im a g i n a r y ar g u m e n t i s r e l a t e d to t h e B e s s e l f u n c t i o n o f the
firs t k in d by t h e e q u a t i o n
n7T j
/ „ ( * ) = r nJ n {iz) = e *2 J n ( x),
w here
77
/ —
2
x = ze
T h e a s y m p t o t i c form of t h e B e s s e l f u n c t i o n w i t h x = I x l e 1^ c a n b e c a l c u l a t e d e i t h e r for
, 77
d) • 4 0
2
. 77
or for o -----0
2
T~ / l \ 77 , „ 77
1
'O) ^ for 0 = (5.64c)
\/2nz
1 (' , . — ( - 4 )
- \ / for 0 - (5 .6 4 d )
J 2 ttz
^ = \h {A
' ^
cos (* Spdx -
X
)+
+ 5 co s(i J p d * --)} , (5.65)
- J \p\dx
- -7 ^ Ip Idx
-f- (B -)- A) cos 7- e •*« }.
— - 7 $ \p\<ix
• (5-67)
I J I p I dx
^ • = ? T p T e '• • (5-69)
we have
*0
\ p rfx + -) (5.70)
V* < X o --------'
[ge * + he ■
Vp
'n\
( g + / 0 e Xo +
V\P\
(5.71)
l‘ig. 5.2. Q u a n t i z a t i o n in a p o t e n t i a l w e l l a c c o r d i n g
to th e WKB m ethod.
the process of quantization in the WKB method will consist in finding the conditions under
which the exponentially increasing solutions on both sides of the potential barrier (x <C Xi
and .v > .v.) vanish.
According to (5.66), the wave function for this case has the following form in that
region of the potential well which is adjacent to the boundary of the barrier at x = x a
(there being only an exponentially decreasing solution inside the barrier);
a (5.72)
V p M " - V+ ■
THE T IM E -D E P E N D E N T SCH RO D IN G ER WAVE E Q U A T IO N 91
Xl x
a' sin ( 1 J p d x + - ) - « sin ( 1 $ / > ^ + - ) = 0,
* *1
* 2 X
sin ( I J p d x + ^ = 0.
*1
X»
These quantization rules differ from the Bohr quantization rules by the presence of a
nonzero term - j 2 -ft, which corresponds to the lowest state (n = 0). A more exact solu
tion of a similar problem in wave theory (for example, the harmonic oscillator problem)
shows that a zero-point energy necessarily exists, although it does not affect the radiation
spectrum.
Let us now find the normalization coefficient of the wave function for the case of a
potential well. In normalizing the function, we can restrict ourselves to integration over
the Interval X i ^ . x ^ x t (potential well), since '{'decreases exponentially everywhere
outside this region. The normalization coefficient can be then found from the equation
+ f |= ' - <5-76)
Xl Xl
Since the sinusoidal function oscillates rapidly, its square can be replaced by its average
value, 1/2, without significantly affecting the accuracy. We then have
92 NON R E L A T I V I S T I C QUANTUM M E CH A N IC S
O-
The oscillation period --(o> is the angular frequency) is
0)
t-., x«
*1 *1
where v = — is the velocity of the particle. Hence, for the normalization coefficient, we
rno
obtain the expression
Consequently, the eigenfunction (5.73) in the WKB approximation assumes the form
•*i
Problem 5. Lb Determine the eigenfunctions and eigenvalues of the energy of a particle
in a three-dimensional potential well bounded by an infinitely high potential barrier:
(0 for 0 < a: < h, 0 < y < /2l 0 < a<
\ oo outside the potential well.
Find the conditions under which different wave functions correspond to the same energy
value E, that is, the conditions under which the energy levels are degenerate.
Answer.
!n\ ni .
In particular, when /a = / 1, we have degeneracy for the case of the two different wave
functions and
E = En^nin3*
Problem 5.2. Find two classes of solutions (symmetric and antisymmetric) for a
particle in a one-dimensional symmetric potential well:
( l/ 0 f°r x < —/ (region 1)
F (.v )= 0 for — / < * < / (region II) (5.79)
I K0 for x > / (region III)
Show that, when V0 can have any arbitrary value, only the symmetric solution must
always have at least one energy level. Find the condition under which an antisymmetric
solution is possible. Show that the antisymmetric solutions corresponding to ,r;> 0 also
represent a complete solution of the system in the case where V (x) is described by
the function
co for X < 0,
111'-' ( p r o b l e m s a r c b a s e d >n t h e m a t e r i a l c o n t a i n e d in b o t h C h a p t e r s A a n d 5.
THE T IM E -D E P E N D E N T SC H R O D lN G ER WAVE E Q U A TIO N 93
Plot a graph of the potential energy and of the symmetric and antisymmetric wave
functions.
Solution. The wave functions in regions 1, II and III have the form
where
Since y p a— as/a >. 0, the minimum value of the angle a must lie in the first quandrant
and can be found from the condition
1 a
cos a = — -
V 1-)- tan2a
This equation will always have one root for any value of p in the region 0 < a < as is
easily shown graphically from the fact that P = const >- 0.
For the antisymmetric solution (x) = — ^ (— .*)], we h avefia = 0 and Ca = — Di.
The energy levels are determined from the equation
a
tan a = (5.82)
/ Pa—aa
Since the right-hand side of (5.82) has a negative sign, the minimum value of the angle a
must now lie In the second quadrant and can be found from the condition
tan a a
sin a =
y 1 -|-tana a P
Introducing the notation
a + y,
2
It is evident that this equation has at least one root only when
t.e., for Vo
?>T 8 mot2'
Since the wave function of the antisymmetric solution vanishes at .v=0, the antisymmetric
solution for .v > 0 is also the solution for a particle in a potential field described by
<5. SO).
F ig . 5.3. T r a n s m i s s i o n o f a p a r t i c l e th r o u g h a p o t e n t i a l
b a r ri e r .
Solution. The solution of the Schrodinger equation for the various regions has the form;
ty[ = Aiei k x Bie~ikx for x < 0 (region I)
in = Aze~*x + floe** for 0 < x < a (region 11)
= A i e ik'x - a' + D, e - ikix~a> for A r>a (region III)
Here As = ^,”“—, z- = (F0— E), A le‘kx and B\e~ikx characterize the incident and
reflected waves, respectively; A3e, klx~a> characterizes the transmitted wave; and
Ih? ,k v , characterizes the reflected wave coming from infinity. Since we have no
reflected wave from infinity in our case, we must set Z1„ = 0,
To determine the transmission coefficient we shall use the boundary conditions at
a = a and x = 0. We first express An and Bo in terms of Aa, making use of the fact that
' a ^ I:
A o = l 2 ‘n- A ae*a,
Do = — ■A,e~ 0,
/l. e*aAa
where
k
n= —
x E'
Neglecting the second term in the exponential for D (this is possible because the quantity
16n*
Is only slightly different from unity), we finally obtain
- 2± V 2m 0(Va - E )
D = <?
I
oo, xcO,
0, 0 < xC I (region I)
O, } = {■<.(»"‘' * 7 - «»«).
These relations hold for any value of E, and therefore the spectrum is continuous.
We obtain quasi-levels by requiring that there should be only a transmitted wave in
the external region (region III), that is to say, only a wave moving in the positive a- direc
tion. Setting D b= 0, we obtain the following equation for the energy of the quasi-levels:
_ 2-
tan a -f e X
+ / a (5.84)
X - - — — tana------_
Y P8 — a 2 — (a \ / P2 —a2
It should be noted that the amplitude Ct in this case is much smaller than the amplitude
A i:
ICa I~ A I ’
96 N ON R E L A T IVI S T I C Q U ANT UM M E C H A N IC S
For a = oo, Eq. (5.84) becomes exactly the same as Eq. (5.82) which gives the energy
levels of the potential well (5.80).
We let E0 denote the value of the energy of a particle in the potential well which we
obtain from (5.82) and, for present purposes, we disregard all real corrections to E q , as
they make no essential difference to the problem. We then obtain the following expression
for the energy:
(5.85)
where
- ^ V ^ - ^ q-E q) mgV-
X E„
2 '
and where
I6 n2 E
(1 4-n2)3’ - / Vo—£ ’
under the condition*/^ 1.
The presence of an Imaginary part In the expression (5.85) for the energy Indicates
that the wave function decays exponentially with time. The transmission coefficient
through the potential barrier for this case can also be found from (5.85). Indeed, the
squared modulus of the wave function inside the well is
| ij/ Is = const-e~u ,
and thus X, which Is called the decay constant, characterizes the decrease of the prob
ability of finding the particle Inside the potential well. The quantity Xis related to D, the
transmission coefficient for a single collision of the particle with the potential barrier,
by the equation
V
where — is the number of collisions with the barrier per unit time. Hence we obtain an
expression for D:
The same expression for D was obtained in another way In Problem 5.3. In Chapter 6 we
shall obtain It for the general case In which the potential energy is an arbitrary function
of the coordinates.
The constant X also determines the duration of radiation t .= (see also Problem
rad x '
3.4). Therefore, the quantity A£ fix should characterize the width of an energy level.
Chapter 6
F ip . 6 .1 . S c h e m a t i c d i a g r a m o f a p o t e n t i a l b a r r i e r
of an a r b i t ra r y b u t s u f f i c i e n t l y s m o o th s h a p e .
The incident and tra n sm itted w a v e s are rep re
s e n t e d by t h e s o l i d c u r v e , a n d t h e r e f l e c t e d w a v e
by t h e d a s h e d cu r v e .
analyzing the wave in region III, where the solution has the sim plest
form, since in this region there can be no wave moving in the
direction of negative x. According to Eq. (5.70), the solution in
region III has the form
( t I p^ + t )
'I'm= -f- /line (6.3)
where
p = V 2 n k ( C - V (x)). (6.4)
D= hr |
(6.4a)
■Zinc I
ih ( dtj/ (6.5)
/ 2 dx
i t r — l ^ n i l 2* (6-8)
*3
i
5 Ip I dx
X (6.9)
+ me
where
\P I= (V -E). ( 6 . 10 )
- J \p\dx = i — j J \p\dx, ( 6. 11 )
X X \
where
X> X-2
r = 4 $ \ p \ d x = j ^ V2nk ( V - E ) d x , (6 . 1 2 )
xi xi
the solution (6.9) can be put into a form such that it can be connected
to the solution in region I by means of Eq. (5.70). Thus we have
V
- T $ I P 1dx . T $ 1P 1
£i.,e'e
7e +-9ffiiic"1
2 M il '® ** (6.13)
V \ p \ ]
100 nonrelativistic quantum mechanics
P rf-v + t ) pax + t )
+ hie (6.14)
]•
/b = /gni(^ + { ^ ) , (6.19)
to , (‘' - W ( 6 . 22 )
, y 'i n c 1 + - « - ■ ') '
From these formulas, it follows that the sum of the transm ission
and reflection coefficients is equal to unity:
/? + D = l . (6.23)
THE Q U ANT UM THEORY OF C O N D U C TIV ITY 101
For cases where the quantity 7 is much larger than unity (these
are the only ones of practical interest), the transmission co
efficient (6 . 2 1 ) is given by the expression 1
- T } V2m„ ( V - F . ) d x
Dg^e-v = e ** .(6.24)
In s o l v i n g th e s i m i l a r p ro b lem o f p e n e t r a t i o n of a p a r t i c l e th rough a r e c t a n g u l a r p o t e n
t i a l b a r r i e r ( s e e P r o b l e m 5.3), w e o b t a i n e d th e s a m e e x p o n e n t a s in ( 6 .2 4 ), b u t in front of
16n*
the e x p o n e n t i a l t h e r e w a s a f a c t o r , w h i c h w a s of t h e o r d e r of
(1 +« 2) 2 V;
u n ity . In th e c a s e of a s m o o t h b a r r i e r ( t h a t i s , a b a r r i e r e x e r t i n g a f o rc e w h i c h is c o n t i n u -
o u s a t a l l p o i n t s ) , t h i s f a c t o r b e c o m e s e x a c t l y u n ity .
102 NONRELATIVIST1C QUANTUM MECHANICS
Since our equations are valid only for the case /(;> 1 , the accuracy
in determining the momentum of the particle and its position inside
the potential barrier will not contradict the uncertainty relation.
^Ew = \ k T , (6.30)
I h ‘- s p e c i f i c h e a l s o f m o n a t o m i c s u b s t a n c e s w i l l b e c o n s i d e r e d i n g r e a t e r d e t a i l in
f h a p i - T 12 | s e c f o r m u l a ( 12. 06) 1.
THE Q U ANT UM THEORY OF CO N D U C TIV ITY 105
( A iii - A n. = A ; i ;, — I )
• 4k
pm a x •I
2 Art1 A/z.iArtJ
2
j p*dp
U
/>max
(6.31)
or
P m a x = = f l ( 3 ~ , Po)1/3• (6.32)
cm __ p max rr-
^max ■
~ 2mo (3"''>pu)‘/ (6.33)
' ’max
p ,i _2 I P~_ <iaP_ pn (6.33a)
av— Vo 3 2«io Srj-'h3 — 5 L max •
u
In agreement with experiment, it follows that at relatively low
temperatures the electron gas makes no contribution to the specific
heat, since
dE'ky
cel
Tl
df
0 .
,:T n r.-’/e-T2
max 0 ( T ’‘).
THE Q U ANT UM THEORY OF CO N D U C TIV ITY 107
which, as we know, implies the following express ion for the average
energy of a free electron:
E * > E ° av (6.38)
= (6.41)
I'ip. 6 . 5 . P o t e n t i a l e n e r g y o f a n e l e c t r o n in a m e t a l
in t h e a b s e n c e a n d in t h e p r e s e n c e o f an e x t e r n a l
e l e c t r i c f ield .
The d a sh e d line show s the behav io r of the p o ten tia l
e n e r g y c u r v e w h e n t h e i m a g e f o r c e i s t a k e n in to
account.
Metal Vacuum
x
F i g . 6 . 6 . Im ag e f o r c e s : a n e l e c t r o n o u t s i d e
th e m e t a l e x p e r i e n c e s t h e a t t r a c t i v e f o rc e
of the in d u c e d charge.
—dx— —
— _ e ft-L-il
4a.2 — v,
o-j- — 12 y g .
x0 — (6.45)
= — Vel&. (6.46)
Thus, taking into account the electric image force shows that when
an external field is applied, the work function decreases and be
comes equal to
W ' = W — Vefa. (6.47)
- -- Yinu, J W ( x ) — E dx
D= e o (6.49)
The integral in the exponent must be taken over the entire width of
the barrier from x = 0 to the point x = xt given by the condition
Then
Xi X\
So
D —e (6.52)
where the quantity 8 0 depends on the nature of the metal and the
energy of the free electrons inside the metal. The cold em ission
current is proportional to the transm ission coefficient
E. CONTACT POTENTIALS
I *
F i g . 6 . 7 . T w o m e t a l s b e f o r e t h e y a r e p l a c e d in c o n t a c t w i t h e a c h
o ther.
Wj a n d W2 a r e w o r k f u n c t i o n s ; E q j a n d E 0 2 a r e ^ e u p p e r l i m i t s
of the fille d l e v e ls (the F erm i en erg ie s).
Fig. 6 . 8 . T w o m e t a l s a f t e r t h e y h a v e b e e n b r o u g h t into
c o n ta c t. Form ation of the c o n ta c t p o te n tia l.
in both metals are equalized, the electric current ceases, but then
there arises a potential difference proportional to the difference
between the work functions of the metals:
= (6.54)
eo
This quantity is called the contact potential.
6 .9 . O n e - d i m e n s i o n a l K r o n i g - P e n n e y m o d e l o f a
crystal.
We first join solutions (6.55) and (6.56) at the point x — cn -|-a (that
is, atx„ = a), obtaining
/ln3 in!?a-f- Bncoska = — A'„ sinh v.b -)- B,',cosh Ab, (6.57a)
Ancoska — Bnsinka = £ (/1,'jCosh %b — B.isinh %b).
Next, joining the solutions (6.56) and (6.57) at the pointx = c(«-f- 1)
and noting that xn— c and x,Iil = 0 , we obtain
B n = B . l+ l,
(6.58)
A 'n = ± A a+ i .
t.-ba
T" P = const.
Let us examine in greater detail the solution for the case of real
values of k'
r = eik'a.
Substituting the values of /l„and B„into (6.65) and using the fact that
x„ = x — an when b-> 0 , we find an expression for the wave function
in the crystal:
<!»= Cseik’xUn, (6 . 6 6 )
The quantum theory of electron motion In a crystal lattice provides a key to distin
guishing between conductors, dielectrics or insulators, and semiconductors (which In a
sense form an intermediate class of solids). We do not intend to treat this subject
quantitatively and shall confine ourselves to a few qualitative remarks based on the one-
dimensional Kronig-Penney model.
As the starting point of our analysis, we shall use Eq. (6.63) to determine the possible
values of the electron energy in a crystal lattice. This equation Is
p
cosk'a = — sin ka -f- cos ka, (6.63)
where a is the lattice constant, and the quantity k = - L _ — gives the electron energy.
The eigenvalues of the electron energy can be found from the condition that k' must be
real. This condition means that the right-hand side of Eq. (6.63) must be less than unity.
Setting P = co, we obtain the energy spectrum of isolated atoms (In this case the atoms
are separated from one another by an impenetrable barrier). The energy spectrum will
then consist of a separate set of levels for each well:
k' = k = Knta,
F iWW (6 .6 8 )
n 2m0aa •
where n = 1, 2, 3 . .. . We shall not consider the negative values of n, since they give
exactly the same values of energy, but correspond to the motion of electrons in the
negative x direction. The first two levels (n = 1, 2)
-Hi* E, = 4 Ei
Ei- (6.69)
2m0a2
of isolated atoms are shown in Fig. 6.10.
-El
e2
e !
F i g . 6 . 1 0 . T h e f o rm a tio n o f e n e r g y b a n d s in a c r y s t a l l a t t i c e .
If P is finite [see Eq. (6.63)], it is easiest to determine the energy levels graphically
(see Fig. 6.11). The allowed values of k (and therefore of the energy) correspond to the
values of the right-hand side of Eq. (6.63) lying between— land-]- 1. In Fig. 6.11, these
allowed values are denoted by a heavy line. Thus, In a crystal lattice containing N atoms
each energy level of an isolated atom is split into N levels. Each of these groups of
levels is called a band (see Fig. 6.10).
116 N O N R ELA TIV ISTIC QUANTUM M E C H A N IC S
The electrons in the crystal tend to occupy the lowest energy levels and, according to
the Pauli exclusion principle, each level is occupied by at most two electrons with
opposite spins.
For example, the crystal lattice of an alkali metal contains N valence electrons If
there are A' atoms. The electrons occupy only half of the lower band (since there can be at
most two electrons in each level). In
the ground state, half of the electrons
— ha + c o s ka move in one direction, and the other
ka
half In the opposite direction; con
sequently, the average current is equal
to zero. When an electric field is
applied, more than half of the electrons
move in some preferred direction,
thus producing an electric current and
moving up into higher energy levels.
Therefore, In a metal, either the first
F ig . 6 .1 1 . D e t e r m i n a t i o n of p o s s i b l e e n e r g y
band must contain a sufficient number
l e v e l s in a c r y s t a l l a t t i c e . of unfilled levels, or it must come into
T h e h e a v y l i n e s on th e a b s c i s s a a x i s i n d i contact with a second empty band, called
c a t e t h e a l l o w e d v a l u e s o f ka ( a l l o w e d the conduction band.
bands). On the contrary, in a perfect insula
tor all levels in the first or valence
band are occupied and the levels of the second (conduction) band are all empty. The energy
spacing between these bands is usually several electron volts. It is easy to show that,
when a field is applied, the electrons cannot acquire a preferred direction of motion. The
direction of motion of an electron can be reversed only if the electron goes into a different
energy state. But since all the energy levels are occupied, this can only happen if the
electron ‘ occupying that other energy state makes the opposite transition to the state
originally occupied by the first electron. Therefore, on the average, there can be no
preferred direction of motion of the electrons even when a field is applied. A diagram
of the energy levels of a conductor and a dielectric is given in Fig. 6.12.
ao
S-a
*5
^ is
N1) o
§•3 U
■o u
•a id
'o ^ -*■
-9-
-9-
C
---- C----------- -------------------«--------
---- •----------- ----------- «-------- 4) 5
9
------------ 1
-------------------------------- ------------------------------------------------------ _______________ "3ua
---- e------------------------------ a-------- >
---- e------------------------------ c--------
---- •----------- ----------- e--------
Metal Perfect dielectric
1 ig. 6 .1 2 . E n e rg y l e v e l d ia g r a m o f a m e t a l a n d a d i e l e c t r i c .
Nevertheless, every dielectric p ossesses some (very small) conductivity. The con
ductivity Is much greater in semiconductors, where the forbidden band is considerably
sm aller than In dielectrics; it Is of the order of 1 ev, and sometimes even le ss (for
example, in germanium, the width of the allowed band is 0.66ev at T = dOOcK). At absolute
zero, semiconductors behave like dielectrics; however, their conductivity increases with
rising temperature, particularly in the presence of impurities. At room temperature,
the resistivity of semiconductors is found to be of the order of I0~2— 102 ohm.cm. At the
same time, the resistivity of dielectrics lies within the range of 103— 1019ohm.cm, and
that of metals in the range of lO"6— 10~9 ohm.cm.
Ihe conductivity of dielectrics and semiconductors can be of two types. Let us assume
that under the influence of thermal excitation or the internal photoelectric effect (absorb-
tion of light by electrons), some of the electrons Jump from the valence Into the conduction
band, leaving vacant states (“holes” ) in the valence band. The electrons that have jumped
THE QUANTUM THEORY OF C O N D U C T IV IT Y 17
into the conduction band become carriers of electric charge, thus producing a current.
On the other hand, as soon as several states become empty in the valence band, an elec
tric current can also be produced due to the motion of electrons in the valence band
itself. It can be shown that the motion of a system of electrons iiran almost completely
filled valence band can be treated in terms of the motion of a set of vacant states or
"holes." Obviously, the holes move in a direction opposite to the direction of electron
motion, so that they behave like positively charged particles. Thus, there are two ways
In which charge can be transported and, therefore, two types of electric current in solids:
n type ( due to the motion of electrons) and p type (due to the motion of holes).
With the help of the band theory, we can easily give a qualitative explanation of a
number of interesting phenomena.
For example, the conductivity of a metal increases with decreasing temperature
because its resistance is due to the interaction between free electrons and the vibra
tions of the lattice. Since the vibrations of the lattice diminish as the temperature
decreases, the resistance will also decrease. On the contrary, the conductivity of semi
conductors decreases as the temperature is lowered, because the number of electrons
in the conduction band becomes increasingly smaller. The existence of p type conductivity
in semiconductors has been demonstrated in investigations of the Hall effect and thermo
electric emf. The sign of the potentials appearing in these phenomena is determined by
the sign of the current carriers. Experiment shows that in some semiconductors the
sign of the potentials corresponds to electron carriers, whereas in other semiconductors
the sign of the potentials is reversed; that is, the current carriers act as if they were
positively charged particles. A natural explanation for this is provided by the concept
of "holes.”
The above conclusions in regard to the conductivity due to transitions of electrons
from the valence band into the conduction band (the long arrow on the left-hand side of
Fig. 6.13) and due to the motion of holes In the valence band are based on the assumption
of a perfect crystal. This type of conductivity is known as intrinsic conductivity.
Another type of conductivity, known as impurity conductivity, also plays a significant
role in semiconductors. It is caused by the presence of foreign impurities or other
structural defects in the lattice. Such disruptions of the perfect periodicity of the lattice
lead to local deviations of the field from a perfectly
periodic one. As a result, discrete levels may Conduction
appear in the forbidden band of the energy spec band
trum of the electrons (see Fig. 6.13). In practice,
the wave functions associated with these levels
differ from zero only in a certain region near the Donor level
given defect. Thus, discrete levels are sometimes
called impurity levels. They do not themselves
contribute to the current (the electrons occupying Accepter
them are not free). However, they may affect the level
number of electrons contained in both the con
duction and the valence bands. Valence
Impurity levels (denoted by short arrows on the band
right-hand side of Fig. 6.13) can be divided into two
types: donor levels and acceptor levels. A donor
is capable of supplying electrons to the conduction F i g . 6 . 1 3 . D ia g r a m o f e n e r g y b a n d s
band, so that free electrons will appear in the in a s e m i c o n d u c t o r .
conduction band. In contrast to a donor, an
a c c e p to r a b s o r b s e le c t r o n s ; a s a r e s u l t a " h o l e ” is f o r m e d in th e fille d b an d s o th at
e l e c t r o n s in th is b a n d a r e a b le to J u m p in to h i g h e r e n e r g y s t a t e s ( p t y p e c o n d u c tiv ity ) .
If the impurity levels are located sufficiently close to the edge of the corresponding
bands, ionization (or other effects) can arise fairly easily due to the energy of thermal
motion of the lattice. Therefore, an appreciable number of free electrons (or holes) can
exist in bands even under conditions when direct excitation of electrons from the valence
band into the conduction band is highly improbable.
In the case of impurity conduction, the magnitude and type of the conductivity are
determined mainly by the nature and concentration of the Impurities. By varying the
impurities, it is possible to control within a wide range the magnitude of the conductivity
as well as its type ( n type or p type).5 This fact is widely utilized in semiconductor
P u r e g erm an iu m h a s a h i g h r e s i s t a n c e b u t t h e a d d i t i o n o f a s m a l l a m o u n t o f i m p u r i t i e s
may g i v e it e i t h e r n or p t y p e c o n d u c t i v i t y .
118 N O N R EI—ATI VI S T I C Q U ANT UM M E C H A N IC S
A th eo ry of s u p e r c o n d u c t i v i t y b a s e d o n t h e s e c o n c e p t s w a s d e v e l o p e d i n d e p e n d e n t l y
by the A m erican p h y s i c i s t s B a r d e e n , C o o p e r, a n d S c h r i e ff e r , a n d by t h e R u s s i a n p h y s i c i s t
Iiopo 1y u b o v . [ s e e N B o p o ly u b o v , V. T o l m a c h e v , D. S hirk ov, A N e w Me t hod in the Th e o r y
"f Superconduct ivi ty ( t r a n s . ) , New York: C o n s u l t a n t s B u r e a u , 1959.1
See R B e i e r l s , (Quantum r/ieory of Sol ids, Oxf ord: C l a r e n d o n P r e s s , 1955; W. S hock-
l e y , liU' et mns ami Hol es in Semi conductors, N ew York: V an N o s t r a n d , 1950.
C h a p Ic r 7
nx ) = M.f(x), (7.1)
(7.2)
b\Cf — CMf,
the operator is the Laplacian which converts the function f(r) into
another function p(r). Conversely, we can solve Poisson’s equation
and find
Here the operator v~'2has the form of a definite integral, the kernel
of the operator being
l _________1____________
K(r.r') = 4r. | r —r’ | Y(x —x'Y + (y — y Y + (z — z')a *
H = T -fF ,
ft 2
consists of the sum of the kinetic energy operator T = — v2»
which is directly porportional to the Laplacian, and the potential
energy V, which is simply a function of the coordinates.
We can regard the action of the potential energy V (x) on the
wave function as the action of a linear operator, because conditions
(7.2) are satisfied. Therefore, besides differential and integral
operators, the linear operators that may be used in quantum m e
chanics include any function of coordinates whose action on the
wave function is simply to multiply it. For example, the coordinate
r is just as entitled to be considered an operator (the position
operator) 1 as the momentum operator p — — iUy, which is a differ
ential operator. It is worth noting in this connection that in quantum
mechanics r does not represent the position of a particle, but is an
argument of the wave function and determines its value in the
coordinate space. The quantity which is equivalent to the position
can be found from the operator r and the function ip(r) in the same
way as the momentum of a particle is found from the momentum
operator, that is, by averaging (see the following).
T his will b e c o m e p a r t i c u l a r l y c l e a r w h e n w e w r i t e t h e S c h r o d i n g e r e q u a t i o n in m o m e n
tum spue v ( s e e b elo w ), w h e r e t h e w a v e f u n c t i o n d e p e n d s on p. In t h i s c a s e , t h e m o m entum
op' r.it tr p will c o r r e s p o n d to m u l t i p l i c a t i o n by a n o r d i n a r y f u n c t i o n , a n d t h e p o s i t i o n o p e r
ator x to d i f f e r e n t i a t i o n w ith r e s p e c t to p.
ST A T IS T IC A L IN T E R P R E T A T IO N 121
d- d2 d2 d■
axoy
i—= oyox ’
, -x-s-
oyoz
= ozoy
, and, so on.
xPx'!t = — i h x ^ , (7-4)
whereas
= = (7.5)
Consequently,
(PXx — xpx)ty = — ifiy, (7.6)
that is,
pxx — xpx = — ih. (7.7)
The Fourier transform of the function i{i (x) , namely, the function
y(/j)which depends on momentum, is called the wave function in the
momentum representation. Equations (7.9) and (7.10) relate the
wave functions in the coordinate and the momentum representa
tions.
Let us find what operator in the momentum representation
corresponds to the position coordinate x. We note that in this
representation we need not write p as an operator (that is, in
roman type). If we substitute <]i(a:) for AAp(x) in Eq. (7.10), then
m j r = — V I/ ( / ■ ) .
M= J (7.12)
jjVW'MO d3x = l .
The average values of the position and momentum, as has already
been mentioned, are in fact computed in basically the same way:
P * = lr(t)* <7-13>
Here x is the coordinate of the center of m ass of the wave packet
associated with the function •]>((), and p^is the momentum of this
center of mass.
Since the outcome of a physical measurement is a real quantity,
the average values must be represented by real numbers. There
fore, the following equation must hold:
= (7.14)
When this requirement is satisfied, the corresponding operators
are said to be self-conjugate (or Hermitian).
In particular, we shall show that the operator pA . satisfies the
condition (7.14), even though it appears to be purely imaginary. As
a preliminary, we must first prove an important theorem for
“ transferring” a derivative. This theorem is as follows. Suppose
we have an integral
oo
G = ^ uvln’dx, (7.15)
— 00
tw In- 1 )
v\ (7.16)
.1
— CO
M -M 0 = > 4 (0 . (7.18)
we obtain for M
On the other hand, suppose that the operator M [Eq. (7.18)] has
several eigenvalues . . . . X„,. . . , corresponding to the functions
c, (i), o.j(/)....... . . . (f°r example, this may be the case for
the energy operator E = //z~, for which l n= En). Then since the
general solution 6 (/)can be written in the form
*(0 = 2 W O . (7-20)
n
F = ^ \ Cn\*En. (7.21)
(8 . 1 )
(8. 2 )
(8.3)
(W - f ** ( - ih ~ ) 2 * d*x.
/ ( * ) = j ( a x ^ + Q ( a x ^ + ^)d3A-> (8.7)
/ ( ? ) = /I a 2 — fla-fC, (8.7a)
where
A = j •'!>*x‘^cPx = x2> 0 ,
c — )r iii I
( Px =
<)x <)x h* ih L f
AVERAGE VALUES OF OPERATORS 129
/(a)ssO, (8.9)
and
From this it follows that inequality (8.9) will hold for all real values
of a, provided the following condition is satisfied:
B* sg 4AC. (8.10)
( 8 . 12 )
I* ip. H. 1. T h e p r o b a b i l i t y d i s t r i b u t i o n f u n c t i o n in (a) c o o r d i n a t e
s p a c e a n d (b) mom entum s p a c e : [ ( A x ) 2 ( A p ) 2]^3 = fl/2.
If th e d i s t r i b u t i o n i n c o o r d i n a t e s p a c e (a) c o n t r a c t s , t h e d i s t r i b u
tion in m om entum s p a c e (b) s p r e a d s .
i). (8.15)
AVERAGE VALUES OF OPERATORS . 131
H = ^ + V (x). (8.16)
or
dV(x)
m0x dx
. _dH _ . dH_
(8.17a)
X‘ dp i ‘ dxj '
Hence the time rate of change of the quantity f [see (8.15)] is given
by the equation
d f_ d f
dt dt + 2 (&%*'+
Using the canonical equations (8.17a), we obtain
dt dt ‘ 1 ’ /1 ’
(8.18)
dji df_ dM d / \
[ H ,
dpi dxi dxi dp -,)
(8.19)
% = W , /]. (8 . 2 0 )
If the Poisson bracket vanishes ([//, /] = 0), the quantity f does not
depend on time, or it is conserved.
/ = const.
132 NON R E L A T I V I S T I C QUANTUM M E C H A N IC S
I f = J r (0 ^ ( 0 ^ +
t i J I(H r (/)) (f * (/)) - r ( 0 f (H <|. (/))] d3x, (8 . 2 2 )
where
H 2 m0 1
= * TIH. H* (8.23)
AVERAGE VALUES OF OPERATORS - 133
The expression
^ = (H T fT = !- (8.25)
H = const. (8.26)
H= ^ J<d:,x = 2 C„ En= E:
that is, Eq. (8.26) is nothing but the law of conservation of energy
{E — const) for a particle moving in a time-independent field of
force.
C. EHRENFEST’S THEOREM
where
H = 5 S + l'M - <8-28>
Since -v and f lx) commute, Eq. (8.27) can be reduced to the form
* = 2^hW*X- X^ - (8*29>
X= &
IHq
. (8.31)
L = ir a = i = - tx • <8*32>
( ^ < 2 |£ § |. (8.40)
T (~Px) = <8-42>
we have
np7) = np*) + — ( W - (8.44)
From this we obtain the condition under which we can make a
transition from the quantum-mechanical expression for the kinetic
energy (8.41) to the classical expression
(8.47)
£5 6??,
r- ' r' '
AVERAGE VALUES OF OPERATORS 37
3 n2
r
4 m<,el
(8.48)
Ti2
Since -— m0elr = a0l where a0 is the radius of the first Bohr orbit, and
r = n2 a0, we obtain instead of (8.48)
3_
n 4 ’
(8.49)
1 * PoA x
+ Oo, X)
Y~inh 6
To transform to the ^representation, we use Eq. (7.10) in the form
1 C —l Jr*
¥ (Po, p) = - y = J 4(Po, •*■') e n dx\ (8.50)
obtaining
¥ (Po, P) = 6 (P — Po)-
The normalization condition has the same form in both x and prepresentations:
The average value of the operators should be calculated from the equations
Po + AP Po 4- AP
M= J dp'0<\>*(p'„, x) M i (j>o, x) d x = ¥* (Po, P) M ¥ (Po, P) dp.
S dp‘ l
Po — * P Po — * P
which gives us
E= Po
P=Po,
2 m0 ‘
^ T h e p r o b l e m s in t h i s c h a p t e r r e fe r to C h a p t e r s 6 and 7 a s w ell.
138 NON R E L A T IV IS T IC QUANTUM M E CH A N IC S
4it/ft* 2pi
I <f(P) I2 (rfW — pH*Y C0S 2f t ’
which satisfies the normalization condition
+»
j \<t(p) \"dp = l.
— CO
Problem 8.3. Investigate the motion of a charged particle in a constant and uniform
electric field g.
’
Solution. This problem is solved most simply in the momentum representation. Since,
according to Eq. (7.1 i), the potential energy in momentum space can be represented in
the form
V= -e% x = - F x = - F M ^ ,
T (/-. P)
1 J r f a -&■;),
\ r 2 T .h l-'
w h e r e , because of the continuity of the spectrum, the normalization coefficient was found
f r o m the condition for S-function normalization:
CO
^ r* (/•’■P) r U-, /') dp = o (II' —
— fiO
AVERAGE VALUES OF OPERATORS 139
The wave function in the position space can be determined with the help of Eq. (7.9)
cos (-------u; ] da
is the Airy function, which is proportional to the Bessel function of order 1/3 (see
Chapter 5), and 5 = (^x + j *%• Examining the asymptotic behavior of the Airy
function
it is readily shown that the region of large negative values of jt,_where — Fx>- E, repre
sents a potential barrier, whereas the region where E > — Fx is quasi-classical,
because
= £3/s + const.
Problem 8.4. An electron moves in a constant and uniform magnetic field. Find the
time derivative of the average value of the position and momentum of the electron (in
other words, generalize the Ehrenfest theorem for the case of motion in a magnetic field).
Solution, According to Eq. (5.9a), the Hamiltonian of an electron In a magnetic field is
Choosing the direction of the uniform magnetic field to be along the z axis (Hz 0,Hx
= H y = 0), the field can be specified by the vector potential
AX = AZ = 0, Ay = x H z.
In order to determine the time rate of change of the electron’s position, let us use the
quantum-mechanical equation of motion
v = 7f {Hf-rH}’
Substituting H, we readily find
1 / e \ P
v= — ------ A = — . (8.51)
tn0 \ p c / m0
For the time derivative of the x component of the momentum operator, we obtain
(8.52)
Problem 8.5. As we know, the behavior of an electron in a metal (at < 0 ) can be
described with a sufficient degree of accuracy by the following potential energy function
(see Chapter 6):
Determine the coefficient of reflection from the surface of the metal for electrons located
inside the metal (x < 0) in the following cases: (a) E < V0 and (b) E > V0. Show that, even
though in case (a) the electrons do penetrate Into the region (x >. 0), ultimately they return
back into the metal. Construct a graph of the change of the potential energy and of the
wave function of the moving electrons.
Answer.
Hint. In choosing the solution for x > 0 (outside the metal), only the exponentially
decreasing solution should be retained In case (a), and only the solution corresponding to
a wave traveling along the x axis in case (b).
C hapter 9
(9.1)
(9.2a)
C = NnBnn.Khu, (9.4)
r ind = N n .Bn.nKhv, (9.5)
where N„■ is the number of atoms in state n'.
ELEMENTARY THEORY OF RADIATION 143
Dividing by the factor e -en/kT and noting that En— En>= fuo, we obtain
An n ’
~Bnn'
Pa, (9.8)
'* n ’« hm/kT
r = jj (9.11)
r quant = g n g n 'I iu A (9 .1 2 )
S e e filso C h a p t e r 2 ° , w h e r e t h e c o e f f i c i e n t A i s o b t a i n e d b y t h e m e t h o d s o f q u a n tu m
e le < t rody n a m ic s.
ELEMENTARY THEORY OF R A D IA T IO N 145
&&'A^M' = | £ ( r ) * v (9.13)
Let us note that Eq. (9.13) contains two averages. One is the
quantum-mechanical average, denoted by a bar, and the other is
the time average, denoted by the subscript “ av.”
We shall now assume that the electron has only two possible
states with energies En and En>. Then the wave function can be
written as
if(t)= C ne~ h C„.e <|v. (9.14)
(9.16)
From (9.16), it follows that this matrix changes into its complex
conjugate when the rows are replaced by columns, and columns
by rows
rnn' = r„'n.
Here we have used the fact that the time average of a periodic
function is zero, since
In these equations
and so forth.
'Ihus, the energy eigenvalues can be used to find the frequency
of the radiation and the eigenfunctions to find its intensity. Thus,
al! basic classical radiation properties can be completely general-
i *1 to the quantum case by means of the Schrodinger equation.
f rom the last equation above it is evident that the intensity of
radiation will be dilferent from zero only for those transitions for
ELEMENTARY THEORY OF R A D IA T IO N " 147
which at least one of the matrix elements Xnn'y ynn' and Znn' is non.-
zero. These transitions are called in quantum mechanics the
allowed transitions.
It should be noted that in very many quantum-mechanical
problems it is sufficient to calculate the matrix elements alone and
thus to set up selection rules, that is, to find the changes in quantum
numbers that correspond to allowed transitions. From a knowledge
of the selection rules, one can answer the question of possible
frequencies of radiation. In the language of classical electro
dynamics, the selection rules correspond to a specification of the
harmonics at which radiation can be emitted by a given system . If
the matrix elements for a given change (difference) in quantum
numbers are equal to zero, there will be no radiation at the corre
sponding frequencies, and these transitions are said to be forbidden.
Here, in speaking of forbidden transitions, we are restricting the
use of this term to electric dipole transitions. By electric dipole
transitions we simply mean transitions whose probability depends
on matrix elements
(9.24)
1
\V = 180c6 (DabY, (9.25)
Problem 9.2. Find the selection rules for dipole and quadrupole radiation for a particle
in an infinitely deep potential well.
Answer. For dipole radiation An must be an odd number, and for quadrupole radia
tion \u is an even number.
Hint. Using the wave function (4.32), it can be shown that the average value of the x
coordinate is
* = j 'I'n x ^ nd x = - .
xi ’a = V„'(X~x)tyndx.
2“ t r k
where co= — = — is the angular frequency and a is the ampli-
tude of oscillation. From Eq. (10.3), it follows, in particular, that
the acceleration
w = x = — aw1cos u>t (10.4)
^ c l = 2£ (l-)lv = — . (10.5)
— JcosWf==y. (10.6)
6
We shall now express the intensity of radiation U^clin term s of the
total energy E = T -j-V of the harmonic oscillator. From the w ell-
known equations for the potential energy
'Aith this equation, we can eliminate the quantity a'1 from (10.5),
obtaining
:\^pxdx — 2-hn, ( 10 . 11 )
where
dT ( 10 . 12 )
p * = w = m»x-
. dx
Let us substitute pxdx = m„Z-^ dt = sinWcf/into Eq. (10.11).
Then, taking into account Eq. (10.9) and integrating over a complete
period, we find
:n/lm, (10.13)
/ _ my**
1
V— ~ T ~ '
From the graph, it is seen that inside the potential well, where the
total energy E of the harmonic oscillator is greater than V (E^> V),
the solutions for & will take the form of harmonic functions. Inside
the potential barrier (E <^F), the solutions will contain two parts,
152 N O N R E L A T I V I S T 1C QUANTUM M E CH A N IC S
, 2 /;i0 f p \ _o (10.14)
dx"- T r- 2 ) r — w-
Setting
2mji n 1 /n„w a . 2E
h- ’ 1 -vj; ft ’ $ — K ~ tuo ’
(10.15)
we obtain
9" + (>*-54)'> = 0 , (10.16)
where
6" = d~‘'y
y dv •
(10.17)
6 = o. (10.18)
THE LINEAR HARMONIC OSCILLATOR 153
and, consequently,
<]»oo= + C2 e‘^ \ ( 10 . 21 )
«= 2 b^ k- (io -24)
fc=0
6 „ ^ 0 , 6 b+s = 0 . (10.26)
X= 2 n + 1 , (10.27)
and, consequently,
(10.28)
A2 ■e*.
The power series with a finite number of terms obtained for the
function u is called the Hermite polynomial
u = Hn<$ = (2 ( ) * - ^ (2 ; r 2 +
I n(«—l)(n—2 )(n—3) ^g.y, _ 4 | | for odd « (10.32)
6 „for even n
In particular,
tfo(S)=l. /*.(*) = * , //*(;) = 4 ? - 2 ,
//,($) = 8 ?3 — 1 2 '. (iU.dd)
The Hermite polynomials //„(?) can be written in closed form 4
^ T h i s c o e f f i c i e n t c a n a l w a y s be c h o s e n a r b i t r a r i l y , s i n c e t h e n o r m a l i z a t i o n f a c t o r of
the w ave fu nction W i s s till undeterm ined.
a n —^ 2
T o s h o w t h i s , w e i n t r o d u c e t h e f u n c t i o n v = e * , w h ic h s a t i s f i e s t h e e q u a t i o n
v' + 2 £ v = 0 .
D i f f e r e n t i a t i n g t h i s e q u a t i o n n + 1 t i m e s , a n d u s i n g t h e L e i b n i t z form ula
n(n~ l) (n- 2 )
( y z ) (n) y ^ z + ny^ 1 'z ---------- y z + . . . ,
2 !
we o b t a i n
u (n + 2) + 2 f u (n + + 2(n+ l ) v (n) = 0.
M a k in g t h e s u b s t i t u t i o n
y (n) -<f2 W,
w e fin d t h a t t h e f u n c t i o n iv s a t i s f i e s E q. ( 1 0 .3 5 ), a n d t h u s it i s p r o p o r t i o n a l to t h e H e r m i t e
polynom ial
£2 dne~€2
w = es ------------ n •
,ifn
T h e p r o p o r t i o n a l i t y f a c t o r An c a n b e fou n d by e q u a t i n g t h e c o e f f i c i e n t s o f £ 2 n . A s a
r e s u l t , i t i s f o u n d t h a t A n = (—l ) n , from w h i c h w e o b t a i n Eq. ( 1 0 .3 4 ).
156 N O N R ELA TIV ISTIC QUANTUM M E C H A N IC S
x0 C - J r ^ / i - = 1 . (10.39)
we find
____ 1 ____
V ™ y rT ^ ’
that is,
Y'inn\ v~- H n
-Vo (10.42)
THE LINEAR HARMONIC OSCILLATOR 57
- Jt
F i g . 10.2. T h e e n e r g y e i g e n v a l u e s a n d th e
b eh av io r of th e c o rre sp o n d in g e ig e n fu n c tio n s
o f t h e h a r m o n i c o s c i l l a t o r for s m a l l q u a n t u m
n u m b e r s (n = 0, 1, 2). F o r c o m p a r i s o n , t h e
c l a s s i c a l probability d istribution functions
p n a r e i n d i c a t e d by t h e d o t t e d l i n e s .
(10.43)
—00
(10.44)
158 N O N R ELA TIV ISTIC QUANTUM M E CH A N IC S
(10.45)
I'his can b e e a s i l y sh o w n on th e b a s i s of t h e f o l lo w in g s i m p l e q u a l i t a t i v e c o n s i d e r
a t i o n s . Hie p r o b a b i l i t y of fi n d in g a p a r t i c l e a t a p a r t i c u l a r p o i n t c a n b e r o u g h l y c h a r a c
t e r i z e d by th e a b s o l u t e v a l u e o f t h e r e c i p r o c a l o f i t s v e l o c i t y , s i n c e t h e t i m e a p a r t i c l e
s p e n d s in a reg io n w ill b e g r e a t e r in r e g i o n s w h e r e t h e v e l o c i t y i s s m a l l e r t h a n in r e g i o n s
w here it i s g r e a t e r . C o n s e q u e n t l y , Lhe p r o b a b i l i t y o f f i n d in g a p a r t i c l e in a r e g i o n w ith
la r g e r v e l o c i t i e s w ill b e s m a l l e r than t h a t for a r e g i o n with s m a l l e r v e l o c i t i e s . In th e
< a s e of a h a r m o n i c o s c i l l a t o r , w e h a v e from E q. ( 1 0 . 3 ) x / n ~ c o s cot a n d x/coa = —s i n cot.
Taking, th<* s q u a r e of bo th e q u a t i o n s a n d a d d i n g them , we get
(a
and. ‘ h'-P'fore,
1
2
P («2-* 2)
THE LINEAR HARMONIC OSCILLATOR 159
Here we have replaced (Ax)* by x2 and (A/ ; ) ' 2 by p'1. This is justified
by the fact that the wave functions are real and are either even or
odd. Indeed, since the expression <j*xty = xyl is odd, we have
x — ^ y*xtydx = 0 .
Hence
fi 1 2
P= T ^ S dx = 2 ^ | — 00
that is.
E = H = _£l-j.!2s^L f (10.46)
we obtain
P I WqM2(X2)
" 8 /;i„(P“) “r 2
(10.47)
From this it is seen that the energy E cannot vanish at any value
of(x2). Indeed, although the second term vanishes for (x2) — 0, the
first term becomes infinite. Conversely, when (x2) = oo, the first
term vanishes and the second becomes infinite.
Thus, the fact that £mindiffers from zero is directly connected
with the uncertainty relation (10.45) or, in other words, with the
fact that it is impossible to calculate exactly the position and
momentum simultaneously.
Let us find that value of (x2) at which Eq. (10.47) has a mini
mum. Setting the derivative of this function with respect to (x2)
equal to zero, we obtain
m0ti>' ___ft2__
2~
0 ,
8 m0(x2)-
or
(x2) n _ i 3
2 m0co 2 x°'
160 n o n r e l a t iv is t ic quantum m e c h a n ic s
(10.48)
Hence, £ min=
min— ^2 , which is exactly the same as the value for £ 0 found
from the wave theory [see (10,29)].
The existence of a finite zero-point energy of the harmonic
oscillator is one of the most characteristic manifestations of the
wave properties of particles. Thus, the experimental verification
of the zero-point vibrations was of great significance for quantum
mechanics. The zero-point energy £ 0 was first observed experi
mentally in the scattering of x-rays by crystals at low temperatures.
If there were no lattice vibrations at low temperatures ( £ 0 = 0), as
predicted, for example, by the Bohr theory, there would be no inter
action oetween the x-rays and the crystal lattice, and consequently
no scattering would occur. If, on the other hand, the minimum
energy were different from zero (£„ 0) for T 0, the scattering
cross section at low temperatures should approach a finite limit.
Experiments have confirmed that the second situation corresponds
to the true state of affairs and, therefore, the conclusions of the
Schrodinger wave theory are justified.
(10.49)
c +°° c +°°
AVn = *0 {y cT2-; 3 V + lM ^ + 'j'c T ^ J tu'-lM *}' (10.52)
1
x (10.55)
2
where a 0 is determined from Eq. (10.15). For the frequency of the
radiation, we obtain exactly the same expression as in the classical
theory:
where
_ 2m0E _ /« X
a~ ~W •
According to Eq. (5.75), the eigenvalues are determined from the equation
Y Oj?
— [iaA-2 dx = a +
-1
Evaluating this integral, we find that the energy eigenvalues are the same as in (10.28);
that Is, the zero-point energy is also present.
Problem 10.2. Construct the theory of the harmonic oscillator in the prepresentation
(for the one-dlrnenslonal case). Find the equation of motion, the eigenvalues and the
eigenfunctions.
Solution, Since X" — — h- in the p representation, we can write the Schrodinger
dp 3
equation as
p- muu>"h~ d - \
THE LIN E A R H A R M O N IC O SCILLA TO R 163
that is, transforming the wave equation for the harmonic oscillator from the x representa
tion to the p representation and introducing the new parameters
, _ 2C _ p
fiu’ 11 pc’
where
Po = V m 0n u ,
(the prime indicates the derivative with respect to r,). Using the solutions (10.28) and
(10.42), we can write In the p representation
En=fla(n + ^j
and
1 (P y ,
ta(P) = :
V "2” j/npo
It is easily verified that this wave function satisfies the normalization condition
^ I (P) \‘dP = ••
Problem 10.3. Find the eigenfunctions and energy spectrum of an electron (c = — e0< 0)
moving In a constant, uniform magnetic field. Show that, according to the quantum theory,
the "electron gas” must be diamagnetic.
Solution. Let the magnetic field be directed along the z axis (/ft = Hy = 0 J I z =<□%''’).
We can then write for the components of the vector potential A v = xJ%",Ax = — 0.
The motion of an electron is described by the Schrodinger equation (see also Problem 8.4)
ft3 2 , I • p i|/ = 0 .
2 W V* + ‘ m0c dy ~‘ 2m„c2
Since the coordinates y and z do not appear explicitly in this equation, we shall look
for a solution in the form
4, = - L c «'l*ay+*a^) / ( . v ) .
2 /n0 ’ gooj^'
It Is easily seen that this equation has the same form as Eq. (10.14) for the harmonic
oscillator.
Consequently, we can use solutions (10.27) and (10.36) to determine the eigenfunctions
and eigenvalues. We thus find
1
V/l/f gMiy+i'az) e 2
IIn®,
(10.58)
L
164 N O N R ELA TIV ISTIC QUANTUM M E C H A N IC S
2/fl Q* "Jl/|
■7(fi23n(n|) a J ls the normallzation co~
P
[
efficient. 0 = _£Zll_is the frequency of Larmor precession, and
2m0c
The last term in this equation ls simply the kinetic energy of a free electron moving
along the z axis, and is of no special interest.
The first term
En= ^JY-(2n+I), (10.59)
where (jl0 is the Bohr magneton, corresponds to the additional energy acquired by an
electron, in the magnetic field. This additional term represents the energy of electron
motion in the x y plane, which is perpendicular to the magnetic field.
This conclusion ls in agreement with classical theory, according to which an electron
placed in a magnetic field precesses with the Larmor frequency o in a plane perpen
dicular to the magnetic field.
In the classical theory, however, the energy of an electron in a magnetic field is
determined entirely by Its unquantlzed kinetic energy. Therefore, according to the clas
sical theory, an electron gas generally exhibits no diamagnetic properties.
In the quantum theory the energy (10.59) can be interpreted in terms of the appearance
of an additional magnetic moment ]i of an electron, which makes the following contribu
tion to the energy:
£ m agn= _ (1^ . (10.60)
Since the number 2n + I assumes only positive values ( n = 0 , 1, 2, 3, ...), the addi
tional moment of an electron will be directed along the negative z axis. This naturally
leads to the diamagnetism of free electrons In a metal.
It should be noted that in quantum mechanics, solution (10.58) corresponds to harmonic
vibrations along the z axis along, whereas in classical theory the circular trajectory
means that there are harmonic vibrations along both x and y axes, with a phase difference
of—. The reason for this ls that the energy is Independent of the momentumhk». Con
sequently, degeneracy occurs and the solution for a given energy has the form
where the coefficients Ck are arbitrary amplitudes satisfying the normalization condition
In r-'asslcal theory there is also an indeterminacy, since the center of the circular trajec
tory Is not uniquely specified.
Ihe general solution (10.61) corresponds to a set of circular trajectories having
different centers located along they axis.
THE LINEAR HARMONIC OSCILLATOR 165
Concluding the above discussion, let us note that the solution (10.61; includes the
harmonic oscillations along both x and y axes. This can be seen by examining the expres
sion for the energy [Eq. (10.59)] which represents a sum of the energies of two harmonic
oscillators (note that ^oa%^ = o/i)
En — 2fto ^n -f- y ) .
Problem 10.4. Show that the matrix element of the product of two operators M (x)and
N (x), which are independent of quantum numbers, is equal to the sum of the products of
the matrix elements of these operators, that is,
(10.62)
k
Solution. Writing
Problem 10.5. Find the selection rules for quadrupole radiation emitted by the
harmonic oscillator. Find the intensity of spontaneous quadrupole radiation and compare
it with the intensity of dipole radiation. Obtain Eq. (9.24), which relates the intensity
of quadrupole radiation to that of dipole radiation.
Solution. In Problem 9.1 we found that the quadrupole radiation is proportional
to the matrix element, which according the the preceding problem can be written as
CO
Substituting the values of x n,k from Eq. (10.55), we find the following nonvanishing matrix
elements of the quadrupole radiation:
(^2)n-8>rt= 4 Vn { n - \ ) ;
(**)»n,» = 4 / (n + 2) (n + F), (x % n = x*(n + ± ) . (10.63)
That is, the selection rules for the quadrupole radiation of the oscillator are
An= 0 , :t 2 .
The probability of spontaneous emission (n' = n — 2, An = 2) is calculated from Eq.
(9.26). For large quantum numbers when E — nflrn, we have
V^quad = J 6_ gSo>Sga
15 m^c5 ’
where aa = -----= Is the square of the classical amplitude of oscillations. This relation Is
m0 o>a
in agreement with Eq. (9.24).
Problem 10.6. Show that the center of a wave packet composed of the solutions for
the harmonic oscillator moves according to the laws of classical mechanics.
Show that this wave packet does not spread with time. Obtain the transition to the
quasi-classical case ( n ^ m 1, where 2m -|- lis the number of waves In the wave packet).
Solution. Let us assume, for simplicity, that the wave packet Is composed of 2m-)- 1
eigenfunctions of equal amplitudes
1 ("+'+ t )*
I {x, t)
/2 7 + T
J w *
i— — V
where yn+/ are the eigenfunctions of the harmonic oscillator, and u Is the mechanical
frequency of vibration.
The coordinate x of the center of the wave packet is given by
a -= j i/*(x, t)xty(x, t) d x =
V
with
V—I
A= - v i/~2("+ y+i)
2m + 1 L \ 2n + 1 1
x -)- <d2x = 0.
Evaluating a 2 In a similar manner, we obtain (A.v)2= x 2— x 2 for the mean-square
deviation
where
Consequently, (A.v)2oscillates about a certain mean value and, therefore, does not spread
with time.
THE LINEAR HARMONIC OSCILLATOR 167
Zel (11.5a)
r
co
( 11. 6 )
B = V if (11.7)
in spherical coordinates.
Bearing in mind that a gradient
expresses the spatial rate of a
change of scalar field in a certain
di>
direction (Bt = V, <]>= ), we
obtain, in accordance with Fig. 11.1,
dty_
Br
W ’
( 11. 8 )
5, d Fig. l l . l . S p h e r i c a l c o o r d i n a t e s . T h e v o lu m e
pdy r sin Sd-p e l e m e n t in s p h e r i c a l c o o r d i n a t e s .
'.j) (BilS)
V •B = liin J l ----------- I ^ £ (BtdSt) d x u
S-.0 d3x d3x (H.9)
170 N ON R E L A T IVI S T 1C Q U A N T U M M E CH A N IC S
(Xi stands for the coordinates/-, Sand <f), and dS,-stands for elem entary areas perpendic
ular to the directions dr, rdft, and pd<f respectively:
from which we readily find the expression for the Laplace operator in spherical co
ordinate:;'
_ i ___an
sin2 ft d^2.] ‘ (11.13)
_ L i ( r^ _ v 2 (11.14)
r2 dr\r dr) n
and
1 a d\ , 1 a2 (11.15)
sin ft aft \Sln &a»j + sin2 ft d<p2
we have
v*=v?+ (11.16)
+ = (H.20)
Since the left-hand side depends only on r and the right-hand side
only on the angles ft and <p, this equation can be satisfied only if
both the left- and right-hand sides are separately equal to a con
stant X, called the separation constant.
We, therefore, obtain the following equations for the radial and
angular parts, respectively:
V t f + (*' — £ ) /? = 0, (11.21)
Vi,,py + X7 = 0. (11.22)
The important point to note is that the angular part of the wave
function does not contain the variable r and is independent of the
specific form of the potential energy V. Consequently, as we men
tioned at the beginning of this chapter, the angular solution will be
valid for any central force.
Using the method of separation of variables for the angular
part alone, we set
y = e (&) <d (<p), (ii.2 3 )
and thus obtain the following equations for the functions 9 and©:
(11.24)
Vi 0 + (X- l i ^ T ) e = 0’
V£©-}-m3© = 0. (11.25)
V§: I
sin 9 d% sin (H .2 6 )
V2 . - - (11.27)
f df’
[ 1 . (11.30)
o
The particular solution for the azimuthal function [see Eq.
(11.25)] can be written in two ways:
® = Ceimf (11.31)
or
(I) = A cos (mcp ?n)- (11.32)
If follows that
CIirm
M O T I O N OF A P A R T I C L E IN A C E N T R A L L Y S Y M M E T R I C F I E L D 173
m = 0, dr 1, ± 2 , i t 3......... (11.35)
l
From the normalization condition (11.30), we find C It
Vfc'
is readily shown by direct calculation that the functions
x = cos 0 (11.37)
s = dLrn.
s = m ^ 0. (11.42)
u= 2 a»xk- (11,44)
*=0
Substitution of this polynomial into Eq. (11.43) gives
l = q-\-m, (11.48)
we find that, just like the numbers q and m, this number can
assume only positive integral values (including zero), that is,
1= 0, 1, 2, 3......... (11.49)
l^m . (11.50)
where
(1 — * V + 2*it/ = 0, (11.55)
which is easily obtained by taking the first derivative of v with
respect to x. Differentiating Eq. (11.55) with the help of Leibnitz’s
rule [see (10.34a)] and setting
»< = = (11.56)
from which, with the help of Eq. (11,39), we find the following
expression for the function 9:
$T = C?P?{x). (11.60)
P?{x) = { \ (11.61)
pr w = ( - i r f r f <n -62>
m = 0, ± 1 , dr 2 , . . ± /;
i q+w)i (\-x*)l ^ r ( x * - l ) ld x = \.
( 2 '/!)! (/ — ;n)l
(l\f 22,+1
\ (1 — x*)l dx
(2 / + 1 )! ’
we find
2B e c a u s e o f t h e l i n e a r r e l a t i o n s h i p b e t w e e n P/m a n d P[ m, many a u t h o r s p r e s e n t t h e
s o l u t i o n for t h e f u n c t i o n 0 in t h e form
0 = c p j ml (x) .
We s h a l l n o t u s e t h i s form, s i n c e in t h i s c a s e t h e r e c u r r e n c e r e l a t i o n b e t w e e n t h e a s s o
c i a t e d L e g e n d r e p o l y n o m i a l s i s more c o m p l i c a t e d t h a n for t h e s o l u t i o n ( 1 1 .6 0 ) ( t h e r e c u r
r e n c e r e l a t i o n i s im p o rt a n t in c o n n e c t i o n w ith th e s e l e c t i o n r u l e s a n d t h e s o l u t i o n o f th e
Dirac equation).
178 N O N R ELA TIV ISTIC QUANTUM M E C H A N IC S
Then
9 ? = Y 0 V + ^ m)' K M - (11-99)
For the spherical harmonic Yr(&, <p), which satisfies Eq. (1.22),
the relations (11,23), (11,36) and (11.66) yield
To p r o v e t h e o r t h o n o rm a lity c o n d i t i o n ( 1 1 .6 8 ), w e s u b s t i t u t e t h e e x p r e s s i o n ( 1 1 .6 7 ) for
th e s p h e r i c a l h a r m o n i c s i n t o ( 1 1 . 6 8 ). I n t e g r a t i n g o v e r t h e a n g l e 9 , w e c a n r e a d i l y s h o w t h a t
1 /r 27rcHm-m,)? d,. g ,
r / mm
27
%/o
I n t e g r a t i n g the L e g e n d r e p o l y n o m i a l s o v e r the a n g l e 5 , we c a n s e t tti1 ■ m. T h e n w i t h o u t
l o s f g e n e r a l i t y , we r u n l a k e / ' / . T h e c a s e I I w a s c o n s i d e r e d a b o v e in c o n n e c -
te- with t h e d e t e r m i n a t i o n o f t h e n o r m a l i z a t i o n c o e f f i c i e n t . In s i m i l a r f a s h i o n , it c a n b e
r e a d i l y sh o w n t h a t for I ^ I t h e i n t e g r a l ( 1 1 6 8 ) v a n i s h e s a s a r e s u l t o f t r a n s f e r r i n g t h e
d e r i v a t i v e from th e f u n c n o n with s u b s c r i p t I to t h e f u n c t i o n w ith s u b s c r i p t I*.
MO TION OF A P A R T IC LE IN A C E N T R A L L Y SY M M ET R IC F IE L D 179
LxLy — LyLx = (ypz — zpy) (zpx — xpz) — (zpx — xpz) (ypz — zpy).
L*= Li + m - L J (11.77)
Multiplying Eq. (11.78) by y and Eq. (11.79) by^— -rj, adding these
products, and remembering that Y = xi -\-y1, we obtain the relation
2 S - x S = c o s ? S - sin?cot&<|- (11-80)
± iL y = flex i f (11.85)
Y 1— x 1 df '
MO TION OF A PA R T IC LE IN A C E N T R A L L Y SY M M ETRIC F IE L D 161
7 1 = » r, + >/'ITrT’ s ) e" ,( l
1+m
— ei>(m+l)(l 2
it follows that
L'Y? = [y (L * + * L v) — *L y) + i (Lj. — i L ,) X
Applying the quantization rules, we find the discrete values that the
angular momentum can assume
L‘ti=n *h \ (11.90)
where
f
perpendicular (Fig. 11.2c). It is obvious that space quantization has
a meaning only when there is some preferred direction in space, for
example, the direction of the magnetic field intensity vector . 4 If
there is no preferred direction, the orientation of the z axis may
be taken as perpendicular to the plane of the orbit.
4It i s , o f c o u r s e , u n d e r s t o o d t h a t In th e p r e s e n c e o f a m a g n e t i c f i e l d t h e c e n t r a l
sym m etry i s d i s t u r b e d .
184 N O N R EL -A T IV IST IC Q U ANT UM M E C H A N IC S
(H.95)
In the wave theory this case corresponds to m = l, when
(n .9 6 )
whereas
Lqm= W + nn = Ljmax + m. (n .9 7 )
The appearance of the additional orbital angular momentum Kll is
related to the noncommutativity of the angular momentum opera
tors L*, Ly , and L2, as a result of which the angular momentum
components cannot have simultaneous definite values. Therefore,
when I'z = k-smix=fil, the components Lx and Ly do not vanish but
have certain minimum values satisfying the relation
+ (‘^ Jmin4~(A^>’)mln. (11.98)
The minimum value of (ALxf and (ALyf may be obtained with the
help of the uncertainty principle [see (8.13)]:
(H . 1 0 0 )
and the sum of (ALx)9mln and (ALv)amin is exactly equal to the addi
tional angular momentum hH.. As a result we arrive at Eq. (11.97).
Thus the nature of this additional term is the same as that of the
zero-point energy of the harmonic oscillator. Both are related
to the uncertainty principle. For large values of the orbital angular
momentum quantum number I , we can neglect the term tl2l in (11.96)
in comparison with h'H2, so that in fact we have the Bohr sem i-
classical solution.
Chapter 12
The Rotator
Py — (12.4)
and, consequently,
nW
Enf — -1— (12.5)
2J ’
V ? R ( r ) ^ [ ^ - -l ^ - ] R ( r ) = 0. (12.6)
We have here set the potential energy equal to zero and substituted
a = / (Z—
1—1) in accordance with Eq. (11.51). Since r = a = const for
the rotator, function R (r) = R (a) = const, that is, ^2
rR(a) = 0. The
energy E, is now easily found from Eq. (11.51)
Comparing this equation with Eq. (12.5), we see that in the Bohr
theory £^wz;, whereas in quantum mechanics E ~ I ((-)- 1). As has
already been mentioned in Chapter 11 (Section D),this difference is
due to the noncommutativity of the components of Lx, Ly and Lz of
the angular momentum operator; it is one of the characteristic
features of quantum mechanics. Both equations become identical
only for large quantum numbers, that is, when /*;>/.
According to Eq. (12.7), the energy of the rotator depends
only on the orbital angular momentum quantum number I. The
magnetic quantum number m, which characterizes the projection
of the angular momentum L on the z axis (and, consequently, the
orientation of the angular momentum in space), does not appear in
the expression for £,. The eigenfunctions Yf corresponding to the
eigenvalue E, [sec (11.67)] do, however, depend on m. Since m can
vary from / to / [see (11.50)], each energy eigenvalue Et will
have 2/ ; ] corresponding mutually orthogonal eigenfunctions; they
describe the state of the rotator and differ only in the orientation
of ;heir angular momentum L relative to the z axis. In this case
tin1 energy level / ; is said to be (2/ -]- l)-fold degenerate.
in general, a state of a system (or a given level) is said to be
.‘.- .'old degenerate if N linearly independent eigenfunctions corre
spond to the given energy eigenvalue.
THE ROTATOR 187
<12-8)
|y
» si2=
ui 4J-
^ . (1 2 . 9 )
y; = j / | T c o s p (I2 .li)
It can be seen from Eq. (12.9) and Fig. 12.1a that, for a rotator
in the s state, the angle?, which gives the direction of the angular
momentum L relative to the z axis, is arbitrary. This was to be
expected since the angular momentum L2= hH(l-\-1) is equal to zero
in this case. A material particle at rest has an equal probability
of being at any point on the spherical surface of radius a. In other
words, all positions of the rotator are possible. There is no cla s
sical analog of this state.
From Eq. (12.13) and Fig. 12.1b it follows that in the p state,
when / = 1 and m = J 1 , the most probable of all the possible orbits
of the rotator is the one located in the xy plane. The states with
m— \ and ni — -1 will have different directions of rotation: for
m— 1 the rotator will rotate clockwise (the angular momentum L is
parallel to the z axis); for m = — 1 it will rotate counterclockwise
(the angular momentum L is antiparallel to the z axis). When
/ = ] and m = 0, the most probable orbit of the rotator lie s in a
plane passing through the z axis [see Eq. (12.14) and Fig. 12.1c].
In this case, the orientation of the angular momentum is perpen
dicular to the z axis.
It is worth mentioning that a sim ilar analysis of the angular
part of the wave function applies to all system s characterized by
central symmetry.
THE ROTATOR 09
B. SELECTION RULES
( 12 . 22 )
cos&Kf = AY?+l + BY?_lt
sin = A± Y? ± + B±Y?±}, (12.23)
1T h e c o e f f i c i e n t s A a n d B c a n b e fou n d in a fa i rl y s i m p l e w ay . We s u b s t i t u t e th e
e x p a n s i o n (1 1 .6 7 ) in to Eq. ( 1 2 .2 2 ), s e t t i n g
m
pm (2l y __ (1 _T 2 ) 2 j.r '~ m x i-m-2 ,
1 2‘I'O-my. } 2(21-1)
T h en , d i v i d i n g a ll t h e te r m s by r?,m<p(1 —x 2)m a nd e q u a tin g the c o e f f ic ie n ts of x * m+I
a n d the co effic i e n t s o f x l r n ” 1 on th e le f t- a n d r i g h t - h a n d s i d e s ( n o t h i n g f u r t h e r i s o b t a i n e d
by e q u a t i n g th e c o e f f i c i e n t s of t h e r e m a i n i n g p o w e r s o f x), w e fin d
(J f 1 - m ) ( m t m) J V*
A (I, m)
V (21 t 1) (2/ I 3)
li (I, m)
T (2i + l ) ( 2/ - l )
( 1 2 . 2 2 a)
S imilarly , we find
A ( (/, m) + J iilig l
T ( 2 / + 1) (2/ h 3)
(12.23a)
H±(/,m) f
f (2/( 1)(2/-1)
THE ROTATOR 191
We note that these selection rules for the quantum numbers m and
I will hold for any centrally symmetric system including, in
particular, the hydrogen atom.
From the selection rules, we can find the possible emission
(or absorption) frequencies of the rotator:
U = (1.2.34)
u>(,;_).!= — y ( J + l ) , (12.35)
where the frequency corresponds to a transition from a higher
energy level to a lower one (a downward transition) and ^,;+i to an
upward transition.
msum = -j m i- (12.36)
— = y — (12.37)
mred " ,nl
where x, and x.2 are the coordinates of the first and second particles,
respectively, and x.2— x, is the distance between them. Introducing
the relative coordinate x = x2 — xt and the coordinate of the center
of mass
we obtain
if : fllsum-v c.m.
- ~ V (x)
d OX d% n
dt dxi dx[ U’
(12.41)
THE ROTATOR 93
V(r)
F i g . 12.4. P o t e n t i a l e n e r g y d i a g r a m for a d i a t o m i c
m olecule.
Keeping only the first three terms of this expansion and noting
that the function V has a minimum at the point r = a [F'(a) = 0and
F"(a)>0], Eq. (12,45) can be reduced to the form
1/ / r\ i ^ ^X*
2 (12.46)
THE ROTATOR 195
Noting that
„ .n d‘R I 2 dR _ I d*(rR)
v ~ </r3 i~ r dr ~~ r dr2 ’
(12.48)
rR = u (12.49)
This equation is exactly the same as Eq. (10.14) for the harmonic
oscillator, and, therefore,
E’ = tu»(k-\- (12.52)
where k = 0, 1, 2, 3,...
2
T h e d i s s o c i a t i o n e n e r g y (~D) i s d e f i n e d a s t h e work r e q u i r e d to s p l i t t h e m o l e c u l e
in to a t o m s ( n e g l e c t i n g t h e v i b r a t i o n a l e n e r g y ) . T h i s e n e r g y i s u s u a l l y o f t h e o r d e r o f
se v e ra l electron volts.
196 NON R E L A T I V I S T 1C Q U ANT UM M E CH A N IC S
E= — + l ) +/ t o( * + 7*). (12.53)
The first term here represents the dissociation energy, while the
second and third terms describe the rotation and vibration of the
molecule, respectively.
We note that the molecule has only a finite number of discrete
energy levels, since it is dissociated when
(,/ = ~ l,'±D
£„ = £ „ -|-£ A+ £„ (12.56)
Since the main part of the radiant energy is now due to the electronic
transition n-*n' in the atom, the quantum numbers k and / may
either increase or decrease
where
wlt c = Bl (I + 1) — BT (/' + 1). (12.62)
= (12.61a)
6 7 8 9 W it 12 13 « IS 16
U)
Si3 2 < 0 Q!23’4 S' 6 7 8
d)0
0 1 2 3 «
Id*
F i g . 12.6. M o l e c u l a r b a n d s p e c t r a : k>+ — t h e
p o s i t i v e b r a n c h : cj — t h e n e g a t i v e b r a n c h ;
— the zero branch.
Problem 12.1. 3 Find the explicit form of the spherical harmonics for the cases when
the orbital angular-momentum quantum number is / = 2 and / = 3. Verify their ortho-
normality by direct calculation. Plot a graph of | Y™ |2 for m = 0, ± 1, ± 2 .
Answer.
^ = / - 4 7 ( t c0S,9- t )'
F f 1= -± ^ sin a cos ie ±
yt 2=Jr V ^ sinS9e±2,?-
Problem 12.2. Find the eigenfunction of the operator Lv, given that its eigenvalue
is zero and the orbital angular momentum quantum number is / = 1.
Solution. Let us look for a solution of the equation for/== 1 and
Ljf'l' = 0
in the form
J sin i d M - f = 1,
we find
Problem 12.3. Investigate the general form of the motion of a free particle In spherical
coordinates. Determine the normalized functions for / = 0(s states).
Solution. The Schrodinger equation for a free particle in spherical coordinates Is
written as
1 rf drR Id M)
r dr + *’ R = 0, ( 12. 66 )
dr
where
2
/I3 •
T h e p r o b l e m s in th is c h a p t e r a p p l y a l s o to C h a p t e r 11.
THE ROTATOR 201
The solution of this equation is a Bessel function of half-integral order. Since the
wave function must remain finite as r — 0, we retain only the Bessel function of the first
kind. Then
const
R 1 (Hr),
W r '+ 2
and for a free particle with a given energy the general solution of the wave equation in
spherical coordinates can be written as
( 12. 68 )
R (k, r) = — sin kr.
Since the spectrum is continuous, this expression must be normalized by a 8function
00
^ R* ( k \ r) R (k , r) rV r = 8 ( k - k ’).
o
Hence, we find:
Problem 12.4. Find the selection rules for quadrupole radiation from a rotator.
Solution. It Is necessary to find the transitions for which the matrix element of the
quadupole moment
Since the matrix element of a bilinear combination can be expressed as the sum of the
products of linear matrix elements (see Problem 10.4),
m " , I"
and, moreover. It follows from Eqs. (12.24), (12.25)and (12.26) that the only nonvanishing
linear matrix elements are of the form
^ ± 1 . ^ + 1 J - ± 1, m — 1 J -± m
m * V, m » m »
we obtain the following selection rules for the quadrupole radiation from a rotator:
lm = 0, ± 1, ±2; A/ = 0, -±2.
202 N O N R ELA TIV ISTIC QUANTUM M E CH A N IC S
As we shall see later, the parity of a spherical harmonic is determined only by the
value of I and Is Independent of m (see Chapter 17). Consequently, for dipole radiation,
the only allowed transitions are from an odd state to an even state or vice versa (A/ = i 1),
whereas for quadrupole radiation, the transitions can occur only between even states
or between odd states.
£, = ( / + — ) \ e‘y*P,(x)dx.
- I
Substituting the expression for the Lengendre polynomials as given by Eq. (11.59) and
transferring the derivative from the function ^ 2 _ |j( to the function e1?*. I times (that
is , integrating by parts I times), we obtain
1
5/ = 277 (/ + -Jrj ily l ^ (1 — x2)lc'vx dx.
we determine the coefficients /!,. The desired expansion of a plane wave in terms of the
spherical waves Is now written as
Let us emphasize that the plane wave e,kz satisfies the Schrodinger equation for a free
particle
VJI]; + *,| = 0
and, therefore, the right-hand side of Eq. (12.83) also represents a linear combination
of particular solutions of the above equation written in spherical coordinates. This
explains the expected proportionality between the coefficients B1 and the Bessel functions
Jl 4_ i/, (&r)-
y —_ (13.1)
r
(13.2)
2 04 NONRELATIVIST1C QUANTUM MECHANICS
Z e 0 . h 2 l (I + l)
2m0rz
T h e d a s h e d lin e s h o w s th e b e h a v io r o f th e w a v e function.
p; ,, ( Z el , p$ (13.3a)
'' I ' 2mnr2
Since p c o n s t a n t fo r c e n t r a l f o r c e s , w e m a y w r i t e
v Zel Pi
‘'off
2 m 0r
where
P = 2 l/A r , (13.6)
^ + + + = (13.7)
(13.8)
which gives
p. (13.10)
R = R J t vu. (13.14)
In this case
According to (13.15),
and, therefore,
P«" + 12 (/ —
J—1) — p] [-~ f= — I — l] « = 0, (13.17)
J ^ = A+ Z+ 1 = «. (1 3 . 2 O)
1= 0, 1, 2, 3,...
and the radial quantum number
k = 0, 1, 2, 3,... (13.21)
av(£ — v) = — av+i (v — 1) (v —
J—2/ —
j—2). (13.23)
208 NON R E L A T IV IS T IC QUANTUM M ECH AN ICS
where p= 2 y rA r.
Recalling that -y= = n, and substituting the
VA
value of B from Eq. (13.5), we find
22
—
na0 r (13.27)
fl'i
where a0= — m0elT is the radius of the first Bohr orbit. CalculatingD
the coefficient Cnl from the normalization condition (see below),
we obtain
Cnt - (\na„)
Z \'h VV n (n—I—41)1(n + /)! (13.28)
As we know, the normalization condition for the radial part of the wave function is
J dr = 1.
7a
Now let us represent one of the polynomials as a series (13.24), leaving the other
polynomial in the closed form (13.25). The normalization condition then becomes
Applying the theorem for the transfer of a derivative (see Chapter 7, Section C) to this
equaUon, we find
CO
C °)3J
*, ( — <rP [(* + 1)! P2' +fc+2 - M k (2 1 + k + 1) d ( = I.
It is obvious that the remaining terms in the series representing the function Ql vanish,
since the order of the successive derivatives is higher than the corresponding powers
of p. Using the well-known integral
OO
\ e-Pp4dp = s!, (13.29)
we obtain the expression (13.28) for C„i . In a similar fashion, we can determine the
average value of (r~v) ^ = 1, 2, 3, 4), which will be useful in the later development:
CO
(r v) = £ ^ , mr - ^ „ i m d 3x [ * !//-’+* dr.
u
On the basis of the above equations we can rewrite this as
Setting •v = 1, 2, 3 and 4 in the last expression, and using once again the theorem for
the transfer of a derivative, we obtain after a few simple operations
1
(r-2) = (\ oo
- )/ ’ -n3(/ + V.) ’ (13.29a)
1
fr"*) _ Uo) «»/(/+*/.)(/+ 1) •
J - f i - V ______ 3n* — 1(1 + 1)
* 1 2n> \ a j '.) (/+ !)(/ + '/.)
I n calculating r~l, we retain only the leading term pft in the polynomial Q j . On the
contrary, in calculating r~s, we retain only the last term p». For r ' , we retain the last
two terms, and so on. The expressions for r~s and r~* are obtained under the assump
tion that^ / + 0. For the s states (1 = 0), Interactions which are usually proportional to
either r~B or r-* are generally replaced by a contact interaction (see below. Chapters 19
and 20).
Substituting the explicit form of the wave function ^100 and bearing
in mind its independence of the angles 0 and <?, we obtain
<X>
p 2Z r
The same value for C can also be obtained from the general equa
tion (13.28) by setting n = 1 and ( = 0. Therefore, the wave function
for the ground state is
THE THEORY OF THE HYDROGEN—LIKE ATOM 211
Zr
(Jo (13.32)
n —I I n—\
2 2
1=0 rn=—l
i = 1=0
2 (2/+ 1)==/z’-
3In particular, as we shall see later, the degeneracy with respect to vanishes even in
I
the case of the hydrogen atom if we take into account the relativistic effects, the nuclear
volume, and the so-called vacuum corrections. Similarly, in the spectrum of alkali metals,
which have one valence electron in the outer shell, the influence of the electrons in the
inner shell removes the degeneracy with respect to I.
212 NON R E L A T I V I S T I C QUANTUM M E C H A N IC S
n2ao f r \ (13.35)
'max
min ±v
The equation of an ellipse in polar coordinates is
r = ■1 +, pe ---- ,
COS tf ’ (13.36)
rmax — a (1 + e ) ,
(13.37)
rm i n = n (> - E)-
nyo
= a, (13.38)
Pi (13.39)
iPh1
Substituting into (13.39) the Bohr value of P%= 1Rn%= Hl (l- \)2
and then the quantum-mechanical value of P? = (I -f- 1), we obtain
(13.40)
e = ] / 1— IU + (13.41)
quan f n-
cmin= l / i . (13.42)
quan V n
j r'lR%,dr = 1,
C
l
the distribution of radial probability density D{r) is
D(r) = rR'„i. (13.43)
214 NON R E L A T I V I S T I C QUANTUM M E CH A N IC S
and, therefore, we find the following expression for D (r) (see Fig.
13.2):
_ 2Zr_
D (r) = const r ne n00 ■ (13.44)
we obtain
rn= a = " ^ c i<). (13.45)
F i g . 13 .2 . R a d i a l p r o b a b i l i t y d e n s i t y d i s t r i b u t i o n f u n c t i o n
in t h e c a s e o f c i r c u l a r o r b i t s .
(13.48a)
4 T h i s i n t e g r a l c a n b e e v a l u a t e d by i n t r o d u c i n g t h e n e w v a r i a b l e p = — ( — + — \ Then,
I t± l . a° ^ U '
expressing Q a n d Q^/ in t h e form o f p o l y n o m i a l s , w e c a n p erfo rm t h e i n t e g r a t i o n term by
term. It i s f o u n d t h a t o n ly t h e i n t e g r a l
co
^ r2Rn ,l Rn l dr = 0 (*V n)
u
v a n is h e s b e c a u s e of the orthogonality condition.
216 N O N R ELA TIV ISTIC QUANTUM M E CH A N IC S
it is easily shown that it does not vanish for any value of n'\ that
is, for all allowed transitions the principal quantum number can
change arbitrarily.
E, ev
T h e w a v e l e n g t h s c o r r e s p o n d i n g to t h e i n d i
c a t e d t r a n s i t i o n s a r e e x p r e s s e d in A n g s tr o m
u nits.
(— -nL) = ^ (13'49)
(13.51)
In the case of the hydrogen atom(Z = l), the Lyman series (see
Fig. 13.3), which corresponds to a transition to the lowest energy
level n ' = 1 (the Is level), is given by
P aschen ^ f 32 /Is ) 1
(13.56)
where n — 4, 5, 6 ... .
218 N O N R ELA TIV ISTIC QUANTUM M E CH A N IC S
where
u = Rr. (13.58)
V (/■= oo) - 0.
2 ^ ( 0 - ( / + y ) V w) X= 0, (13.60)
to which the WKB method is applicable. For the argument z which
determines the asymptotic behavior o f [ s e e Eq. (5.55)], we obtain
THE THEORY OF THE H Y D R O G EN -L IK E A T OM 219
/ ( / + l ) - > ( / + i - ) 2. (13.63)
c 2 m0Zel
a 2m0E
fta ' ft2
into Eq. (13.64) and introducing the notation
we obtain
ro
z i I \ p 'd \n r^ +
i-i - 7 + n* +
ln — (* + ■§■) ^r + const.
'+ 2 -i-'I' + i
On the basis of Eq. (5.51), the asymptotic solution for this case
has the form
z = F f f l (C'<r' + i V ')’
where
|p| = *(' + y)
R = -yr <13-67)
we find
R = Cr‘ + Dr-{,+1).
R = Crl, (13.68)
P = rhk. (13.70)
2maE . 2m0Z e l
«= b = —V JL
(13.71)
— ('+4 )’ .
Then, since V7 ***kr-\-y, we obtain at r->co
11 + T+ l[t + 2kr
zm k r-j-y- In ■ -rr + Tln (13.72)
I" T+ i *+ T =
= In ] A 2 + (/ + -)* + arc tan (13.73)
In
F ro m a m ore a c c u r a t e c a l c u l a t i o n (a c a l c u l a t i o n o f t h e a s y m p t o t i c e x p r e s s i o n for th e
ar g u m e n t z from a n e x p a n s i o n o f t h e c o n f l u e n t h y p e r g e o m e t r i c f u n c t i o n ), t h e f o l lo w in g
v a l u e i s o b t a i n e d for t h e p h a s e 5^ ( s e e P . M. M orse a n d H. F e s h b a c h , Methods of T h e o
re tical P h y s ic s , P a r t II, N ew York: M cG raw -H ill, 1953):
®I = -argT (/ + 1 4 iy) .
If |/ + i y | ^> 1, t h e n u s i n g S t i r l i n g ’s formula
r o + / + iy) = i r o + / + i r ) | e ~ iS/° = .
we o b t a i n for t h e a p p r o x i m a t io n ( 1 3 . 7 4 a ) S®.
222 N O N RELA TIV IST1C QUANTUM M E C H A N IC S
C sin
[kr~ l l) (13.77)
R?=0) (k) = ------
= dr-f-
ro
r _____________________________________________
C = j / ' |. (13.79)
pi on
^-min= /? ^ = —
LQo
- = 13.59 ev.
1+ M
The Rydberg constant then becomes
("0 = — (l - % ) . (13.82)
which differs from the previous one [see Eq. (13.52)] by the factor
but that each of these lines is associated with a second line situ
ated somewhat to the right. This second series of lines (see Fig.
13.5) is described by the equation6
B a lm e r n fi 1 \ /I 1\
cop = X « ’[1 (13.86)
A c c o r d i n g to t h e l a t e s t e x p e r i m e n t a l d a t a ,
w h e r e t h e n u m b e r s r e p r e s e n t t h e v a l u e s o f R s p ( s e e f o o t n o te , p a g e 20).
226 N O N R ELA TIV ISTIC QUANTUM M E C H A N IC S
»?■"”" = (13-87)
Another very important consequence of accounting for the
motion of the nucleus was the discovery of ionized helium, first
detected in spectroscopic studies of the sun. The solar spectrum
was found to contain a series of lines, with positions described by
the equation
(13.88)
E n e rg y i s r e l e a s e d in th e f u s i o n o f d e u t e r i u m a n d tr itiu m in t o h e l i u m , j u s t a s in t h e
f issio n of U or P u ^ u n d e r n e u t r o n e x c i t a t i o n . F u s i o n , h o w e v e r , i s p o s s i b l e only
if Ihe p o t e n t i a l b a r r i e r o f th e C o u lo m b r e p u l s i o n b e t w e e n t h e D a n d T n u c l e i i s o v e r c o m e .
H ig h .'ftipera tu res 1 0 8 d e g r e e s ) a r e t h e r e f o r e r e q u i r e d i f t h i s r e a c t i o n i s to b e s e l f - s u s
t a i n i n g , w h e r e a s in t h e c a s e o f f i s s i o n e v e n l o w - e n e r g y n e u t r o n s c a n e a s i l y p e n e t r a t e t h e
n u c l e u s , an d t h a t e v e n a t low t e m p e r a t u r e s . T o o b t a i n a t h e r m o n u c l e a r r e a c t i o n , a m i x t u re
o f d e u t e r i u m a n d tritiu m m u s t f i r s t b e h e a t e d to a t e m p e r a t u r e o f t e n s o f m i l l i o n s o f d e g r e e s .
S u ch t e m p e r a t u r e s may b e c r e a t e d in a n a t o m i c e x p l o s i o n .
THE T HEORY OF THE HYDROGEN-LIKE ATOM 227
(13.89)
This series is, in effect, the Balmer hydrogen series (n, = 3, 4, 5,...)
with a number of intermediate lines, which form the so-called
Pickering series, characterized by the half-integral quantum
numbers n ,= 5/2, 7/2, 9/2........... At first, the Pickering series
was explained by assuming that hydrogen was in a special state
in the Sun, so that the quantum number n could assume half
integral values. The spectral lines, however, were later found to
be located further to the right than is indicated by Eq. (13.89),
and consequently this assumption had to be abandoned. The second
hypothesis assumed that the observed spectrum arises from singly
ionized helium (,Hei)+ , whose nuclear mass is M = 7360m0 and
whose charge is Z = 2. According to (13.83), its radiation fre
quencies then are
(13.90)
(13.91)
where n = 5, 6, 7, 8, . . . .
To answer the question of whether the Pickering series was
due to the radiation of hydrogen atoms (under the assumption that
the quantum numbers may assume half-integral values) or to the
radiation of ionized helium atoms (with the usual integral values of
the quantum numbers), it was necessary to find an experimental
value of the Rydberg constant. In the case of hydrogen.
(13.92)
(13.93)
Problem 13.1. Starting with Eqs. (13.26) and (13.28), show that the radial wave func
tions R ni for the principal quantum numbers /: = 1, 2 and 3 are as follows:
Ri<t = 2Ne~f/2,
Ne~**{ 2 - p ) ,
'2/2
Rn ■ N e ~f/2 p,
'2/6
N e ~ ?/2 (6 — 6p -f- p3),
9 / 3
Ne~ p/2p ( 4 - p ) ,
9 /6
Ru = -—
9 /3 0
where
Zy/i . 2 Zr
N= — and p = ----- .
\j0/ nao
On the basis of these specific examples, show that the functions Rni are orthonormal,
that is.
Hint. In proving orthonormality, make use of the fact that at different n and n', the
corresponding values for p also differ from one another.
Problem 13.2. Show that the average electrostatic potential produced by a hydrogen
like atom in the ground state is
2Zr
’ 00
r ^ \a 0 ^ r } \ m0e-J
Hint. Find the average electrostatic potential produced by the electron <I>j (r) =
— f o ^ ’I'* (r1) i" ( f ) d*x' and add it to the potential of the nucleus. Integrating
over the solid angle W, use the Identity
r'<r,
= 4it
§ r' > r.
Problem 13.3. Show that at Z = 1, the maximum of the probability density distri
bution D = r !R J’ in the states Is, 2p and Ad occurs at distances of tia, 4fl,i and <)u„ from
the nucleus (circular orbits). Why do the 2s, As and Ap stales have several maxima
(see Fig. 13.6)7
THE THEORY OF THE H Y D R O G E N - L I KE A T O M 229
r (I;,
U(r) ?P D(r) 3p D(r) 3d
f=9aa r
Problem 13.4. Using the functions given in Problem 13.1, verify the following equa
tions for n = 1, 2 and 3:
and
Find the spread (3r)- = r2 — r- of the radial deviations for these states. ___
On the basis of Fig. 13.6 and the uncertainty principle, explain why (&r)adoes not
vanish for circular orbits in the quantum theory.
Problem 13.5. By means of the quasi-classical WKB method, find the discrete energy
spectrum of the hydrogen-like atom.
Solution. According to (5.75) and (13.62), the eigenvalues of the discrete spectrum
(E <t 0) may be found from the equation
ri
where r i andr2 are roots of the integrand and
2 w0E „ nioZel
T '
Since the value of this integral is
I
we obtain exactly the same expression for the energy of the hydrogen-like atom as via
the Schrodinger theory.
8T h i s i n t e g r a l c a n b e e a s i l y e v a l u a t e d from ( 1 3 .6 4 ) b y s e t t i n g a = —A, b = 2B a n d
c = —(/ + 1 / 2 ) 2, a n d r e g a r d i n g Tj a n d r 2 > r j a s r o o t s o f t h e e q u a t i o n f (r) = 0. T h e l o g a r ith m
of t h e co m p le x q u a n t i t y [ a s s u m i n g t h a t a t e n d s to z e r o from t h e d i r e c t i o n o f p o s i t i v e num
b e r s ( a = + 0 ) | m ay b e t a k e n as :
, . ( i'O > 0 ,
In (fi + l a ) = In /3 + ] .
/ nr 0 < 0 ,
, | I i2 n fi > 0 ,
In — Ia) = + | p<0
230 N O N R ELA TIV ISTIC QUANTUM M E C H A N IC S
Problem 13.6. Determine that magnetic field intensity at the center of the hydrogen
atom which is due to the orbital motion of an electron. Find its numerical value for the
2p state. __ __
Answer. t l x = Hy = 0,
m0el\3 1______
H, = — m
ni0c
Hz ~ 101 gauss.
Hint. Take the classical expression H = — r x /> for the intensity of the
where the perturbation energy V' V°, and the main part of the
potential energy F° is chosen in such a manner that the Schrodinger
equation of the system
( £ — H) = 0 (14.2)
(£ — H° — V') = 0. (14.2a)
t£° - 11") ■/’ 4- [(£' - V') y + (£» - 11")'/) + ( £ '- V') 'J/=0. (14.2c)
TIME-INDEPENDENT PERTURBATION T HEORY 233
*p-’!»
o po pn po
*‘“•tli •••j
and eigenfunctions
r?i° t»i° ,»in
Vl> ?■>> ?3i •••■
i!.°
?«> •••>
Keeping all this in mind, we shall now investigate the equation for
the first-order approximation of perturbation theory
ft = 2 f t f t . (14.8)
n‘
Our problem, therefore, reduces to determination of the unknown
coefficients C„■of a generalized Fourier series. Substituting (14.8)
into (14.7a), we have
C. NONDEGENERATE CASE
J W -' <?X=lnn..
Since the left-hand side of Eq. (14.10) is equal to zero (E°n— ££. = 0
for n’ = n, and 8n«' = 0 for n' n), the energy correction E'n is
where
V'nn=^<ln*V'<fnd*X. (14.11a)
Thus, the energy correction E'n of the system quite naturally turns
out to be equal to the average value of the perturbation energy V'.
It is worth noting that the expression (14.11) for the energy
correction was obtained by setting the left-hand side of Eq.
(14.7a) equal to zero after it had been multiplied by <JC and in
tegrated over all space. Since is a solution of the homogeneous
equation (14.6), it follows that the right-hand side of the inhomo
geneous equation
Wj = f (14.12)
{ <p*fdix = 0 . (14.13)
T IM E -IN D E P E N D E N T PER TU R BA T IO N THEORY 235
2 C„ ” (EJ - Ei» ) = ■ - (E ; - V)
^ = ’ (14.14)
where
^ = £ ^ '< 1 ^ . (14.15)
= ^5 + 2 ’ (14. 16)
n’
where the prime on the summation symbol indicates that the sum
is taken over all n' except n' = n. Finally, the as yet undetermined
coefficient Cn of the zero-order wave function can be found from
the normalization condition
J < ^ nc f * = l (14.17)
where
c ;= l+ c „ . (14.19)
ICJI* J W<t'n<Px +
+ 2 { c r c n-
nr
J r*¥ n ' CPx + Cn'Cn J « dh) = 1. (14.20)
236 N O N R EL A TIV ISTIC QUANTUM M E CH A N IC S
C 5 = l. (14.21)
and therefore C„ = 0-
Consequently, for the wave function in the first-order
perturbation theory, we finally obtain
vv I/’*
% = 'r’n 4* 2 E° — Et. ■«' (14.22)
From Eqs. (14.22) and (14.11), it can be seen that both ^ and
E'n are proportional to the first power of the perturbation energy
(that iS, to the parameter X). If we were to compute the corrections
to the energy and the wave function in second-order perturbation
theory, both £" and ''/n would turn out to be proportional to the
second power of V' (that is, to X2).
D. DEGENERATE CASE
j
(14.23)
/=*i
(£,; — n °)^ = o,
Using the theorem for the transfer of a derivative [see Eq. (7.17)],
we have
J '^n'lYni,d3X='bn.nl,'
we obtain instead of (14.26)
where
and the prime on the summation sign indicates that the sum extends
over all i” s, except /' = /. Since the subscript i in (14.27) can take
any value from 1 to j, we have a system of j homogeneous equations
from which we can determine the energy E'„ and the coefficients C;:
. (14.17a)
(E'n- V n) , - V u...... - V [ ,
- V n, (En-V'n)....... -V',1
= 0. (14.31)
&n u c l. fly
• 1 0 s volts/cm ,
2T h e e l e c t r i c f i e l d i n t e n s i t y w i l l h a v e n o e f f e c t on t h e o s c i l l a t i o n s a l o n g t h e x a n d y
a x e s , w h i c h a r e p e r p e n d i c u l a r to E .
3T h e f i r s t q u an tu m l e v e l (n = 1 ) i s n o n d e g e n e r a t e an d , t h e r e f o r e , i s n o t s p l i t .
240 NON R E L A T I V I S T I C QUANTUM MECHANICS
Vi = fi(r)z, (14.36a)
(14.37a)
H
(14.38a)
Jl
1
where
(14.40)
i
where
V h = J rm }d*x =
vanish, since for each of them the integrand must be an odd func
tion with respect to at least one of the coordinates x, y and z. Only
the matrix elements
which are even functions of all three coordinates, differ from zero
Let us substitute the values of Eq. (14.39) for /, (r) and (r) and
note that, according to Eq. (13.28a),
R»= — (2 -
2 1^2 a 3/* \ °o/
and
242 NONRELATIVISTIC QUANTUM MECHANICS
^ = = °>dr.(14.43a)
e -y dP= r(s+l),
0
we obtain
= — 3^60,. (14.44)
E:| 3a en8 0 0
3a e0 8 E\ 0 0
(14.45)
0 0 0
0 0 0
£ '= (£ /- 9 a M 8 4) = 0 . (14.45a)
E-t 1 —
£ ; i2'= 3a0 e0 8, (14.46)
y (p x = i,
In a sim ilar fashion, it can be shown that the state with energy
£ •2
f (3, = <Ki,. (m = 1 ),
or
(m = — 1 )
----------------------- T +3e*°oS
-Ri Rh
T j
------------------------------% - 3 e 0 a0 6
a b
F i g . 14.1. S p l i t t i n g o f t h e s e c o n d s p e c t r a l ter m o f h y d r o g e n in
a n e l e c t r i c f i e l d ( t h e l i n e a r S ta r k e f f e c t ) :
a) e n e r g y l e v e l in t h e a b s e n c e o f a f i e l d (S = 0 );
b) e n e r g y l e v e l in t h e p r e s e n c e o f a f i e l d ( & ^ 0 ).
(Ex = Ey = 0, £ z = &),
n= V e”,
where s is the dielectric permittivity (the magnetic permeability
(i is taken to be equal to unity). If the index of refraction n becomes
larger as the frequency of light increases (that is, — ^>0 ),th e
dispersion is said to be normal. A typical example of normal
dispersion is the spectral resolution of white light by a glass or
quartz prism (the deflection of violet rays from their initial direc-
dti
tion is larger than that of red rays). If, however, ^ < 0 i n a certain
range of frequencies, the dispersion in this region is said to be
anomalous. As a rule, anomalous dispersion occurs at frequencies
at which light is absorbed by the medium.
To determine the index of refraction (one of the most important
problems in the theory of dispersion), we use the equation relating
the electric field intensity E, the displacement vector D, and the
polarization vector P 4
D = e E = E ^ 4 -P . (14.53)
Since t = we have
P =n
^ E . (14.53a)
T h e p o l a r i z a t i o n P i s d e f i n e d a s t h e t o t a l d i p o l e m o m en t of t h e a t o m s p e r u n i t v o lu m e.
246 NON R E L A T I V I S T I C QUANTUM M E CH A N IC S
We shall assume that the frequency of the incident light wave is o>.
Then
“S—«■ (14.57)
TIME-INDEPENDENT PERTURBATION T HEORY 247
“ o. “ I. “>9. • • • . “ a >•••>
na— 1 jL y n* (14.57a)
An rrto Zj “ 1 —<->3 *
n1 — 1 _ _ Ne\j
4t. ~ m0 w§ (14.58)
On the other hand, for frequencies u)J>u>0, the value of the index of
refraction is given by the relation
—1 Ne\
4t. ~ m0“* ' (14.59)
Fx = — e0gocos w t > Fy = F z = 0.
( - ^ - H° - r ) < M 0 = °- (i4.6i)
Let us suppose that Eq. (14.61) has an exact solution for V"= 0:
(0 = ^ e - m)Ek‘ = , (14.62)
( £ * - H ') ^ = 0. (14.63)
= (O + MO- (14.64)
( - 7 £ - 1 1 ° ) *2 (0 = 0, (14.63a)
the equation for < } > * (0 and the first-order correction to the energy
£* is
(14.65)
To eliminate the time t from this equation, let us look for a solu
tion ty't, (() in the form
(t) — ue~ u (“*-") ve~11(“+<u/(). (14.66)
We note that these two equations have exactly the same form.
Consequently, it is only necessary to solve Eq. (14.67) for u, since
the solution for v of Eq. (14,68) can be obtained from u by substi
tuting — io for (o. Since the time does not appear explicitly in Eq.
(14.67), we can find u by the perturbation method in the form ap
plicable to stationary problems. Thus we shall look for a solution
in the form of an expansion in eigenfunctions of the unperturbed
problem [see (14.8)]:
(14.69)
k'
(£ » » -H ')fr = 0 . (14.70)
(14.71)
250 NON R E L A T I V I S T I C QUANTUM M E CH A N IC S
2 ft ) ’ (14.73)
h'
& = N p = — Nenx.
J j d'x—0,
which follows from the fact that the integrand is an odd function of
x. Comparing Eqs. (14.78) and (14.53a), we obtain the dispersion
formula
2 Wlo
fk’k n ^k'k | Xk'k[ , (14,80)
n2 — 1 N e\ V I f k'k
47u m0 £ u2k,k—u: (14.81)
k'
Here let us make an observation sim ilar to the one made in regard
to the classical treatment: namely, if we had included the radiation
damping in the quantum-mechanical treatment, we should have
obtained a finite value of a* for frequencies u> in the neighborhood
of u>k'k (see Fig. 14.3a, the dotted line).
F i g . 14.3. D i s p e r s i o n c u r v e s .
a) p o s i t i v e d i s p e r s i o n (o)^=
b) n e g a t i v e d i s p e r s i o n
hk
xk1 1 . k V h<k+ ) and xk \.k
1
2/7 lo°7|)
We can see that in this particular problem the quantum and clas
sical theories yield the same value for the index of refraction n.
The phenomenon of negative dispersion is not observed. The
reason for this is that the regions of positive and negative disper
sion coincide since I | 4 = I“*
■i, * I1, so that the stronger effect of
positive dispersion masks the negative dispersion.
G. RAMAN EFFECT
e= Hm (14.84)
7
More p r e c i s e l y , t h e law of c o n s e r v a t i o n of e n e r g y may b e v i o l a t e d in i n t e r m e d i a t e
s t a t e s . It i s r e q u i r e d to h o ld on ly in t h e f i n a l r e s u l t .
254 N O N R ELA TIV ISTIC QUANTUM M E C H A N IC S
u/ = u) — to*'* to
these lines appear only when the light is scattered by excited atoms.
It is obvious that at low temperatures only Stokes lines can be
observed. As the temperature increases and the atoms of the
substance begin to undergo transitions to excited states, anti-
Stokes lines appear.
The Raman effect provides much important information in
studies of molecular structure. In Chapter 12, Section C, we saw
TIME-INDEPENDENT PERTURBATION THEORY 255
that the rotational and vibrational levels (and also the vibrational-
rotational levels), which provide data on molecular structure, are
all located in the far infrared region of
the spectrum and are very difficult to
observe. In studies of the Raman effect,
it is possible to use visible light in gI I ------------ 1
CO
determining molecular spectra, since a/ of
these spectra are superposed on the bI I I I
lines in the spectrum of the incident
light. The experimental values of <o' and F ig . 14.6. S u p e r p o s i t i o n o f th e
a>" (see Fig. 14.6) yield the molecular m o l e c u l a r f r e q u e n c i e s on t h e
frequencies f r e q u e n c y of i n c i d e n t l ig h t:
a ) s p e c t r a l l i n e a> in t h e a b
s e n c e of m o lecu lar o s c illa tio n s ;
aik'k = u>—«/ and Whh"= id"— u),
a n d b) s h i f t o f t h e s p e c t r a l l i n e
d u e to m o l e c u l a r o s c i l l a t i o n s
from which the selection rules can be ( < u '= co— b>k ' k a n d a i " = “ A * ")-
derived.
Problem 14.1. Find the energy correction for a system in second-order perturbation
theory.
Solution. Including the terms up to and including the second order in the expansions of
the wave function 4 (14.3) and the energy E (14.4), and substituting these expansions into
the Schrodinger equation (14.2a), we obtain the equation
(£ “ - H °) V n = - ( £ ’„ - V )V n - K C
Since the solution 4^* of the homogeneous equation (£° — H°) 4„ = 0 is orthogonal to the
right-hand side and since we can substitute the expression (14.22) for 4 n ,we have
n'
The value of V'n,n is given by (14.15), and we have used the relationship
V
v /m , = V'h
n n*,
which holds for Hermitian operators.
We note that the second-order correction (14.85) to the energy of the ground state is
always negative, since all the other levels Eh' are higher than the level Eh, that is.
Eh' > Eh.
Problem 14.2. Using the results of the perturbation theory, find the energy of the
anharmonic oscillator including the terms up to the order of take the Hamiltonian of
the system to be equal to
maa-x-
H= t2m
;—0 + ■y\
where V’ = ax3 -(- fU4 (the constants a and ? are classical quantities).
Solution. The energy of a harmonic oscillator (V' = 0) is
£ ; = i / ; n = a (A- \ n + p(A-1u
256 NO N R ELA TIV IST1C QUANTUM M E CH A N IC S
E’n= JlL V
tla ^ (n —il')
The only nonvanishing matrix elements are [see Eq. (10.55) and Problem 10.5]
/ .- a \ __ / a *2 \ /*a v 3 ~ \ f ~ n (n 0 ( 11 2)
v ' /n. n -a — ('*• /n . n- 2 \ x )n -* > / i - a — * '■ 0 1/ g »
h
where x 0=
Hence
/ III uwo
15 n- a-
«'iV «’ + « +
En (14.87)
4
P a r t II
(15.2)
— cV — mjc* -0 , (15.3)
260 R E L A T IV IS T IC QUANTUM M E C H A N IC S
E = *7Z~, p = - i h V , (15.4)
which act on the wave function | (r, (). Replacing tn0 by hk^fc and
dividing by tl2c-, we obtain 1
V2 ---- ______ £2 ) a —
c2 d t2 *n l v — & : A - ko ) ^ = 0- (15.5)
Here
xt - - x, x.3= y, x3= 2 , Xi = id (15.6)
s/- f + 4 r = ° - (15*7)
d2 d2 (15.8)
(r * - <!< "r r ) = 0 .
P
. 'fi.L * d't (15.10)
2m0c2 L‘ M
and
ell
(15.11)
J 2im0
(15.12)
where
A'i = id, j 4 = icp. (15.13)
d 5 .i4 )
k a !-=5 ? 1 4 4 t] <15-15>
gives rise to some difficulties in the relativistic theory. The
Klein-Gordon wave equation is a second-order differential equation
and, therefore, both ^ and ^ can be arbitrarily defined at some
dt
given time t. Consequently, the density p0 (15.15) is not positively
defined, unlike the nonrelativistic probability density
^ + — { ( E - F ) ’ - m ^ } t = 0. (15.20)
Since the potential energy does not depend on time in this equation,
we can transform to the time-independent case by means of the
substitution
2T h i s e q u a t i o n c a n a l s o b e o b t a i n e d from t h e r e l a t i v i s t i c H a m i l t o n i a n for a p a r t i c l e in
an e l e c t r o m a g n e t i c f i e l d
It is onl y n e c e s s a r y to t r a n s f e r to t h e l e f t - h a n d s i d e , s q u a r e b o th s i d e s , a n d r e p l a c e
p an d A by t h e i r q u a n t u m - m e c h a n i c a l o p e r a t o r s .
T H E KL E I N —GORDON S C A L A R R E L A T I V I S T I C W A V E E Q U A T I O N 263
A
(15.24)
± t M i > _ n i + n - z v R t= 0 (1 5 25)
Ro = Crs.
We then obtain an equation for s
sliS= — L ± y p + D ’- z v . (15.27)
Consequently,
R. -}■- C./5*. (15.28)
2 64 R E L A T I V IS T IC QUANTUM M E CH A N IC S
If
both roots s, and s.2 will be real for all values of I (I = 0 , 1 , 2 , . . .).
We retain only the solution for rR0 that does not diverge at r = 0;
that is, we set Ca= 0 . Similarly, only the exponentially decreasing
solution for the wave function as r —*oo should be kept when
£ < / 0 (/I > 0 ) . The asymptotically decreasing solutions for the two
limiting values of r yield the same equation for the energy spectrum
as in the Schrodinger theory, as can be seen from Eq. (13.20) by
substituting s, for I. Thus, for the eigenvalues we have the equation
Z 2al 2
Fnl = m ^ 2 —m^c1.
(15.30)
RhZ2 a2Z 2
En l i+ (15.31)
Aw 'h i - J L ™ _ i £ 7! (15.32)
7l 3 1C ■
Za>l (15.33)
Problem 15.1. Find an expression for p and j if the scalar relativistic equation con
tains a term arising from the presence of an electromagnetic field.
g
Solution. Let us substitute p —■p — — A, E — E — e<b in the Klein-Gordon equation,
Repeating the calculations that lead to Eq. (15.15), we obtain the generalization
where E = ± chK Is the energy of the particle. Since the charge density Is given by the
expression
It follows that p is the charge density rather than the particle density, since this re
lation can be Interpreted only if it is assumed that particles described by an amplitude
B (negative energies) have a charge of opposite sign from particles described by an
amplitude A (E >■ 0).
Chapler 16
F= —kr. (16.1)
k = m0t»l. (16.2)
(16.4)
h (,,i”z = 0 .
MOTI ON OF AN ELECTRON IN A M A G N E T I C FIELD 269
^ v e n in t h e c a s e o f very s t r o n g f i e l d s ~ 106 g a u s s ) t h e q u a n t i t y 0 i s o f t h e
o r d e r o f l O 1^ s e c w h e r e a s t h e f r e q u e n c y o f o s c i l l a t i o n of a n e l e c t r o n in an atom (the
o p tical spectrum ) is ^ 10*^ s e c T h e r e f o r e , t h e i n e q u a l i t y 0 « <±>q i s p r a c t i c a l l y
alw ay s satisfied .
270 RELATIVISTIC QUANTUM MECHANICS
1 1 j <3TdS.
c dt
Assuming thats/T and S depend only on time and that the switching
on of the magnetic field does not alter the radius of the stationary
orbit, we find
r
8 = — 2c dt ‘
The additional velocity imparted to the electron ( e = —e0) by this
electric field can be found from the equation
which gives
ymag _reo_ ^
2m„c
y mag — ^m ag^i___ ^
L = ml) r x v ,
(16.12)
(E — Hs ) | = 0 (16,13)
H S = e©(r) + — (16.14)
Ax = - ^ y ^ T , Ay = \ x J r .
Remembering that
where
d
L, — ih d't -
Lt Y? = hmY?.
we obtain the equation
which also includes the effect of the magnetic field on the atom.
This equation may be written as
(£ — Hs ){| = 0,
Hs + <16-19>
e = — e„.
The last term in the Hamiltonian may be attributed to the presence
of the orbital magnetic moment of the atom, which gives rise to an
additional energy
Vm&s = — \i-H = — p ^ = -£j^in-££ (16.20)
we obtain the same relation between the magnetic moment and the
angular momentum as in the classical theory [see (16.12)]:
Fz __ go (16.22)
Lz 2 m ac
liz = — Hm>
^ = J ^ = 9-273- 10’4' erg.gauss-', (16.23)
It was shown in the last section that the Schrodinger theory is able
to explain only the orbital angular momentum and magnetic moment
of an electron. The basic equations characterizing these properties
are Eq. (16.22) for the ratio of the orbital magnetic moment and
L e t u s n o t e t h a t t h e r e a s o n s f or t h e u s e o f t h e t e r m s “ n o r m a l Z e e m a n e f f e c t " a n d
"an nSTlous Z e e m a n e f f e c t " a r e p u r e l y h i s t o r i c a l . U e f o r e t h e d i s c o v e r y o f e l e c t r o n s p i n ,
o n l y t h e c l a s s i c a l t h e o r y o f t r i p l e t s p l i t t i n g ( n o r m a l Z e e m a n e f f e c t ) w a s k n o w n . Whe n a
m o r e c o m p l i c a t e d s p l i t t i n g w a s d i s c o v e r e d it w a s c a l l e d t h e a n o m a l o u s Z e e m a n e f f e c t
b e c a u s e n o t h e o r e t i c a l e x p l a n a t i o n c o u l d b e g i v e n f o r it u n t i l t h e d e v e l o p m e n t o f t h e t h e o r y
of e l e c t r o n s p i n
MOTION OF AN ELECTRON IN A MAGNETIC FIELD 275
the orbital angular momentum and Eq. (16.24), which indicates that
the number of possible orientations of the magnetic moment relative
to the z axis is necessarily odd, since the
number of states with different quantum
numbers m is equal to 2/ + 1. The Schrodinger
theory, however, does not adequately account
for all the experimental data, the analysis of
which led to the discovery of the spin prop
erties of electron. Let us briefly discuss
these experimental results.
1. First of all let us consider the Einstein-
de Haas experiment (1915), which was carried
out in order to verify Eq. (16.22):
^L z = —gs —
2 m ac '
F ig . 1 6 . 2 . D i a g r a m o f t h e S t e r n —G e r l a c h e x p e r i m e n t f o r t h e
d eterm in atio n of th e m a g n e tic m om ent o f m o n o v a le n t atom s.
276 R E L A T IV IST IC QUANTUM M E C H A N IC S
(*■*= — IVrc,
c mag
i V qS L drJT
— e mag ^ co s a qz — P* dz (16.27)
(16.28)
2 2 v1 M dz
We n o t e t h a t in o r d e r to d e t e r m i n e th e m o tio n of th e c e n te r of m a s s o f th e m a g n e tic
d i p o l e , it i s q u i t e i m m a t e r i a l w h e t h e r w e r e g a r d i t a s a r i g i d d i p o l e o r a c u r r e n t l o o p .
MOTION OF AN ELECTRON IN A MAGNETIC FIELD 277
8z = 0 (m — 0), = + (m = zh i ) .
e„h (16.29)
f1"- 2m„c
5, = ± i * ; (16.30)
D. PAULI EQUATION
Pp = — \ i - H. (16.32)
+ <16-34)
Next, it was necessary to find suitable quantities to describe
the intrinsic magnetic moment of the electron. It is well known
that introduction of the spin is related to introduction of a
fourth quantum number, characterizing the internal properties of
an electron. On the other hand, the wave function ^ of a particle
depends only on three quantum numbers, corresponding to quantiza
tion of the three spatial coordinates. In order to describe spin,
Pauli introduced two wave functions T, and 'F.2in place of the single
w a.c function ^. One of the wave functions describes a state with
one spin orientation and the other wave function describes a state
with the opposite spin orientation. The actual wave equation
represents a system of two equations. It is possible to represent
a system of two or more equations, such as
MOTION OF AN ELECTRON IN A MAGNETIC FIELD 279
(16.35)
(16.35a)
where p.0 is the Bohr magneton and a' stands for the three 2 x 2
Pauli matrices
0 1' 0 —i ./I 0
10 1 0 ■* \0 - 1 , (16.38)
These matrices are denoted by the letter a with a prime (the same
letter without a prime will be used to denote the 4 x 4 Dirac
m atrices). These matrices characterize the components of the
spin vector along the coordinate axes.
Using the rule (16.36) for matrix multiplication, it can be
readily shown that the square of each Pauli matrix is equal to
unity
ajs = Q-i = °3S— I > (16.39)
{ £ + e0 <l»— 1^ m— ^ 1 - -0 ,
| £ -j- eo® — -f- (i.03/^ — | 1 .2= 0,
(16.44)
(£ + M> + wX" — 2"-) Vi = 0 ;
that is, the wave function T, describes a state in which the intrinsic
magnetic moment of the electron is parallel to the z axis, and the
wave function T,, a state in which the magnetic moment is anti-
parallel to the z axis. These are the orientations of the intrinsic
magnetic moment which were observed in the Stern-Gerlach
experiment.
As the function 'I'1’ Pauli suggested taking the Hermitian adjoint
of '[', that is, the matrix >I;v = (M;*'I'*), whose elements are ob
tained by taking the complex conjugates of the elements of lIr
and transposing them (interchanging rows and columns). Thus, if
MOTION OF AN ELECTRON IN A MAGNETIC FIELD 20
V1>F= (V
F*’F 1j = ' F * ' F , _|_ ' F j 'F,, (16.45)
_ ipnp _ipnp ■
(16.46)
that is, 'F f ' F , and lF * * F s represent the probability densities of states
in which the electron has a spin orientation parallel and antiparallel
to the z axis, respectively. Using the expression for the intrinsic
magnetic moment in the Pauli theory
lir = — -P-°- S,
m 0c
we find that
S -- -.y Tift'. (16.47)
S ^ , - S vS , = i7 iS ,,
svs, - s,sv= ihsx, (16.48)
SZSX— SVS_. = itiSy.
282 R E L A T IV IS T IC QUANTUM M EC.HAN1CS
where o is the Larmor frequency. It follows that the component of the angular momentum
in the field direction (z axis) is a constant of the motion. The components along the x
and y axes, however, precess around the z axis with the frequency o .
Problem 16.2. Show that the spin operator S is vectorial; that is, if we construct the
linear combination
Problem 16.3, Show that in a homogeneous magnetic field which is a function of time
only, the wave function of the Pauli equation can be resolved into a product of coordinates
and spin functions. What form does this solution take if the field is time Independent?
Solution. Let us look for a solution of the Pauli equation in the form
It is readily shown that the coordinate part of the wave function 41(r, t) satisfies the
ordinary Schrodinger equation without the spin
= t).
while the spin part of the wave function may be obtained from the equation
(CfC,*i(§)=CfC, + « C , = l.
Then the time-independent parts of the wave function are determined from
(E — Es) ^ = H S
Problem 16.4. Find the eigenvalues of the operator of a spin component along the
direction specified by the spherical angles 8 and <p. Investigate the particular cases in
which this direction is the x , y or z axis.
Solution. Consider first the case in which the spin is directed along the z axis. Then
the initial equation takes the form
where
n /1
'l 2 \0
{ c , - i c , = o,
1 c , + > .c , = 0.
where
0 n 0 s
S.* 10 9 o,r
284 R E L A T IV IST IC Q U ANT UM M E CH A N IC S
(b) the solution corresponding to the case in which the spin is antiparallel to this
direction
Setting 8 = 0, y = Owe obtain the same solution as above. The cases in which the spin is
directed along the x or y axes may be obtained by setting, respectively.
n
”*
Problem 16.5. The electron spin is parallel to the z axis. Find the probability that
the component of the spin (a) in a direction parallel to the a- axis, and (b) in a direction
making an angle 0 with the z axis, will have the values-g- ft and---- ry ft.
Hint. Take the wave function describing the state in which the spin is parallel to the
z axis and then expand it in terms of the functions corresponding to the cases in which
the spin is parallel and antiparallel to the direction forming an angle 8 with the z axis.
Both these functions are given in Problem 16.4. Without loss of generality we may set
the angle <p= 0.
Then the squared modulus of the expansion coefficients gives the probabilities
of the components of the spin along the corresponding directions; these are equal to
In order to find the x component of the spin, we must set 8 = — in the last equations,
Clinplrr 17
E= p= — ih v, (17.2)
E= c + mlc1= c J W (17.3)
p-.=o
where
p0= m0c, Pi = px, Pi = Py, Pi = Pz■ (17.4)
We note that
3
— 20^^-, (17.7)
that is, all four quantities <*., anticommute with one another
<=1. (17.9)
We recall that the 2 x 2 Pauli matrices also possess analogous
properties
0 1 o —i '
a
10 c,
o
a (17.10)
since they anticommute [see (16.40] and the square of each is equal
to unity [see (16.39)]. To extract the square root of the four-term
polynomial, however, it is necessary to have four relations (17.7)
(p = 0, 1, 2, 3), instead of three [Eqs. (16.39) and (16.40)] that are
satisfied by the Pauli matrices.
Accordingly, Dirac proposed that we take a system of 4 x 4
matrices cn and P„ that are related to the 2 x 2 matrices by the
expressions
0n Cn 0' ( « = 1 . 2, 3),
o' < (17.11)
(17.12)
0 0 and l'= 1
00 ?)■
Hence we find, for example,
/0 1 0 O'
/ 1 0 0 0 and so on.
3i I0 0 0 1
\0 0 1 0 .
o“= P; = I (17.13)
or, more exactly, are equal to the 4 x 4 unit matrix
/I 0 0 O'
0 1 0 0
(17.14)
I0 0 1 0
\ 0 0 0 1 /
= 1, 2, 3), (17.17)
T 0'\
“o—Pa= O' —I')- (17.18)
i /°0 0 0 i\ /° 0 0 ■
a ,=
0 1 o\ a - 0 i o\
0 1 0 or [ 00 —i 0 O’
Vi 0 0 0J \i 0 0 0/
(17.19)
/I 0 0 0
//°0 0
0 1
0 - °\
-M, (0 1 0 0
ao= Pa='100--1
—\l 0 0 °' 0
Vo --i 0 0/ \o 0 0 -- 1.
E= P ~ ~ iflV’
^ B e c a u s e of Ihe fo u r c o m p o n e n t s o f t h e w a v e f u n c t i o n 0 , e a c h s t a t e c a n h a v e e i t h e r
p o s i t i v e or n e g a t i v e e n e r g i e s ( s e e b e l o w ) a n d tw o d i r e c t i o n s o f t h e s p i n ( s e e C h a p t e r 18).
In th e c l a s s i c a l c a s e t h e r e l a t i o n (1 7 .1 ) b e t w e e n t h e e n e r g y a n d m om entum c a n b e
r e p r e s e n t e d in a form s i m i l a r to (1 7 .2 0 )
m0r2 m0u
._ and p -— —— .
y'l- £ 2 \/l-/32
C o n s e q u e n t l y th e ma trix at • v c m u st p l a y t h e ro le o f t h e v e l o c i t y , w h i l e p 3 * \J 1 —/32 i s
a s c a la r that c h a r a c te r iz e s the L orcntz contraction.
THE DIRAC WAVE EQUATION 209
'<Pr
(17.22)
^3
•'W.
The complex conjugate of this function is understood to be the
Hermitian adjoint, that is, the row matrix
^ = (17.23)
- ^ i H V = ihvy-, = — (17.27)
Multiplying (17.28) on the left by <{>+ and (17.29) on the right by <(>
and adding them, we obtain
4 -p + v -y = o , (17.31)
where5
p= e<]>+<jj, j=ec<Sf+a.']/. (17.32)
'h\
p0= - = ^ = ( M W ) = W t + W * + W . + K 'hi (17.33)
0 0 0 1
= ^ = o 1 0 0 = r ■=
\1 0 0 0 i
= « < ! • « + w. - (17.33a)
We note that, contrary to the Klein-Gordon theory, the density
p„ is a positively defined quantity. This does not mean, however,
that in the Dirac theory p„ can be considered the particle
density. Just as in the Klein-Gordon theory, there will be particles
with a sign opposite to that of the electrons (positrons). From
Eq. (17.32) it can again be concluded that ca should be regarded
as the velocity operator.
^ i ^ i l a r r e l a t i o n s w i l l a l s o h o l d in t h e c a s e w h e n a f i e l d i s p r e s e n t .
T VT=?’ c
Here we have used the fact that e-1"1 = cosh y*, - sinh ya, = cosh
f — a, sinh y, since a?" = 1, *?"+'= where n is an integer. In
order to satisfy the above relations, we must set
^c o s h — a, s i n h l) y = e 2
(17.37)
O'1"= F ^ cosh 1— a, s i n h =i}»+e 2
Then, since
(17.38)
(17.39)
The above relations follow from the transformations for the cur
rent vector:
U = ix cos <P + /, Sin <P,
j 'y ~ = jy COS j x s i n cp, (17.40)
In the Dirac theory, however, which takes into account the electron
spin, the component of the orbital angular momentum does not
commute with the Hamiltonian
H = c(a.p) + p3 m(1c‘!- f V{r) (18.3)
J = L + --/b = L + S (18.6)
2 94 R E L A T IV IS T IC QUANTUM M E C H A N IC S
We shall now show that the operators for the components of the
total angular momentum in the Dirac theory satisfy the same com
mutation relations as the operators for the components of the
orbital angular momentum in the nonrelativistic quantum theory
(see Chapter 11). It can be seen that the operators L and S com
mute with each other, because they act on different variables.
Therefore,
-- Iftj ^,
JyJz - J zJ, = i7LJ,, (18.9)
J J A— J A.J . = ihiy.
The last two relations are obtained from the first by cyclic per
mutation of the coordinates
which obeys the Pauli equation (see Chapter 16), can be related by
means of the law of conservation of angular momentum
(18.15)
where K,m are the spherical harmonics (see Chapter 11). Then,
since
L4 © = w + » K ) . (18.17)
_l_
n
or
where
(18.18a)
(q — m - 1)C,+ ] / ( / + 1 — m ) ( l - \ - m ) C i = 0,
V ( l + 1~ m ) (l + m) C, + (q + m) C, = 0.
7 = '. / = i + 7„ (1.8.22)
7= i = i ~ % c, = Y (18*23)
Wo n o t e Ih <i t t h i s i c l o t ion s h i p b e t w e e n th e s p h e r i c a l h a r m o n i c s i s o b t a i n e d o n l y in t h e
c a s e of s p i n - o r b i t i n t e r a c t i o n , w h i c h we h a v e t a k e n in t o a c c o u n t w i t h t h e a i d o f t h e r e
l a t i o n s (18 .1 5 ). If t h e r e i s no i n t e r a c t i o n , t h e two s o l u t i o n s w ill b e c o m p l e t e l y i n d e p e n d e n t .
T H E D I R A C T T H E O R Y OF T H E M O T I O N OF AN E L E C T R O N 297
>F( ; = i + 1/s) =
Y/ + 1—m
l + '«
'll f I
y m —I
/
y m
v u ^ t + ' h)
*I m (18.24)
- V 2/+ I
For /' = / —‘/s. / = 1, 2, . . . (the second type of solution), the wave
function has the form
f /- m l y m - \
qn;=/-'/2) V 2/ + 1
y m
\ - YIu m= ‘ i / 2)
(18.25)
l/" —
V '21 +
where /' = / + 7* corresponds to the case in which the spin and orbital
angular momenta are parallel, and / = / —7* to the case when they
are antiparallel. This condition follows immediately from the fact
that the spherical spinor Y\i]m is a single-row matrix and from the
orthonormality condition for spherical harmonics. The spherical
spinors (18.24) and (18.25) are spinor generalizations of the ordinary
spherical harmonics (see Chapter 11) and represent the angular part
of the solution for all problems involving the motion of a particle of
half-integral spin in a central field of force.
Substituting these solutions for the function T into (18.15), we
find that the component Jz of the total angular momentum takes the
value Jz — hrrij, where the quantum number is equal to rtij= m — -7.
For the first type of solution (/ = / + --). it can be seen from (18.24)
that it ranges from — / (m; = — I ——= — /) to I -f- 1( my= / -(- -7 = j ),
since the coefficient at the function Y?~\ which does exist tor the
last value of m, vanishes. In exactly the same way the number m
in the second type of solution (/ = / — -7) ranges from — / - f 1
(rrij = — j) to I (rrij = j).
Thus our results can be summarized as follows. The square
of the total angular momentum has the eigenvalues
298 R E L A T IV IS T IC QUANTUM M E C H A N IC S
From the relations (18.6) and (18.7) and the quantization rules
(18.26) and (18.27), it is easy to obtain quantization rules for the
scalar products L • S and J • S', which are important in spectros
copy
L- S = l ( P - L 2- S 2) = - { / ( / + l ) - / ( / + l ) - s ( s + l ) } , (18.28)
and by analogy
T h e num ber j is also called the internal quantum number. This number was introduced
by speclroscopisl s before the discovery of spin on a purely em pirical basis. Il e x p r e s s e d
certain internal properties of p a rticle s that were s ti ll u n c l e a r at th a l stage.
^T h e lack o f m a t h e m a t i c al rigor lie's, for example, in the fact thal the square of the
vector j is e q u a l t o j ( j i 11 r a t h e r th an to / 2.
From the vector model we can quickly find a number of quantities. For example, we
can find the quantization rule of the angle between the vectors I* and s*. From the
oblique triangle we obtain
that Is,
F _u-i(/ + I)
1 2ma(i-
(for simplicity let us take the radius of the rotator equal to unity:
a = l ) . If in place of the spherical spinors we substitute their
values from (18.24) and (18.25), the matrix element (18.35) be
comes
From this it is seen that the two integrals in (18.37) agree exactly
with the integrals in Eqs. (12.19)-(12.22). The selection rules for
the quantum numbers / and m will therefore be the same as for
a spinless rotator, that is,
and
______ I_____
C ( / = ' ' + 1/o . / = ! - ' / : ) = _ V r + l — m'Vl + m
/(2 /'+ I)(2/+ 1)
from (18.24) and (18.25), and the expressions for A and B from
(12.22a), we find that the coefficient of V , + 1 vanishes; that is,
the transition Aj = — 2 is forbidden. At the same time, the co
efficient of 8/'. ;_i does not vanish, that is, the transition Ay = 0
is allowed, but the intensity of the lines is weak in comparison
with A/ = ± 1. In a sim ilar manner it is easy to show that the
transition A; = 2 is forbidden not only for q = z, but also for
q = xAziy (Am = zbl).
In accordance with the above discussion, the selection rules
for the quantum numbers which characterize the state of a particle
in a central field of force, when the spin is taken into account,
have the form
A/ = ± l , Am; = 0, zb I, (18.39)
. . / — I (normal intensity),
^ 10 (weaker intensity).
D. PARITY OF A STATE
= /•); (18.41)
that is, it reverses the signs of the space coordinates. The eigen
values X of this operator may be found by applying this operator
twice:
I^ = X^. (18.42)
X= zb 1; (18.43)
that is, either the wave functions remain unchanged with respect
to space inversion (even functions, X= 1), or they reverse their
sign (odd functions, X= —1).
We shall now find the quantities which determine the parity of
a wave function in a central field. In the spherical coordinate
system r, ft, ?, space inversion affects only the angular part
as can easily be seen from the fact that the sign of the coordinates
T h i s o p e r a t o r c o n v e r t s a r i p h i - h a n d e d s y s t e m of c o o r d i n a t e s into a l e f t - h a n d e d system ,
a n d v i c e v e r s a . In t h e D i r a c t h e o r y I </ Cr) ^ ^
T H E DI R AC T H E O R Y OF T H E M O T I O N OF AN E L E C T R O N 303
lr;" (0, <p)=J'7(« — », ic + <p) = const P"‘ (— cos 0)e"» <*+ *>=
—(—l)'Y'in, (18.46)
because
p/n( - x ) = ( - i
- 1 = i - ( H I - I H ) = 0. (18.47)
Let us consider the motion of a free particle of spin 1/2 and con
stant momentum such as, for example, an electron. Without loss of
generality we may take the z axis to lie along the direction of
momentum; that is, in Eq. (17.20) we set
Px = Py = 0, Pz 0. (18.48)
304 R E L A T IV IS T IC QUANTUM M E C H A N IC S
k = ^ { n = 0, 1, 2, ...),
K = VW+%, =
and the 4 x 4 matrix
&'■>= (18.53)
\'2
T H E DI RAC T H E O R Y O F T H E M O T I O N O F AN E L E C T R O N 305
1
M3) = —L bM (18.54)
V2 \/2
that is, the states (18.53) differ from the states (18.54) in the
sign of the energy.
To determine the physical significance of the states bU) with
different i = 1, 2, 3, 4, let us find the projection of the spin on
the direction of motion, that is, on the z axis. First of all, we
note that, since L, = xpv— ^ * = 0 when a particle is moving
along the 2 axis, the projection of spin on the z axis must be con
served. This follows directly from the fact that the matrix s3
commutes with the Hamiltonian in (18.49).
We can find the eigenvalues of this operator o3 by applying
it to the spin functions b[,). We have then
/1 0 0
0 — 10 0
/V* V1
\ 0 = _L6(d. (18.55)
0 0 1 0 — 2° ’
lo 0 0 - 1/ \yiY'0
that is, for this solution the eigenvalue of the operator l/i a, equals
V«. In exactly the same way it is easy to show that
Thus, the four possible states correspond to the four possible com
binations of the sign of the energy and the spin direction. The solu
tion bn) corresponds to positive energy ( s = 1) and the projection of
this spin along the positive z direction (5 = 1). In a similar way we
have e = 1, s = — 1 for the solution 6(4'and we have e = — 1, s = 1
and s — —1, s = — 1 for the solutions6(3,and b(i), respectively, where
s is double the projection of the spin in the direction of the mo
mentum.
In the nonrelativistic limit (v < c), the wave functions <t3 andijj4
will be of the order of tim es the wave functions ^ and<|)2
( |3~ y ^ i) for positive energy states (e = 1). For states with nega
/> ^ ( ,+ - if
\ - v r = ~ ’ ('-4 T
which is a generalization of Eqs. (18.50), (18.53) and (18.54).
8
Problem 18.1. Show that the energy and momentum operators E = ift ^ andp = — iftV,
0
respectively, transform like a four-vector under the Lorentz transformations:
E' | „ < > E'
E 7 + PP* _ _ P* + T P _ _f
c y i __1 y j ps ’ Pv
where
Hint. Use the Lorentz coordinate transformation [Eq. (17.34)] and change to new
variables in the process of differentiation.
Problem 18.2. Prove the relativistic invariance of the scalar equation for a free
particle.
Hint. First, let us prove the invariance of the operator relation
Problem 18.3. Show that in the case of spatial rotation of the coordinate system
around the y axis by the angle 8 , the wave function transforms according to the relation
C) ^ V »
, t.__j.,
( cos + (o3 sin —J ip= e 2
Problem 18.4. With the aid of the vector model of addition of angular moments,
find the angles between j * and s* and between j* and /* taking into account the geo
metric vector addition in quantum mechanics; that is, find
cos (j*s*)\ cos
Hint. Use a method similar to the one that led to Eqs. (18.30) and (18.31).
Problem 18.5. Show that the wave function where C = ia3pa, and t];*ls
the complex conjugate (but not the llermitian adjoint, that is, noti^+) of the Dirac wave
function for an electron with negative energy satisfies the Dirac equation with positive
energy and opposite (positive) sign of the charge, that is, describes the motion of a
positron (a charge conjugate transformation).
Solution. The Dirac equation is
ft d_ i in 0 e
i dx + “2 ^Tdy~T Ay +
for the complex conjugate of this equation, we may write (taking into account the fact that
of - «i, a* = — “ 2 , of = o8, pf -- p8):
{ t + c [“■( t S + 7 ^ ) - ' (7 ^ + 7 ^) +
+ °a ~ ’!'* = 0.
We note that the complex conjugate ip* differs from <|i+ (the Hermitian conjugate): namely,
I* =
whereas
Let us substitute <jJ= io.p,^* into the complex conjugate of the Dirac equation. We
then find that IJi satisfies the Dirac equation if the charge e is replaced by - e.
-f|£ |,
4> *
Since the state <Ji(r, t) = e i}' (r ) Is treated as a state with positive energy (E = | E | ),
N (19.4)
W— »F .j.
/'t'A
» w ) = < w j ) ( J;). (i9.5)
'h' a'*p
2 m„c
N (19.6)
which holds both for the Pauli and Dirac m atrices, we find
( W i) J;) (19.8)
p l a n e ( by = 0). T h e n we o b t a i n
='l 8m?cs - ) ©
\ •
PS (19.9)
8my
Substituting (19.9) into the first of Eqs. (19.2) and neglecting terms
of the order of ( v / c )3, we find
{£ — e$ 8maca <E-C4.)p>}© =
P1 (19.10)
= (h ; t« -p ) 16m jica Hi
= PJ
where / / i s the magnetic field since the operator v acts only on the
vector potential A, and not on b.
In exactly the same way, with the aid of the relations
(o', p) (£ — eO) (o', p) = (£ — e<D) p* — ihe (o'- £) (o', p) =
= (£ — e<t>) p’ — ihe (£*p) -f- eh [o’.(£ x p) ]
and
= p ^ £ _ e<t) = ( £ _ ea , ) p ^ 2-le £.p (19.13)
, Ps _ P*_ eh
e ‘I> -------- 5 -
2m0 8 mlc- 2m0c
(o './/)-
eh
4 m lc-
[o'.(£xp)] + (19.14)
1/ r e l _________ ____
(19.15)
— 8m2ca
H = V + p V = m Qc 2 +
S= -ff\ (19.17)
0= — S. (19.18)
This Interaction may also be Interpreted from the classical point of view In the follow
ing way: a magnetic dipole moving with a velocity v (the spatial component of a tensor
312 R E L A T IV IS T IC QUANTUM M E CH A N IC S
which Interacts with the electric field of the nucleus. This additional energy of inter
action Is
eti
Vcl = — £.jiel
2/»;c: [o’. (Exp) ] (19.21)
This classical expression for the interaction energy is twice as large as the correspond
ing quantum expression [see (19.19)]. Even before the advent of the Dirac theory, an
attempt was made to explain the fine structure by the sem iclasslcal introduction of spin-
orbit interaction. To obtain an agreement with experiment. Thomas and Frenkel sug
gested that we substitute the coefficient 1/2 into the classical expression for the inter
action energy (19.21). This interaction, which follows automatically from the Dirac
theory, is called the Thomas-Frenkel correction.
= E = -°r, e = - e 0. (19.22)
(19.24)
is proportional to i *lr (0) i2, and it will differ from zero only for the s
state since, according to (13.28a), only in this case | ( 0 ) r/ 0.
For all other states (I 0) this square of the wave function vanishes
when r = Q. In this sense the contact term may be regarded as
the spin-orbit interaction for the s. state. We can see now that
the last two terms in the interaction energy (19.14) characterize
the spin properties of an electron.
Problem 19.1. Show that the matrix ca Is the velocity operator, and that In the case
of a free particle a is not a constant of the motion, unlike the momentum operator
(p = — . Explain this difference. Determine In what case the average velocity
» = c2§r. (19.26)
Hint. From the Hamiltonian (17.21) it is possible to obtain the velocity operator
. /
r = (Hr — rH) = ca,
It can also be shown that the velocity Is not a constant of the motion, since a ^6 0 . Con
sequently, if for a given k we take a linear combination of positive and negative energy
states [see Eqs. (18.53), (18.54)] there will exist interference terms that will fluctuate
with time ( ~ e— 2,cKt). As a result, Ehrenfest’s theorems will hold only on the average
in the Dirac theory. The interference terms will disappear in the calculation of the
average value of the momentum operator.
Equation (19.26) holds only if the states with positive energies are retained (e = 1).
Chapter 2 0
( 20 . 1)
xW = RniY(». (20.2)
Here Rn[ is the radial part of the wave function and is a spher
ical spinor: for / = /-)- ‘/.2the spin is parallel to the orbital angular
momentum and for j = l — ‘/a it is antiparallel.
Although terms of the order of {vjc)'1 are not formally accounted
for in Eq. (20.2), the relationship between the spherical harmonies
in the spherical spinor that determines the zero-order approxima
tion of the wave function is established by the spin-orbit interaction,
which is of the order of (vjcfl
The spherical spinor can therefore be used only when the atom
is not subject to external perturbing forces of magnitude greater
than those involved in the spin-orbit interaction. If that is not the
case, the spin-orbit coupling will be disrupted and a new set of
prem ises must be set forth in order to establish a relationship
between the spherical functions.
Spherical spinors, just as spherical harmonics, satisfy the
equation
Vo, <pYim= —; (* + 1) (20.3)
therefore, taking into account (11.17), the radial function in (20.2)
satisfies the same equation that was derived in the nonrelativistic
Schrodinger theory:
„2 d | ( 2moEn | 2 m0Zel I (/ + 1)
VA ( T ft2 T ft3 rfi Rnl= 0.
^ T h e r e a r e s e v e r a l c a s e s in q u a n t u m m e c h a n i c s in w h i c h a s m a l l i n t e r a c t i o n e n e r g y
e n a b l e s u s to f in d a r e l a t i o n b e t w e e n t h e c o e f f i c i e n t s o f th e f u n c t i o n s in t h e z e r o - o r d e r
a p p r o x i m a t io n . We h a v e a l r e a d y e n c o u n t e r e d a s i m i l a r s i t u a t i o n in t h e t r e a t m e n t of t h e
S tark e f f e c t ( s e e C h a p t e r 14).
316 R E L A T IV IS T IC QUANTUM M E CH A N IC S
= (£ •+ £ ? )* (/),
where a = tic
^ Vm is the fine structure constant.
THE FIN E S T R U C TU R E OF S P E C T R A 317
— _ Z J_ 2RhZ
a0 n 2 e'iir ’
Equation (20.8) agrees exactly with the formula for the rela
tivistic energy, which was calculated in identical approximation by
means of the relativistic Klein-Gordon equation [see (15.31)].
In a similar manner, with the aid of (19.23), we find the addi
tional energy due to the spin-orbit interaction
n2 .
~2 9 for 1^0,
S•L
o for / = o,
In these equations
/ for / = ^+ '/2 ,
?= /(/+l)-i(' + l)-s(s+l) = - ( f + l ) f o r / = *-«/*,
( 20 . 11 )
z At f i r s t g l a n c e it may s e e m t h a t t h e s p i n - o r b i t i n t e r a c t i o n , w h i c h i s i n v e r s e l y p r o p o r
t i o n a l to t h e th ird p o w e r o f t h e d i s t a n c e , c a n n o t g i v e a s t a b l e s t a t e . T h i s , h o w e v e r , i s n o t
so. At s m a ll d i s t a n c e s t h e s p i n - o r b i t i n t e r a c t i o n b e h a v e s j u s t l i k e th e r e l a t i v i s t i c i n t e r
a c t i o n ; t h a t i s , it i s i n v e r s e l y p r o p o r t i o n a l to t h e s q u a r e of th e d i s t a n c e . I n d i r e c t p ro o f of
t h i s i s th e f a c t t h a t / \ E S,°' d i f f e r s from A E rel o n ly by a n u m e r i c a l f a c t o r o f th e o r d e r of
u n ity .
318 R E L A T IV IST IC QUANTUM M E C H A N IC S
f 0 for I 9 ^ 0 ,
!" = | . for ( _ 0 . (20' 12)
Finally, the energy corresponding to the contact interaction, accord
ing to (19.24), is given by
where
[see (13.28a)], and using the fact that 1^ 1*= -^ when/ = 0 and
j = 1l2, we find
l lJr(°)l2 = i ( f ) 3’ (20.1.4)
that is,
A £ = A £ re l f- A £ s-°--|~ A £cont =
— - ______ <7"(l ~ 5/o)__
' n' [ l + lh 4 21 (l + lh)(l+ 1 )
Incidentally, Eq. (20.10) for the contact interaction may be obtained when the expres
sion for the spin-orbit interaction (Eq. 20.10) is allowed to go to the limit as / 0, if we
■*
discard the factor 5/q in 20.10. Therefore, many authors use this procedure and neglect
the contact interaction in deriving the fine-structure formula. However, the agreement be-
lw<err the two formulas is accidental since for the slates the numerator of Eq. (20.10) is
s
always zero, while the denominator vanishes only in the nonrelativistic approximation. In
a number of other problems, for example, an atom containing several electrons, the energy
associated with contact interaction is no longer a limit of the expression for the spin-orbit
interaction.
THE F IN E STR U CT U RE OF S P E C T R A 319
AEnj = - R h ^ r (20.16)
J+2
Therefore, summing both results [(20.1) and (20.16)], we obtain
the fine structure formula for the spectrum of a hydrogen-like atom5
72a 2
RhZa n (20.17)
Enj — Eh + AEnj lla
I +I —
n2
j +2
From this it is seen that the splitting of the levels is proportional
to the square of the fine structure constant.
When we take the fine structure into account the position of the
energy levels in the hydrogen atom is found to depend also on the
total angular momentum quantum number /. Therefore the terms
will be denoted in the following manner:
EnIJ RZ2 1 + £ l ( ___
n (20.18)
(ntj) n -b „» [J+ >,,
From this formula it is seen that the fine structure, according
to the Dirac theory, depends only on the principal quantum number
n and the total angular momentum quantum number /. In contrast to
the Klein-Gordon theory, it is independent of the orbital angular
momentum quantum number / (up to terms of the order of a2).
SAn e x a c t s o l u t i o n o f t h e D i r a c e q u a t i o n g i v e s t h e f o l lo w in g g e n e r a l i z a t i o n o f Eq.
( 1 5 .3 0 ), w h i c h t a k e s in to a c c o u n t t h e r e l a t i v i s t i c e f f e c t s in t h e c a s e w h e r e s p i n i s a l s o
p resen t:
E nj = m 0c 1+ i 0c ( 2 0 .1 7 a )
( n - j - ■/, + V o' + 54)2 - z V ) 2_
E q u a t i o n (2 0 .1 7 ) may b e o b t a i n e d from ( 2 0 . 1 7 a ) , if t h e l a t t e r i s e x p a n d e d in a s e r i e s a n d
w e r e s t r i c t o u r s e l v e s to t h e f i r s t two term s.
S i n c e t h e minim um v a l u e of j i s e q u a l to 1 / 2 , w e fin d t h a t s t a b l e m o tio n in t h e C o u lo m b
f i e l d o f a p o i n t n u c l e u s , a c c o r d i n g to t h e D i r a c th eo ry , w ill e x t e n d to Z cr = 137, w h e r e a s
in t h e K le i n - G o rd o n th e o r y i t w a s l i m i t e d b y Z cr = * 137 [ s e e ( 1 5 .3 3 )]. Such a n i n c r e a s e of
Z cr i s , a s w e h a v e a l r e a d y m e n t i o n e d , d u e to t h e s l i g h t c o m p e n s a t i o n of t h e r e l a t i v i s t i c
e f f e c t s by t h e s p i n e f f e c t s .
320 R E L A T IV IS T IC QUANTUM M E C H A N IC S
The diagram given in Fig. 20.1 shows that all terms are doubly
split, since to each value of I there correspond two values of /;
for example, instead of a single term 2p (/= 1 ) we now have two
terms 2pn, and 2p.,,.,. The exceptions
■E /h =■0 are the s terms (1 = 0), for which j
can have only one value (j = '/.2). Thus
the relativistic and spin effects som e
--------------------3ds/2 what reduce but do not split the s terms
3 /23j>3/2,3d (see Fig. 20.1).
3st/2, 3p,/2
The degree of degeneracy also
changes owing to splitting of the energy
----------------------------------3-P 3/2 _ levels. We know that the principal
3s t/2, 3Pt/2 quantum number may take the following
values: n = l , 2, 3, 4 ,___ The orbital
*St/2 angular momentum quantum number I
varies from 1= 0 (s state) to I = n—1.
F i g. 20. 1. Energy level dia The total angular momentum quantum
gram of t h e h y d r o g e n atom. number / takes the values j — lzh '/.2(I ^ 0)
and j — l/i (l = 0) and, finally, the mag
netic quantum number m;-= — / ....... that is, for a given / there
are 2 /+ 1 half-integral values of tiij. The degree of degeneracy,
which is characteristic for any central field of force and is related
to the equivalence of the various directions in space, is therefore
equal to 2y'-f-1 for particles with a spin of 1/2 (we remember that
for spinless particles it was equal to 2/-f- 1). In contrast to the rela
tivistic spinless theory, the degeneracy with respect to I is still
present when we take into account terms of the order of a2 and
even the following expansion terms proportional to a4. When the
finite size of the nucleus is taken into account, the degeneracy with
respect to / is removed. We note incidentally that even greater
splitting with respect to I is due to vacuum fluctuations (see
Chapter 22). The magnitude of the splitting of spectral lines can
be determined from the selection rules (20.4). For the Lyman
series we then have two lines (instead of a single one):
(2sVa) = (2pVa) = | [ l + j ( 2 -
(2pa/a) = ^ [ l + a- ( l
( 20 . 21 )
which is about 1.095 • 104 Me.6 If only the relativistic effects are
taken into account (Klein-Gordon equation) the corresponding
splitting is [see (15.32)]
AcoK-G = (2 s ) — (2 p ) = = 4 l6 - ; (20.23)
that is, the frequency is almost three times greater than the one
found from the Dirac theory. Consequently, the spin properties
of particles somewhat reduce the influence of the relativistic
effects.
The conclusions of Dirac’s theory have been accurately con
firmed by experiment.
It is interesting to note that the fine structure of the spectrum
of the hydrogen atom was first theoretically calculated by Sommer-
feld who applied a relativistic Hamiltonian to the steady states
of the Bohr classical theory. Sommerfeld obtained [see (2.61)]
6 1 Me = 1 0 6 s e c t h a t i s , t h e a n g u l a r f r e q u e n c y a), e x p r e s s e d in s e c is connected
w ith t h e f r e q u e n c y v, e x p r e s s e d in Me, by t h e r e l a t i o n
0) = 277 • 10 V
322 R E L A T IV IS T IC QUANTUM M E C H A N IC S
By m i c r o w a v e u l t r a h i g h - f r e q u e n c y r a d i o e m i s s i o n we m e a n t h e r e g i o n o f t h e e l e c t r o
m a g n e t i c s p e c t r u m l o c a t e d in t h e w a v e l e n g t h r a n g e from m i l l i m e t e r s to t e n s o f c e n t i m e t e r s
( 1 0 6 — 1 0 3 Me). S u c c e s s f u l a p p l i c a t i o n o f m i c r o w a v e s p e c t r o s c o p y to t h e i n v e s t i g a t i o n of
a t o m i c s p e c t r a i s d u e to t h e f a c t t h a t t h e d i s t a n c e s b e t w e e n t h e c o m p o n e n t s o f t h e l e v e l s
s p l i t by t h e r e l a t i v i s t i c , s p i n a n d v a c u u m e f f e c t s are o f t h e s a m e o r d e r o f m a g n i t u d e a s th e
w a v e l e n g t h s in t h e m i c r o w a v e re g io n .
THE FINE STRUCTURE OF SP E C T R A 323
F i g . 2 0 .2 . D iag ram o f t h e L a m b - R u th e r f o r d e x
p e r i m e n t s on t h e d e t e c t i o n o f t h e s p l i t t i n g o f
t h e 2si/, a n d 2pi/, l e v e l s : 1) t u n g s t e n f u r n a c e
e m i t t i n g a b e a m o f h y d r o g e n a t o m s ; 2 ) b e a m of
e l e c t r o n s e x c i t i n g t h e h y d r o g e n a to m s; 3 ) r a d i o
f r e q u e n c y f ie ld ; 4 ) t a r g e t ; 5) g a lv a n o m e t e r .
T h i s i s c o r r e c t for a d i p o l e t r a n s i t i o n , b u t c a l c u l a t i o n s h o w s t h a t t h e q u a d r u p o l e
t r a n s i t i o n b e t w e e n t h e s e s t a t e s i s a l s o f o rb id d en .
324 R E L A T IV IS T IC QUANTUM M E C H A N IC S
1
•2s,>/2
1058
2 P t/? 2S//J, 2p,/2
b
1'ig. 20..'i. S p l i t t i n g o f e n e r g y l e v e l s in t h e h y d r o g e n
atom, a) e x p e r i m e n t a l d a t a ; b) a c c o r d i n g to t h e D i r a c
theory ( n e g le c tin g vacuum effe cts). T h e fre q u e n c ie s
of th e c o r r e s p o n d i n g t r a n s i t i o n s a n d t h e d i s t a n c e s
a r e g i v e n in Me.
In t h e e x p e r i m e n t s of L am b a n d R u t h e r f o r d , t h e f re q u e n c y of t h e m i c r o w a v e r a d i a t i o n
w a s f i x e d a n d t h e r e s o n a n c e c o n d i t i o n , c o r r e s p o n d i n g to t h e d i f f e r e n c e in t h e Z e e m a n co m
p o n e n t s b e t w e e n th e s t a t e s 2>si/, a n d 2/d/, or 2p y , w a s o b t a i n e d by a d j u s t i n g t h e m a g n e t i c
f i e l d ■Z /C . T h e n , e x t r a p o l a t i n g t h e r e s u l t s to th e c a s e 'O iT -- 0, t h e a u t h o r s f o u n d t h e l e v e l
sh ift.
THE F I NE STR U CTU RE OF SP E C T R A 325
Jo
2m0 2w0 lll0C 2hi. d-i (20.25)
where Frel, F s>0* and l/cont are given by Eqs. (19.15), (19.23) and
(19.24), respectively. Upon averaging of these terms
H , - _ » i £ ( y £ * .- f ) . (20.28)
pmag — ^- + 0 3 ], (20.29)
^ G e n e r a l l y s p e a k i n g , t h e s p i n - o r b i t i n t e r a c t i o n V s ’0, i s o f f u n d a m e n t a l i m p o r t a n c e in
th is c a s e . Since, how ever, the r e la tiv is tic term s are of th e sa m e order a s the spin-orbit
interaction, we may se t
s . 0 . •v AE
AE nj ■
326 R E L A T IV IST IC QUANTUM M E C H A N IC S
which gives the following value for the additional energy of the atom:
(Ff)*(y™)d2 = 1.
= ij-o y f (m v 2 ) 2t + \•
THE FI NE STR UCTURE OF S P E C T R A 327
Recalling that m/ = in— '/* * the last two expressions may be written
as a single formula
A£mag_ ^yjTgrtij = ofignij, (20.33)
t= 'M - <2°-34>
F i g . 2 0 .4 . Z e e m a n e f f e c t : a) p o s i t i o n o f t h e e n e r g y l e v e l s
in t h e a b s e n c e o f a f ield ; b) a n o m a l o u s Z e e m a n ef fe c t;
c ) n o rm al Z e e m a n e f f e c t .
g° = 7s. g = 2.
Hence
. 2 . 4
All)! = CD, — C00= - j0 , A co.2= — 0,
Acd3= -^-
Oo, A<d4= — u o. (20.36)
where L, 5 , and J are the orbital, spin and total angular momenta
of the atoms and
J = \L±S\.
gs = 2. (20.38)
For atoms with two electrons in the outer shell (for example,
helium atoms), single lines (S = 0, J = L) are possible along with
the triplet state S = l . For the single lines we have g = 1, and,
therefore, in this case spin effects should be of no importance;
only the normal Zeeman effect (that is, triplet splitting) should be
observed in either a weak or strong field.
THE FINE STRUCTURE OF SPECTRA 329
In the latter case we first solved the problem by taking into account
the spin-orbit interaction; this establishes a relation between
the spherical harmonics that form the spherical spinor (18.24) or
(18.25); then we found an additional energy that leads to the
anomalous Zeeman effect, since the Lande g factor does not equal
unity.
In the case of strong fields, when the splitting due to the external
magnetic field is greater than that due to the spin-orbit interaction
the magnetic field “ breaks” the spin-orbit coupling and the zero-
order approximation solutions, expressed in terms of spherical
spinors [see (18.24 and 18.25)], are no longer true.
In this case we may neglect the interactions Frel, Fs'°* and
p c°m (20.26), which, when (20.29) is taken into account, becomes
Ze\
r ^ - - Na5 r (m+ l ) ) T 1= 0,
Zel (20.41)
£ -f
Since the interaction between the atom and the external magnetic
field (20.29) contains only the matrix oj, which does not couple the
wave functions lF , and ' F j , transitions from the state with m s = ‘/ a to
the state with ms = — */2 > Educed by this interaction, will be for
bidden in this case and hence Ams= 0.
Taking into account this circumstance, and also the selection
rules for the quantum number m (Am= 0, rkl), we find from (20.42)
an expression for the Zeeman splitting of the spectral lines
which In conjunction with (20.33) gives the following expression for the additional energy
of an electron In a magnetic field:
Here when calculating the perturbation energy proportional to x 2 + y 2, we have used the
spherical symmetry, which must occur in the zero-order approximation(g/f = 0),an d
have set
Problem 20.2. Adding geometrically the orbital and spin angular momenta, show that
the anomalous Zeeman effect Is associated with the fact that the total magnetic moment
p Is not parallel to the total angular
momentum J. With the aid of the geo
metric model, also explain the Paschen-
Back effect.
Solution. First let us find geometri
cally the angular and the magnetic mo
ment vectors
J = L -(- S; (i = ■
2 ^ L+ 2S>-
In the geometric representation (see
Fig. 20.5) ‘we may choose the scale so
that u. = — „ g° (it is immaterial at
2m0c
the moment whether the vectors are
parallel or antiparallel).
Then
F i g . 2 0 .5 . G e o m e t r i c i n t e r p r e t a t i o n o f t h e
a n o m a l o u s Z e e m an e f f e c t . f-i = L, and pJ = 2S;
that is, the total magnetic moment ji will undergo two rotations in a magnetic field: one
with an angular velocity u around the direction of the total angular momentum (this
angular velocity corresponds to the frequency associated with the transition between
components of the spin-orbit splitting of the spectral lines <u / ? a ‘J), and the other corre
sponding to the Larmor frequency of precession around the direction of the magnetic
field H (H X = H V= 0, II, = p5T).
When m ;j> o, the additional energy should be calculated from the relation
mag __(|t • J) (J • H)
A £ mag _ gohmj,
In strong fields o « we must consider Independently the rotation of the orbital and
the spin moments |i around the 2 axis. Then the additional energy becomes
Problem 20.3. By means of the relativistic scalar wave equation and the Dirac equa
tion, find the frequency of the allowed transitions between the n = 2 and n = 3 states.
Show that, according to the Dirac theory, there are seven lines, five of which are distinct,
and that, according to the scalar theory, there are only three distinct lines.
Chap ter 21
As has been mentioned in Chapter 13, the position of the spectral lines is shifted when
the finiteness of the nuclear m ass is taken into account. The Rydberg constant R in the
expression for the energy of a hydrogen-like atom
R nz«
( 21. 1)
n~ n?
is somewhat reduced and becomes equal to
( 21. 2 )
where
D
*°° 2h3
is the Rydberg constant corresponding to infinite nuclear m ass. Consequently, the
Rydberg constant will have somewhat different values for hydrogen, deuterium, and
tritium. With the great accuracy attainable in modern spectroscopic techniques, this
effect can be used to detect the presence of different isotopes (see Chapter 13).
In a sim ilar fashion, the finite size and the magnetic moment of the atomic nucleus
have certain effects on atomic spectra.
Zel ,'3_ _ J_ r 2
for r < R A'. (2U)
X'A* 2 **A.
T H E E F F E C T O F N U C L E A R S T R U C T U R E ON A T O M I C SPECTRA 335
The shift of energy levels due to the finite size of the nucleus can be calculated with
the help of perturbation theory. We shall assume that the perturbation energy consists
of the difference between the potential energy of a point nucleus and the potential energy
of a nucleus with charge uniformly distributed over the nuclear volume
(2 1.5 )
hr
*5 *(r).
with
1 for r < R V)
e (21.6)
0 for r Rn.
The perturbing force therefore differs from zero only inside the nucleus,
Since | ^ |2 does not change appreciably in the region r ^ R t this Integral can be readily
evaluated by substituting for | <{/12 Its value at the origin.
Rn
i£V<i ', [7 -jfe(T- 7 &)] dr--
2n ( 21. 8 )
j Z W ' K O ) I*.
336 R E L A T IV IS T IC QUANTUM M EC H A N ICS
AE M l - (21.9)
a\n* ’
that is, in nonrelativistic theory, the shift of the energy levels is different from zero only
for s states (/ = 0;.
( R A*
—- I
n* °n
(where an — a ^ -p ) , and for d states a factor of the order of j , and so on. *
1 Con
sequently, the shift in the energy levels for p and d states can be neglected in the first
approximation.
For hydrogen, the first-order shift In the energy levels Is about 1 Me; this Is much
too small to account for the Lamb shift, which Is equal to approximately 1,057 Me (see
Chapter 20),
The volume of the nucleus Is important In connection with the Isotope shift, that Is,
the shift In the energy levels of atoms with the same atomic number Z and different
m ass numbers A . The chief factors that give rise to the Isotope shift are the different
m asses (the mass effect) and different volumes (the volume effect) of the Isotopes. The
m ass effect Is manifested In a shift of the spectrum lines towards the ultraviolet as the
m ass number A Increases. For example, for Z = 1 the highest frequencies are found
in tritium with A = 3, then deuterium with A = 2', and, finally, ordinary hydrogen with
A — I [see (21.2) and also Chapter 13]. On the other hand, the volume eftect is mani
fested In a shift of the spectrum lines towards the Infrared as A Increases. For Instance,
it can be seen from Eqs. (21.1), (21.9) and (21.13) [see below], that the energy levels of a
hydrogen-like atom will be given by the following expression when the shift due to the
volume effect is taken into account:
RZAh 1 2lR
5 a\n
Experiment shows that an Isotope shift towards ultraviolet Is observed In elements whose
atomic number Z Is le ss than 40-50. For elements with a larger value of Z an isotope
shift in the opposite direction Is observed, that Is, towards the Infrared. This Indicates
that, for relatively light elements, the Isotope shift is caused mainly by the mass effect,
whereas for heavier elements It Is caused by the volume effect. This, however. Is a
rather simplified picture of the isotope effect, and other features associated with the
structures of the atom (for example, nuclear spin and polarization of nuclei by electrons)
also have to be taken Into account.
C. MESIC ATOMS
The finite size of the nucleus has a particularly Important effect on the position of
energy levels in a m esic atom—an atomic system consisting of a fi meson revolving about
a nucleus. The }i meson is a particle that has the same spin as the electron (that is, spin
T o c a l c u l a t e t h e s h i f t o f t h e p l e v e l s w e m u s t s u b s t i t u t e t h e s e c o n d le r m in t h e e x -
p a r s i o n o f k l( r ) i n t o ( 2 1 . 7 ) , n a m e l y ,
s i n c e t h e m a i n te r m | i / / ( 0 ) |2 v a n i s h e s fo r p s t a t e s .
T H E E F F E C T O F N U C L E A R S T R U C T U R E ON A T O M I C S P E C T R A 337
1/2 in units of /l) and a mass 207 times greater ( m |1 = 2 0 7 m „ ) , so that the p meson Is
basically a "heavy” electron. Meslc atoms can be produced by passing negative p mesons
through matter. After losing Its energy and slowing down, a p meson may be captured
In an orbit about a nucleus, forming In this way a ji-rnesic atom. Meslc atoms have
been obtained for almost all elements of the periodic system, from hydrogen up to
the heavy elements (uranium, neptunium, and so on).2
The motion of a p meson about the nucleus Is determined mainly by the Coulomb
attraction. Just like the motion of an electron. A p meson, however, also has nonelectro-
magnetic Interactions with the electron—neutrino and nuclear fields; these Interactions
may result In spontaneous decay of the p meson into an electron, neutrino, and antineutrino
(the lifetime of a p meson at rest is t = 2.2 x 10~6 sec). The p meson has, in addition,
a definite probability of being captured by a nucleus. Thus the lifetime of a p-m eslc
atom Is determined by two competing processes: natural decay of a p meson Into an
electron, neutrino, and antineutrino and nuclear capture of the p meson. In light meslc
atoms ( Z < 10 ) the probability of the first process Is greater than the probability of
the second; that is , the lifetime of a meslc atom Is determined by the lifetime of a
p meson at rest ( t ^ l O -0 sec). For Z >■ 10, nuclear capture begins to predominate
and the lifetime slowly decreases to ~ 7 -10“9 sec (for Z = 82).3
In the theory of p -m eslc atoms, ordinary electrostatic Interaction plays a funda
mental role. In the first approximation, we can regard the nucleus as a point charge
and calculate the energy of the meslc atom and the radius of the orbit using the equations
derived for an ordinary hydrogen-like atom, replacing the electronic mass by the mass
of a p meson. Then the energy and radius of the orbit will be given by [see (13.33),
(13.45)]
_ ft2 na
( 21. 10)
' r"—m~el Z ’
where w|1= 207w0. It can be seen that the energy of the p meson In a m eslc atom Is
207 times greater than the corresponding energy of an electron In the atom, and that the
radius of the orbit on the contrary is reduced by the same factor. If electrons remain
In the atom along with the p mesons, they will move about the nucleus In considerably
larger orbits than the p meson and therefore cannot exert a significant Influence on the
p meson rotating around the nucleus. A meslc atom, therefore, may be regarded as a
hydrogen-like atom that can have both large and small values of Z.
Since the radius of the "Bohr” orbit of the meson Is 207 times smaller than that of
the electron orbit, the probability that the meson will be located In the nucleus is con
siderably greater than for an electron in a hydrogen-like atom. The main correction to
the energy levels of a meslc atom will therefore come from the volume effect. The
equation for the energy of s states can be obtained from (21.9) and (21.10) by replacing
the Bohr radius aa by the corresponding radius In a m eslc atom
7l2
a
f- (21. 11)
obtaining, therefore.
F _ m ^7Jel /. 4 Z*R*N
( 21. 12)
— 2 h 2n 2 \ 5n o 2
2 T h e s e s u b j e c t s a r e t r e a t e d i n m o r e d e t a i l in a p a p e r b y D. D. I v a n e n k o a n d G. E. P u s -
t o v a l o v a , U s p e k h i f i z i c h e s k i k h n a u k , 6 1 , 27 (1 9 5 7 ) .
^ F o r 7 t m e s o n s , w h i c h s t r o n g l y i n t e r a c t w i t h n u c l e i a n d a r e r e s p o n s i b l e fo r n u c l e a r
f o r c e s , t h e l i f e t i m e w i t h r e s p e c t to d e c a y i n t o a y m e s o n a n d n e u t r i n o i s e q u a l to 2 . 6 x
i c r 8 s e c , w h e r e a s t h e l i f e t i m e w i t h r e s p e c t to c a p t u r e b y a n u c l e u s i s m a n y t i m e s s m a l l e r .
In p a r t i c u l a r , f o r 7 7 - m e s i c h y d r o g e n t h e c a p t u r e t i m e o f a n e g a t i v e p i o n fr om t h e o r b i t i s o f
th e o rd e r o f 1 0 sec.
338 R E L A T IV IS T IC QUANTUM M EC H A N IC S
Using the fact that the nuclear radius Is related to the m ass number A ~ 2Z by the
expression
It follows that, for the s levels, the energy correction due to the nuclear volume will
be proportional to 2 */a • and therefore attains very large values for heavy elements.
In heavy elements, the orbit of a p meson may even be inside the nucleus, at large
Z and small n. The Bohr radius of the m esic atom becomes equal to the nuclear radius
for Z = Z cr = 45
For orbits inside the nucleus (Z > Z c r ) , the main part of the potential will no longer
be determined by the Coulomb law (21.4) but instead by formula (21.3) which corresponds
to the potential of a three-dimensional harmonic oscillator (on the assumption of a
simplified, model of the nucleus in which the charge is uniformly distributed over the
volume). Thus the energy has to be determined from the following equation Instead of
(13.4):
d2R . 2 dR . 2m^
dr 2 ^ r dr + ft2' X
1 Zelr 2 /( / 4- I)ft3 (21.15)
x ( £ + 2 TiV 2 R\N 2m^r- I = 0.
vS/? = 4p ^ f ^ ' + l W 3_ \
P I dp2 ^ p dp ^ 16p2 H )
it follows that the equation for the energy of a three-dimensional harmonic oscillator is
formally identical with the equation for the hydrogen-like atom
d 2R ‘ , 2 dR 1 (21.16)
dp2 p dp + H + T - f ) s '= ° ’
but has different values of the constants, namely,
-i('-T)-
To determine the eigenvalues of the energy we may use Eq. (13.20), according to which
= e + k4
V a
Substituting for /?, A and I' their values from (21.17) and using the fact that k = n I 1,
we can find the energy of the meson in an orbit inside the nucleus:
o+ ft“ (2/j f / + 3i), (21.18)
T H E E F F E C T O F N U C L E A R S T R U C T U R E ON A T O M I C S P E C T R A 339
Since the wave function of a spherical oscillator Is sim ilar to the wave function for a
Coulomb field (It Is determined from the same wave equation with r replaced by p = r-),
It decreases exponentially as r Increases. The
correction (21.19) is therefore significant only . — 2S t/ 2
when the radius r„ of the meslc atoms Is close
to R s
As an example, let us consider the 2/? —* Is 2P3/2
transition in lead {Z = 82). If the m eslc atom 2 P,/2
of lead Is assumed to have a point nucleus, the
energy released In this transition can be found tS ,/s
from (21. 10)
F i g . 2 1 .2 . E n e r g y l e v e l s in a m e s i c
3 m Z-p 1
E, — El = = 14 Mev- (21.20) atom.
The relativistic and spin effects in the 2p , /a — Is1/o transition increase this energy by
the amount
A (A E )^ 2 Mev. (21.21)
Incidentally, such a significant role of the relativistic and spin effects Is due to the fact
that the energy Is expanded in terms of(Za)s. This quantity Is comparatively large for
lead (Z = 82, a = 1/137). Comparison with experiment shows, however, that In this
transition an energy of 6 Mev Is liberated Instead of the predicted 16 Mev. This dis
crepancy between theory and experiment arises because the Is state for lead lies Inside
the nucleus. In the Ip state the orbit lies outside the nucleus and the volume effect of
the nucleus Is small. If we take the energy of the Is level from (21.18), and the energy
of the 2p level from (21. 10) for a point nucleus, the energy of the 2p — Is transition
will be 3.6 Mev. If we add to this the correction (21.19) for the energy of the Is level,
the energy of the transition will come to about 5 Mev, which Is relatively close to the
experimental value.
A study of the multlplet structure of the 2p level In meslc atoms enables us to
determine the spin of a p meson. For a particle with Integral spin, the level splits
Into an odd number of components (for spin 0 no splitting occurs, for spin 1 three lines
are observed, etc.). Since the 2p level splits Into two components (2pi/a and 2/73 ., ), It
was established that the spin of a p meson is 1/2. On the assumption of a point nucleus,
the theoretical splitting calculated from Eq. (20.17) should amount to about 0.55 Mev,
with the 2pi, and 2si/a levels coinciding. When the finite size of the nucleus Is taken
Into account aln a meslc lead atom, the splitting of the 2/73/2— 2pi/a levels Is reduced
to 0.2 Mev and the 2sJ/2 level Is raised above the 2p ,/o level. This is Illustrated In
340 R E L A T IV IS T IC QUANTUM M EC H A N IC S
Fig. 21.2. From the above data It follows that heavy m eslc atoms will emit gamma
quanta having energies of several Mev. Lighter meslc atoms emit x-rays.
Because of the significant influence of the size of the nucleus on the spectra of heavy
m eslc atoms, the charge distribution Inside the nucleus can be determined from an
analysis of the spectra. It has been found that the value that should be substituted for
Ro in the formula for the electromagnetic radius of a nucleus with mass number /I:
R n = R (,A'/‘ (21. 22)
is L 2x 10“ 13 cm rather than 1.4 x 10-13 cm (the value assumed for nuclear Inter
actions).
Similarly, the multiplet structure of the spectral lines of m eslc atoms can be used to
determine the magnetic moment of the p meson; Its value is close to the muon magneton
ge
(21.23)
2m^c
where the Lande g factor Is £ = 2.
The theory of ji -m eslc atoms Is based mainly On electromagnetic Interactions. By
contrast. In the theory of v -m eslc atoms, a great part Is played by the nuclear Inter
actions, the theory of which Is far from complete. Further experimental study of
7r-meslc* atoms and an explanation of the semlempirlcal laws that describe their be
havior will have important bearings on future work in the theory of nuclear forces.
These topics, however, lie beyond the scope of this textbook.
The Dirac equation describes the motion of particles with spin 1/2. It applies to
electrons as well as to protons and neutrons. In the presence of an electromagnetic field
it is necessary to take into account the charge of the proton, as well as the so-called
anomalous magnetic moments of the proton and the neutron. We recall that the energy
of interaction between a charged Dirac particle and an electromagnetic field is
Ve = e<b— e a . A . (21.24)
In the nonrelatlvistlc approximation this expression contains a magnetic moment due to
the intrinsic (spin) angular momentum ( h / 2 a)
eh
V-e = o. (21.25)
2mac
This quantity Is known as the kinematic or Dirac magnetic moment. In passing to the
relativistic equation, we must replace the mass w„in Eq. (21.25) by its relativistic value
mo
~ y —-y--, and, therefore, the Dirac magnetic moment vanishes as the velocity approaches
V *—
the velocity of light (v ~ c).
In addition to the Dirac magnetic moment, which appears only In the nonrelatlvistlc
approximation and which depends on the charge, a particle may have an anomalous
magnetic moment that does not vanish even In the relativistic case and is Independent
of the particle’s charge.
We shall now find the energy of interaction due to the anomalous magnetic moment.
The energy of Interaction (21.24) of an electron with an electromagnetic field Is a
scalar quantity, since In four-dimensional space i4>= At, Ax = A t, A y = A«, A: = AS.
In the same way the unit matrix I Is the fourth component of the velocity matrix a |i
(that Is, a, = (T).4 The Interaction energy (21.24) may, therefore, be represented as a
scalar quantity In four-dlmenslonai notation
Ve = —e ^ V 'V (2L26)
ii=i
^ M o re p r e c i s e l y , t h e q u a n t i t y j ^ < > v.' b + a. ^b | Se e ( 1 7 . 3 2 ) 1 , w h e r e a a l , 2 ,3 , a 4 w ill
transform a s a four-vector.
THE EFFECT OF NUCLEAR STRUCTURE ON A T O M IC SPECTRA 341
The Interaction energy of the anomalous magnetic moment with the electromagnetic field
Is, therefore, given by
•i
= n y V 'V - (21.29)
a 12 = — i p 2a 2, a ]3 = — [ P 2 33
(£•1*«3U)
are the matrix elements forming a second rank tensor. The energy of Interaction between
the anomalous magnetic moment and the electromagnetic field takes the form
l/m = F[P3='-// + P2i- £ l. (21.31)
An electron has a charge, a spin, and also a Dirac magnetic moment. Its anomalous
magnetic moment Is relatively small (see below). A neutron has no charge, but it does
have an anomalous magnetic moment; this magnetic moment determines the interaction
between the neutron and the electromagnetic field. As for the proton, it has both a
charge and a spin, and hence a Dirac magnetic moment; in addition, it has an anomalous
magnetic moment. It should be noted that nuclear interactions are of great importance
in the theory of nucleons.
The procedure for determining the magnetic moment of the neutron, proton, and
complex nuclei is basically the same as for the magnetic moment of the electron (the
Stern-Gerlach experiment). The basic principle consists in applying a magnetic field
perpendicular to the direction of motion of the particle. The particle will react dif
ferently depending on whether Its magnetic moment is oriented parallel or antiparallel
to the field.
Let us first consider the possibility of determining the Dirac magnetic moment
and the anomalous magnetic moment of a free particle. Suppose a free particle moves
perpendicularly to the z axis. The Hamiltonian describing its motion has the form
The component of the intrinsic angular momentum perpendicular to the direction of this
motion
(21,33)
S‘ = 2 n3
does not commute with this Hamiltonian. The component of the total angular momentum
along the z axis does commute with the Hamiltonian
J z = y V x — x V 3/ + \ f a z , (21.34)
and can therefore be determined exactly together with the energy.
Let us evaluate the error in the determination of the orbital angular momentum by
means of the uncertainty relation. If the origin of the coordinate system Is taken to be
at the center of the wave packet, the error in the orbital angular momentum will be
AZ.* A yipx — AxApy.
In accordance with the uncertainty relation, we have Apx ~ — , Apy ~ — . Since the
+ (■ $ )• (21-35)
The error A[,z will be minimum when | Ax | = | Ay |. Thus the error AL: due to the
translational motion of the particle is of the order of the spin, and therefore the per
pendicular components of the intrinsic angular momentum and the Dirac magnetic
moment cannot be determined simultaneously.
We recall that the Stern-Gerlach experiment allowed for the determination of a mag
netic moment of a bound electron; since, however, the orbital angular momentum in the
s state is zero, it was the spin (or Dirac) magnetic moment that was actually measured.
In accordance with (21.31), the interaction energy associated with the perpendicular
component of the anomalous magnetic moment is
This component commutes with the Hamiltonian (21.32) and can therefore be measured
exactly. Consequently, it is possible to measure the magnetic moment of a free neutron
when the magnetic field is perpendicular to its motion, as was done by Bloch and Alvarez
(1940).6 In their experiments when a beam of neutrons was passed through a piece of
magnetized iron ,7 the most pronounced scattering was observed for those neutrons whose
magnetic moment was parallel to the magnetic induction vector inside the iron. The
emerging beam consisted, therefore, mostly of neutrons whose magnetic moment was
antiparallel to the magnetic induction vector. If the neutron beam passes now through
two magnetized iron plates in succession, the experiment is completely analogous to
the transmission of light through two Nlcol prisms; that is, the first iron plate acts as
S i m i l a r l y , it m a y b o s h o w n t h a t o n l y t h e l o n g i t u d i n a l c o m p o n e n t o f s p i n c o m m u t e s
w i t h t h e H a m i l t o n i a n . In p r i n c i p l e , t h e r e f o r e , it a l s o c a n b e m e a s u r e d e x p e r i m e n t a l l y .
7N e u t r o n s h a v e n o e l e c t r i c c h a r g e a n d t h e y p a s s q u i t e f r e e l y t h r o u g h m a t t e r . A c t u a l l y ,
t h e i r o n l y i n t e r a c t i o n s o c c u r on c o l l i s i o n s w i t h t h e n u c l e i .
T H E E F F E C T O F N U C L E A R S T R U C T U R E ON A T O M I C S P E C T R A 34 3
a polarizer and the second as an analyzer. This phenomenon was used to determine the
magnetic moment of the neutron.
A schematic diagram of the Bloch-Alvarez experiments Is given In Fig. 21.3.
Unpolarized neutrons moving along the direction of the x axis pass through the
polarizer (first Iron plate, with a magnetic induction vector directed upwards). The
emerging beam consists mostly of neutrons whose magnetic moment Is directed down
wards. These polarized neutrons will pass freely through the analyzer if its magnetic
Induction vector Is directed upwards, like that of the polarizer. On the contrary, they will
be transmitted much more weakly if the iron plates are oppositely magnetized.
Between the polarizer and analyzer there Is a device that reorients the neutron
spin. This device Is sim ilar In principle to the one used by Rabl in his nuclear magnetic
resonance experiments to determine the magnetic moment of the proton and of heavier
nuclei. The basic principle of the Instrument Is as follows. In the space between the
polarizer and the analyzer, a relatively strong, constant magnetic field Is applied
parallel to their magnetization vectors (see Fig. 21.3). A neutron whose magnetic
moment is antiparallel to this magnetic field acquires the additional energy
If, however, the magnetic moment Is parallel to the field, the neutron loses an energy
shows that the magnetic moment of the neutron, Just as for the electron, Is directed
opposite to the spin.
we obtain
= 6(r), (21.42)
where S is the absolute value of the total spin, which is equal to either zero (antiparal-
lell spins) or to unity (parallel spins). Then
| b ' 2 + ^ 2 + 2(o'-o;)] = S ( S + I ) .
we obtain the following expression for the shift of s levels (hyperfine structure) of
hydrogen:
A lls = N pp J - [2 S (S + 1) - 3] , ( 2 1. 4 5 )
%2
where a0 = -----j- is the radius of the first Bohr orbit, and the value of | ^ (0) | 2 is taken
m0ei
from Eq. (20.14),
Two cases should be distinguished:
(1) Spins of the proton and the electron are antiparallel (S = 0); then
i £ 5 = 0 = - 8 H-oFp— • (21.46)
(2) Spins of the proton and the electron are parallel (S = 1); then
8 1 (21.47)
iES=l— 3 n»a» •
The difference between these levels represents splitting of the s term due to the
Interaction between the electron and the magnetic moment of the nucleus
AES= i A/; 5=0 _ 32 poFp _1_
Aoj= 3 % n 3a3'
(21.48)
If we use (21.48) to calculate the s-term splitting for the case n = 1, then substituting
the value of pp obtained in Rabi’s experiments and setting p0 equal to the Bohr mag
neton, we find
Autheor= 1417 Me.
On the other hand, a careful experimental verification of the splitting of this level by
microwave spectroscopic methods has given the value
The relativistic corrections and the corrections for the finlteness of the nuclear mass
do not raise the frequency iu ^ 601" to the required value AwexP . The proton’s
magnetic moment has also been measured very accurately. To explain this anomaly,
therefore. It remained to assume that the magnetic moment of the electron is not exactly
equal to the Bohr magneton, but is instead somewhat larger. Kusch and Foley showed that
to obtain agreement with experiment the magnetic moment of the electron must be taken
to be
8 = 0.00116.
These considerations show that an electron will have a very small anomalous mag
netic moment p ^ om = p05 in addition to the Dirac or kinematic magnetic moment
(— po). We shall discuss the nature of the anomalous magnetic moments in the next
chapter.
Concluding this section, it is necessary to point out that the hyperflne structure can
not explain the Lamb shift of the 2si/2 level (1.058 Me relative to the 2/» i /2 level). First
of all, itfollows from (21.48) that the splitting of the 2si /2 term Is of the order of 200
Me, and, in addition, the center of mass of the s terms is not shifted. Suppose, the
level with S = 1 (parallel spins of the electron and the proton) is raised by a certain
amount [see (21.47)]; then the level with S = 0 (antiparallel spins of the electron and
the proton) is lowered by an amount three times as large [see (21.46)]. Since a state
with S = 1 is three times more probable than a state with S = 0 (when S = 1 the
spin may be directed along the z axis, opposite to the z axis and perpendicular to the
z axis), the center of mass of the s states remains unaltered, so that It occurs in the
same position as when the magnetic moment of the nucleus is neglected. The hyper
flne structure cannot therefore account for the Lamb shift. The theory of this effect
will be considered In the next chapter.
Problem 21.1. Find the shift of the Is levels in the light p-m esic atoms as a result
of the influence of the nuclear structure, taking into account the nuclear motion and
also the variation of the wave function within a distance comparable to the size of the
nucleus.
Hint. The shift of energy levels should be calculated by the perturbation method. We
use Eqs. (21.5) and (21,7) where the wave functions are those of hydrogen-like atoms with
the meson mass substituted for the electron mass. Because of the finiteness of the
nuclear m ass, the motion of the nucleus has much greater Influence on the position of
the energy levels of a meslc atom than on the levels of an ordinary atom, it is there
fore necessary to use the reduced meson mass in more accurate calculations. The
wave functions of a meson change appreciably inside the nucleus; therefore, the quantity
| ’'j iL
' can be replaced by its value at the origin only in a comparatively crude qualita
tive estim ate, as in the derivation of (21.14).
Answer. In the general case
For sfnall b and ,U — oo, this expression may be obtained from Eq. (21.12).
(Hiapter 22
The Dirac theory, which includes spin effects and relativistic effects, was able to
account for the fine structure of the spectral lines of hydrogen-like atoms and the
anomalous and normal Zeeman effects. The Dirac theory, however, also gave rise to
a number of major difficulties in connection with the interpretation of negative energy
states. These difficulties were not overcome for some time, but they eventually led to
fundamental new discoveries in relativistic quantum mechanics.
In our treatment of the motion of a free particle (Chapter 17), we mentioned that the
Dirac equation allowed solutions corresponding to both positive and negative values
of energy. It is worth noting in this connection that solutions with negative energy are
not characteristic of the Dirac theory alone, but appear in any relativistic theory, in
relativistic mechanics, the energy of a free particle is connected with the momentum
and rest mass by the well-known expression
E- = c-p- + m-uc\
which has two roots
E = i t Yc-p- + /njc1.
The regions of positive and negative energies are separated by an interval equal to
2moc2 (see Fig. 22.1). At first glance, states with negative energy do not appear
to have a real physical meaning, since the region of negative energies extends to Infinity
( E — — oo) and, therefore, there is no lowest
state. This would imply that no ordinary state
is stable, since a spontaneous transition to a
lower energy state would always be possible.
Furthermore, a particle with negative mass
(negative energy) would have a number of
strange properties: for example, it would repel
a particle with positive mass. -E=0
In classical physics, states with negative
energy do not cause any difficulties, because
the energy of a moving particle can only change
continuously; therefore, transitions from the
states with positive energy to the states with
negative energy cannot take place, since the
energy would have to change discontinuously by F ig . 22.1. A llo w e d e n e rg y le v e ls of a
the amount &E 5 : 2 m0c2. Defining the energy free D irac p a rtic le .
to be positive from the start, we may, therefore,
neglect the negative energy states.
The situation is quite different in quantum theory, where transitions can take place
between the states in a discrete spectrum, as well as in a continuous spectrum. States
with negative energy cannot be excluded simply by defining the energy to be positive
348 R E L A T IV IS T IC QUANTUM M EC H A N IC S
at the initial time, because the probability of the transition between the states with
energy + m 0c2 and — m0c- is not equal to zero.
In order to avoid transitions of electrons to negative states, Dirac suggested (1931)
that we regard all negative energy levels as occupied by electrons (see Fig. 22.2),
so that an electron with a positive energy cannot
• Jump into a negative energy level under ordinary
---------------------------------------conditions. 1 The state In which all negative
• ~~' — energy levels and no positive energy levels are
occupied is called the "electron vacuum.”
Let us now assume that a 7-ray photon with
energy t i> 2 m0c2 excites an electron from the
electron vacuum into a positive energy state. In
this case, the absorbed 7-ray photon will be
replaced by an electron with positive energy, and
a "hole” will appear In one of the negative energy
states (see Fig. 22.3).
The decisive factor that led to the success of
F ig. 22.2. Z e ro -p o in t e n erg y d ia Dirac’s hypothesis was that he interpreted the
gram of the e lectro n -p o sitro n "hole” as a particle (the "positron” ) with posi
vacuum .
tive mass equal to the mass of the electron, but
with the opposite charge. 2 Let us suppose that
there are no particles at the initial time. Then
the "zero-point energy” EVac (the energy of the electron vacuum) is equal to the sum of
the energies of the electrons In the negative energy states /i_
E —V E ( 22. 1)
c vac — Z . n
eCvac v e0.
Thus, from the standpoint of the hole theory, the absence of any real particles means
that all positive energy states are empty, and all negative energy states are occupied.
This case corresponds to the electron vacuum (see Fig. 22.2).
When an electron jumps from a negative energy state n_ to a positive energy state n+ ,
the total energy change of the system is
AU= E’\ + 1n’' £»1- 2 £»L= E"+- E»_ = En++ IEnJ . (22.2)
n'
This change represents the sum of the positive energies of two nascent particles .3 Similar
arguments with regard to charge show that the charge of the nascent particle correspond
ing to the "hole” is opposite to the electron charge
*In a c c o r d a n c e w i t h t h e P a u l i e x c l u s i o n p r i n c i p l e ( s e e C h a p t e r 6), o n l y o n e e l e c t r o n
can occupy each stale.
A s i m i l a r c o n c l u s i o n w a s r e a c h e d in Ih o t r e a t m e n t o f t h e h o l e t h c o r y o f s e m i c o n d u c t o r s
( s e e t h e d i s c u s s i o n o f th e b a n d t h e o r y o f c o n d u c t i o n in C h a p t e r 6).
* T h e p r i m e a t t a c h e d to t h e s u m m a t i o n sip.n (?. ) m e a n s t h a t t h e s u m m a t i o n e x t e n d s o v e r
all s t a t e s e x c e rp t t h e s t a t e n ‘_ .
ELECTRON POSITRON AND ELECTROMAGNETIC VACUUMS 349
Ihus, the transition of an electron from a negative energy state to a positive energy
state (as a result of the absorption of a 7-ray quantum with energy greater than 2 muc'J)
leads to the creation of a pair of particles. The unoccupied negative energy state ("hole")
may be regarded as a state occupied by a particle with positive charge + e 0 and positive
energy.4 This particle, which was predicted by
Dirac, was called a "positron” and was dis
covered by Anderson in cosmic radiation (1932).
Once It is Interpreted In this way, the Dirac
theory describes In a natural manner both the mnc ‘
electron and the positron. The positron Is an
antiparticle; Its wave function satisfies the Dirac
equation with positive energy and positive charge
(see Problem 18.5). -mr
In the Dirac theory, pair annihilation — a
process which Is the reverse of pair creation—is
also allowed. This process takes place when an
electron with positive energy Jumps Into a hole.
In this case the electron and positron are con F ig . 22.3. F o rm a tio n o f a n e le c tr o n -
verted Into 7-rays. p o s itr o n pair.
In these transformations, the laws of conser
vation of energy and momentum are rigorously
obeyed. As already mentioned, pair creation due to the absorption of a 7-ray photon can
occur only In the presence of a third particle (for example, a nucleus), that takes up the
excess momentum of the photon
In order to show this, we may choose, without the loss of generality, a coordinate system
In which the electron and positron move with opposite momenta, so that = — k+ = k
(the center-of-mass system). Then, according to the law of conservation of momentum,
the total momentum of the two photons which are formed as a result of annihilation must
also equal zero
n ( k l + k») = 0. (22.5a)
The lowest value of the photon energy is obtained when k = 0 (that Is, when the electron
and positron are at rest). Then ( mjn = moc'-. These two 7-ray photons move apart with
the same energy and oppositely directed momenta. It is easy to see that the electron-
positron pair cannot be converted Into a single 7-ray photon ( k 2 = 0), since the laws of
conservation of energy and momentum cannot be satisfied simultaneously with only one
7 -ray photon.
4
With t h e h e l p o f q u a n t u m f i e l d t h e o r y , w e c a n c o n s t r u c t a t h e o r y o f t h e e l e c t r o n -
p o s itro n vacu u m th a t is s y m m e tric w ith r e s p e c t to c h arg e. H o w ev e r, ev en w ith th e theo ry
d e s c r i b e d a b o v e , w h i c h i s a s y m m e t r i c w i t h r e s p e c t to e l e c t r o n s a n d p o s i t r o n s ( a n e l e c t r o n
i s a p a r t i c l e , w h e r e a s a p o s i t r o n i s a “h o l e ” ) w e c a n g i v e a c l e a r e x p l a n a t i o n o f m a n y
pheno m en a in v o lv in g th e tra n sfo rm a tio n of p a r tic le s .
350 R E L A T IV IS T IC QUANTUM M EC H A N ICS
The law of conservation of total angular momentum (orbital plus spin) is also very
Important in the annihilation processes. If an electron moves with a nonrelativlstlc
velocity, this law (as shown in Chapter 18) can be resolved into a law of conservation of
orbital angular momentum and a law of conservation of spin.
The law of conservation of spin can be observed particularly clearly in the annihila
tion of positronium (a hydrogen atom in which the proton is replaced by a positron, or
more exactly, a system in which an electron and a positron rotate about their common
center of mass). In this atom, the nucleus (that is, the positron) and the electron have the
same m ass, and therefore the reduced mass Is The energy of levels In the positron
ium atom will, therefore, be one half as large as in the hydrogen atom
= _M _
2n1 '
while the radius of the orbit will be twice as large. The velocities of the electron and
positron may be regarded as nonrelativistic. Just as in the hydrogen atom. If the spins
of the electron and positron are antiparallel (paraposltronlum), positronium can decay
into two 7-ray photons (the corresponding mean life Is 1.25 • 10~10 sec).
The total spin of parapositronium Is equal to zero, and therefore the two photons move
apart with opposite directions of spin (that is, their total spin Is zero).
If, however, the spins of the electron and positron are parallel (orthoposltronlum) the
system must decay Into three 7-ray photons (the corresponding mean life is 1.4 • 10- 7sec).
The spin of orthopositronium is equal to unity. Orthopositronium cannot decay into one
7-ray photon with spin 1, because in this case the law of conservation of momentum
would be violated. It cannot decay into two 7 -ra y photons because then the law of con
servation of spin would be violated (the total spin of two 7-ray photons is either two or
zero). Only if orthopositronium decays into three 7 -ray photons will the law of con
servation of momentum and the law of conservation of spin be satisfied.
The discovery of the positron opened a new stage In the study of elementary particles.
This discovery showed that particle shad a new fundamental property—lnterconvertlblllty—
and confirmed the existence of antiparticles. We can regard the creation of a positron
as the conversion of a 7-ray photon into an electron-positron pair, and the annihilation
of an electron-positron pair as the conversion of an electron-positron pair into 7-ray
photons.
( 22 . 6 )
ELECTRON-POSITRON AND ELECTROMAGNETIC VACUUMS 35
Since the rest mass of a "pseudo-photon” Is zero, we may write the relation between Its
frequency <*> and the wave number k as follows:
“ = -«- = I T = e 1/ * T F * j T * * . (22.7)
The components kx , ky, k, are related to the Integers n,, n2, n, and the period I. (see
Chapter 6) by the expressions
kx = - — - , and so forth,
so that
Skx Sky Sh2 = k 2 dkdQ,
m2dus 8n3
—3
c3— dQ= ~rr
L■
’
If the system Is spherically symmetric, this relation may be written as
o- da* I (22. 8)
L1 “ca—'2nr
With the aid of Eq. (22.6), we find that the energy of the electrostatic field Inside a region
of volume V is
(cos U c o s u '0 a v =
Just as In the theory of the harmonic oscillator, the energy of the field In the lowest
energy state is not equal to zero, but instead is equal to the sum of the zero-point
energies of the harmonic oscillator (see Chapter 10)
S« = 2y { s"- (22.10)
(1)
The coefficient 2 In front of the summation sign takes into account the fact that each
harmonic corresponds to two different states of polarization of the "pseudo-photons.”
In the state where there are no real photons the total energy of an electromagnetic
field (the vacuum), must be equal to this zero-point energy (e p^p# = e0). Hence, taking
(22. 8) into account, we find
( £ (» ) )* = 71“ = - ^ - ~ P - . (2 2 .10a)
In the absence of real particles and external fields, the vacuum (including the electro
magnetic one containing photons and pseudo-photons) does not, as a rule, have any
observable effects because it is isotropic. On the other hand, when real particles
352 R E L A T IV IS T IC QUANTUM M EC H A N IC S
and external fields are present, the Isotropy Is disrupted and virtual particles (“pseudo
photons” or electron-positron pairs) are created and subsequently annihilated (vacuum
fluctuations).
A simple physical picture of a few basic notions Involved In the theory of the vacuum
was obtained somewhat unrlgorously by Welton by means of a sem iclassical, nonrelatlvlstlc
treatment of the motion of an electron, taking Into account Its Interaction with the vacuum
fluctuations. As an Illustration of Welton’s calculations, let us attempt to give a more
concrete meaning to the zero-point fluctuations (22.10X In a rough approximation we
can use the ordinary classical equation
/n0r = ££p>p> (22.11)
to find how an electron will be affected by the vacuum field of "pseudo-photons." Ex
panding £_ _ in a Fourier series
P* P*
(22-13)
<D
The mean-square deviation of the position is given by
(17F = ^ - ( r ) ’ = ^ 2 i ^ ^ , (22.14)
U)
since
from which a finite (observable) part can be separated out If the range of variation of the
frequency is cut off from above at a frequency corresponding to the rest energy of the
electron
(l)max —
m0c2 (22.16)
ft ’
and from below at a frequency corresponding to the minimum energy of the electron In
an atom 5
2 _h_
(22.18)
(ir)2 —a Mn
»v.
’ T h e l i m i t s o f t h e r a n p e o f v a r i a t i o n o f fv a r e s p e c i f i e d m o r e a c c u r a t e l y in r e n o r m a l i
z a tio n theory.
ELECTRON PO SITR O N AND E L EC TR O M A G N E TIC VACUUMS 353
Thus it can be seen that the vacuum field of "pseudo-photons" will cause the electron
to perform a motion somewhat resembling Brownian motion with a definite value of the
mean square displacement. It is well known that the Brownian motion of a particle Is due
to Its collisions with randomly moving molecules of the surrounding medium. In similar
fashion, an electron undergoes "collisions” (of a special kind) with the assembly of
virtual particles forming the vacuum.
As was shown by N. Bogolyubov and S. Tyablikov, the vacuum fluctuations cause a
certain "smearing out” of a point electron. As a result, the electronic radius turns out
to be the geometric mean of the classical radius and the Compton wavelength
fT a ~ (22.19)
r «ioc »i0c muc
The existence of this effective radius should have several consequences. In particular,
the interaction between the electron and the nuclear charge should be changed, and this,
in turn, should lead to an additional coupling energy and thus to a shift of the energy
levels. The usual expression for the potential energy of an electron In the field of a
nucleus
V = — e0‘I>(r) (22.20)
Is replaced by
The additional energy of interaction between the electron and the nucleus Is given by
To obtain an expression for the shift of energy levels In the hydrogen atom. It Is necessary
to average the additional Interaction energy over the corresponding state
This shift occurs only for s states, since the quantity | ^ (0) |2In the approximation under
consideration vanishes for other states ( / = 1, 2, 3 , . . . ) , whereas for the s state
This Is in fairly good agreement with recent experimental data (A E = 1057.77 Me) for
the Lamb shift (see Chapter 20),
A complete study of the shift of energy levels of atomic electrons with the help of
relativistic quantum field theory gives considerably better agreement between theoretical
and experimental results than the sem iclassical formula (22.28). The discrepancy is
reduced to le ss than 1 Me.
For the sake of brevity, we shall not discuss the modern theory of the vacuum In
greater detail, and we shall confine ourselves to a mere enumeration of the main results
that this theory has yielded.
C. ELECTRON-POSITRON VACUUM
Equation (22.28) for the Lamb shift was obtained from a calculation of the Interaction
between electrons and the electromagnetic vacuum. In addition to the electromagnetic
vacuum,. there exists an electron-positron vacuum and also vacuum states of other
particles. The method of second quantization (which is In some measure applicable to
all fields) can be used to calculate the influence of the electron-positron vacuum.
In modern quantum field theory, the study of the properties of the vacuum states of
different particles plays a particularly important role. The vacuum gives rise to inter
actions between particles. In particular, the electromagnetic Interaction of two electrons
(Coulomb’s law) may be regarded as an interaction which takes place through the electro
magnetic vacuum, with one electron emitting a virtual photon and the other electron
absorbing it.
On the other hand, the vacuum represents a sort of "reservoir,” from which real
particles are "drawn” when they are created, and to which they "return" when they are
annihilated. We have already come across the electron-positron vacuum as a "collection”
of electrons in negative energy states. Unfortunately, it has no classical analog and
therefore we cannot use a sem iclassical analysis, as was possible in the case of the
electromagnetic vacuum. The Coulomb field of a nucleus can polarize the electron-
positron vacuum (so that an electron will behave as if it were in a dielectric), giving rise
to an additional interaction
<2 2 - 3 0 >
Comparing (22.30) with (22.24), we see that the level shift due to Interaction with the
electron-positron vacuum is about 1/40 times as large as the level shift due to the fluc
tuations of the electromagnetic field, and Is opposite in sign.
The electron-positron vacuum exerts a particularly strong influence on the magnetic
properties of the electron. It was shown by Schwinger that the magnetic moment of the
electron becomes somewhat larger than the Bohr magnetron
Fe.-p.= (22.31)
The change in the magnetic moment of the electron, with the second-order term taken
Into account.
is in good agreement with experimental data obtained with the aid of microwave spectro
scopic methods (see Chapter 21). Qualitatively, the appearance of the additional magnetic
moment of the electron and the sign of the moment can be explained as follows. The initial
electron A (see Fig. 22.4), whose spin is directed upwards (we specify in this manner a
preferred direction), creates a virtual pair—electron /!1 and positron fl1—which generally
E L E C T R O N -P O S IT R O N AND EL E C T R O M A G N E T IC VACUUMS 355
have opposite spin directions. Since we have a preferred direction (determined by the
spin of electron A) the spins of electron A' and positron B‘ can be directed in two ways:
either the spin of A will be antiparallel to the spin of /V and parallel to the spin of B'
(case 1 in Fig. 22.4), or the spin of A will be parallel to the spin of /V and antiparallel
to the spin of B' (case 2 in Fig. 22.4).
In determining the additional magnetic mo
ment, it Is necessary to consider the following
possibilities of pair creation and annihilation: bY a)
(a) Palr/VB 1 is created and then annihilated.
This is possible in case 1 and case 2. No cor
rection to the magnetic moment should arise,
because the probability of the creation process
is the same in both cases, and the magnetic
moments have opposite signs in the two cases.
(b) Since ,1 and B' have opposite spins in
case 2, positron B' may be annihilated together
with A ', as well as with the initial electron A.
In case 1, however, this process is less prob >1 if-
able, since the spins of pair A and B' are
parallel to one another. This process gives a A n t i p a r a l l e l Parallel
preferred state in which the additional magnetic s p in s of spins of
moment is parallel to the Initial magnetic mo electro n s electrons
ment of electron A, since, as can be easily A and A1 A and A1
seen, the magnetic moment of all three particles F i g . 2 2 . 1. V i r t u a l c r e a t i o n o f a n e l e c
(A, A' and B') is directed downwards. Conse tr o n - p o s itro n p a i r by a n e le c tr o n . T h e
quently, there arises an additional magnetic s p in s of th e real a n d v irtu al p a r tic le s
moment of the electron, equal to
a r e d e p i c t e d by s o l i d a n d d a s h e d a r
^ e .-p . =—2fM. row s, r e s p e c tiv e ly .
where f Is a numerical factor which determines the probability that the initial electron
A will be annihilated together with positron B'.
These simple considerations give the correct sign of this anomalous magnetic mo
ment. More rigorous calculations led to Eq. (22.32).
In addition to the Dirac magnetic moment, u = — y 0o , an electron will have an
a ^
anomalous magnetic moment |ian = — ,-j- ^oP«®, which arises owing to the interaction
between the electron and the electron-positron vacuum.
The anomalous magnetic moment of nucleons (neutrons and protons) can be explained
in meson theory. The accuracy of the results, however, is much le ss striking than in
calculations of the anomalous magnetic moment of the electron. This Is due to the fact
that meson theory is still in a much less satisfactory state than quantum electrodynamics.
According to the meson theory, protons and neutrons interact with the 7r-meson field.
Since the proton can dissociate into a neutron and a positively charged ir meson, and the
neutron Into a proton and a negatively charged ir meson, the anomalous magnetic moments
of protons and neutrons due to the ir-meson field must be approximately equal in magni
tude and opposite in sign.
The appearance of the additional magnetic moment of a neutron is explained as follows.
The neutron has a definite probability of dissociating into a proton and a ir meson. Since
the spin of a tt meson equalsO.it does not possess an intrinsic magnetic moment. There
fore, there will be a contribution to the magnetic moment of the neutron from a plon
which is, for example. In a p state (we recall that the orbital angular momentum in the
s state is also equal to 0). The magnetic moment of a ir meson is —£ «= 7 times greater
than the nuclear magneton, and therefore the tt meson produces the main contribution to
the anomalous magnetic moment of the neutron. For the virtual process to fulfill the
law of conservation of angular momentum, it is necessary that the direction of the
orbital angular momentum (equal to 1) of the virtual ir meson should coincide with the
direction of spin of the neutron, while the spin of the virtual proton (equal to *,», just
like the spin of the neutron) should be directed opposite to the spin of the neutron so that
s„ = s„ — sp, s„ = Sp = 4j-, s „ = l . Since the ir meson has a negative charge and its
angular momentum is parallel to the spin of the neutron, it gives rise to the negative
magnetic moment of the neutron.
356 R E L A T IV IS T IC QUANTUM M EC H A N IC S
In order to explain the magnitude of the anomalous magnetic moment of the neutron,
one must assume that the neutron spends ‘A of Its time in Its dissociated state. This
estimate is perfectly reasonable and gives the correct sign and order of magnitude of the
magnetic moment of the neutron(n®n ~ — 2t± nuc). Similarly, it can be shown that the dis
sociation of the proton into a neutron and a ji+ meson gives rise to a positive anomalous
magnetic moment of the proton (|J-pn~ + 2m-nuc)- Moreover, the proton has a Dirac mag
netic moment (|J-jp= p- nuc)- More accurate data on the magnitude of the magnetic moment
of the proton and neutron are given in the preceding chapter.
Thus we have a basis for regarding the anomalous magnetic moment of Dirac particles
as a secondary effect which can be explained on the basis of the field theory. This
moment does not appear in the initial equations, but arises as a result of the interaction
either between the electric charge and the electron-positron field (in the case of electrons),
or between the nuclear charge and the 7r-meson field (in the case of protons and neutrons).
D. RENORMALIZATION
One of the most important sections of modern quantum field theory is concerned with
the question of renormalization. This subject is not yet in a mathematically satisfactory
state, but a number of important results have been obtained.
The basic idea involved in renormalization is the separation of finite, observable
terms from the divergent terms describing the interaction between an electron and the
electromagnetic or electron-positron vacuum.
In effect, the question of renormalization first arose in classical electrodynamics—
for example, in the theory of the electromagnetic mass of the electron. If we assume
that the electron charge is distributed inside a sphere of radius r0, its classical electro
magnetic m ass will be equal to
field — y e‘ (22.33)
~ Tr0c2’
where 7 is a factor of the order of unity which depends on the charge distribution inside
the sphere. The attempts to construct a classical electrodynamics in which a finite
radius r 0~ 10“13 cm would give a reasonable value for mfield (the Lorentz theory, the
nonlinear Bom-Infeld theory, the Boppe-Podolsky theory with higher derivatives) did not
give any satisfactory results. Moreover, all these theories gave rise to fundamental
difficulties in connection with quantization.
On the other hand, the theory of a point electron leads to an infinite value of the mass
as r0 — 0 both in the classical and quantum forms of the theory. It was therefore a major
achievement of renormalization in modern quantum field theory that it was able to
separate, from the Infinite Interaction energy, finite terms associated with the Lamb
shift of the atomic levels and the additional magnetic moment of the electron.
Modern renormalization theory extends to the problem of the self-m ass and charge
of the electron. It has been found that when the electron-positron vacuum and the electro
magnetic vacuum are taken Into account, the electromagnetic m ass (22.33) drops out
completely. Therefore, the main mass of the electron (the so-called “bare” mass) will
not be associated with the electromagnetic field. The Interaction with the vacuum (in this
case the main contribution is obtained when the electromagnetic vacuum and the electron-
positron vacuum are taken into account simultaneously) yields an additional m ass which
diverges only logarithmically
. field .
Am — am In ‘max (22.34)
bare m 0c-
Renormalization is faced with the important question of finding the value c max at
whLh the logarithmically divergent expression should be cut off. This problem Is a
theoretically important one, although in practice the logarithmically divergent terms may
be regarded to be of the order of unity, and therefore the quantity Am will remain of the
order m bare /137 over a comparatively wide range of values e max.
ELECTRON PO SITR O N AND E L E C T R O M A G N E T IC VACUUMS 357
In exactly the same way, the Interaction of the electron with the electron-positron
vacuum (rather than with the electromagnetic vacuum) leads to a decrease of Its "bare
charge” by the amount
p = p ’ + tik
be satisfied, where p and p' are the initial and final moments of the electron and tik is
the photon momentum.
Squaring these relations and then subtracting one from the other, we obtain the follow
ing expression for the cosine of the angle of photon emission:
C0S°
= ^ + f ['“ A }
Using the relation for the index of refraction n of the medium
e = Hoi = c'hk = —
a 1 , nk(, 1\
COSe=F + ^ ( 1 - ^)-
Radiation can take place when p« > 1 (that is, when the electron velocity v remains less
than the phase velocity of light in vacuum, but becomes greater than the velocity of light
in the medium c > v' > c'). In the classical case (ti = 0), the angle at which the photon is
emitted satisfies the relation
COSt) = S
s-/t .
H; = T, + VV T, = = (23.2)
and the subscript j takes two values: / = 1 in the case of the first
electron, and / = 2 in the case of the second electron. From these
equations we can obtain the energy values Enj (see Chapter 13)
E = — m r' (23.3)
ni n-j
*We a r e a l l o w e d t o d o t h i s b e c a u s e in t h i s a p p r o x i m a t i o n t h e p r o b l e m h a s a s o l u t i o n
in t h e fo rm o f a p r o d u c t o f tw o f u n c t i o n s , d e p e n d i n g o n t h e s p a t i a l a n d s p i n c o o r d i n a t e s ,
r e s p e c t i v e l y ( s e e C h a p t e r 2 4 ).
360 R E L A T IV IS T IC QUANTUM M EC H A N IC S
« = M n ) <!>»,(/•*)■ (23.i°)
Substituting (23.10) into (23.9) and using (23.1), we get
(23.11)
where Eni and En., are the energies of the two electrons on the as
sumption of no mutual interaction. This result can be explained as
follows. In the absence of the perturbation V', the motion of the
electrons is determined by their interaction with the nucleus Ze„;
that is, their motion is completely described by the Schrodinger
equation (23.1) which has the eigenvalues En. [see (23.3)] and
eigenfunctions •}‘nj. Since one of the electrons is in a state ih and
the second in a state n4, the total energy of the system is E„, -f- En,
when V' = 0.
Since the two electrons move independently of one another, the
total wave function will be the product of the corresponding two
independent one-electron wave functions obtained in the one-
electron problem. By direct substitution into Eq. (23.9) it is,
however, easy to show that, in addition to the first solution (23.10),
there will be a second solution (<]>0 = y) corresponding to the same
energy eigenvalue (23.11):
V= tym(f“i) 'i'ni (^l)‘ (23.12)
This solution differs from u by a permutation of the electrons. The
first electron is now in state n4 and the second electron is in
state «i.
This state of the system has, therefore, an additional degeneracy
which is due entirely to the indistinguishability of the electrons;
this is known as exchange degeneracy.
If both electrons are in identical states (n, = «.,), the wave
functions u and v are identical and there will be no exchange de
generacy since
w^ u= (]<ni (/*i) <]<n. (/"c)). (23.12a)
In order to find the values of the coefficients Cj and C.2 and the
energy E of the perturbed system (the system with V' taken into
362 RELATIVISTIC QUANTUM MECHANICS
£ = £° + £';
^= t/>-{-f . (23.14)
In (23.17), let us substitute r2 for rx and /”i for r2. The function
v [see (23.12)] then becomes u [see (23.10)], and the function v
becomes u , while the perturbation energy V' remains unchanged
since \r2— r2\ = \r2— r x\. Thus Eq. (23.17) takes the form
(23.18)
pn (/-,) and p.i (/".,) characterize the so-called density of mLxed (or
exchange) states , 3 when each electron is partially in state n, and
partially in state nt.
The orthonormality condition gives
and
T h e s e d e n s i t i e s h a v e no c l a s s i c a l a na l og.
364 RELATIVISTIC QUANTUM MECHANICS
^ (l>*stysd:'xid:ix2= ^ (})*a(J)ac(3.K:1d3 X2 = 1 .
(23.31a)
When both electrons are in the same quantum state (nl = n.l ),
the functions u and v are identical. In this case Eqs. (23.16) and
(23.17) reduce to the same equation
^ u* (E'— V ^ u d ^ c P x ^ 0. (23.33)
E' = K, (23.34)
4 Wc r e c a l l t h a t u n d e r a p e r mu t a t i o n o f c o o r d i n a t e s ( t hat i s, wh e n a n d T2 a r e i n t e r
c h a n g e d ) t he f un c t i o n u a n d u t r a n s f o r m i nt o o n e anot her . T h e w a v e f un c t i o n d o e s not
c h a n g e s i g n a s a r e s u l t of t h i s o p e r a t i o n ( s y m m e t r i c f unc t i on) , w h e r e a s t he f unc t i on 0
r e v e r s e s i t s s i g n ( a n t i s y m m e t r i c f unct i on) .
THEORY OF THE HELIUM ATOM NEGLECTING SPIN STATES 365
and thus the exchange energy disappears in th iscase. For the wave
function we obtain a unique symmetric solution
$* = ,, = 1^ (23.35)
Es = E°-\-K. (23.36)
Z*el , , v 1 (Z \ 3 / 3 e- fa»
1 (23.37)
Ex *l(r>= 7 rU r)
ft2
where °o— is the radius of the first Bohr orbit.
The Coulomb energy of interaction between the two electrons is
Here |rj — ri \= Vr\-\-r\ — 2 r2 cos t), and & is the angle between the
vectors and r To integrate (23.38), let us choose the direction
of the z axis along the direction of the vector rim Substituting the
expression (23.37) for the wave functions into Eq. (23.38) and in
tegrating over the angles, we find 6
T h i s i s d u e to t h e f a c t t h a t the p e r t u r b a t i o n r e m o v e s t h e d e g e n e r a c y , a n d t h e r e f o r e
th e c o e f f i c i e n t s th a t w e r e i n d e t e r m i n a t e in t h e z e r o t h a p p r o x i m a t io n c a n now t a k e s p e c i f i c
v a l u e s . ( S e e a l s o C h a p t e r 14, t h e S tark e f f e c t . )
6 In i n t e g r a t i n g o v e r th e a n g l e t? (x c o s i9), w e u s e d th e r e l a t i o n
2
for r j < r 2 ,
d x _______ >~2
I
v ri > i - 2 ri'Y: —2 , r , >^ r2 .
for
(continued)
366 RELATIVISTIC QUANTUM MECHANICS
£o = 2£, = — (23.40)
a0 ’
the total energy of two electrons in the ground state is equal to
£ = P + i( = - ^ i - f | z J (23.41)
it
Let us now find the ionization energy of the helium atom, that
is, the energy that must be expended in order to remove one
electron from the first orbit. For a singly ionized helium atom
(a hydrogen-like atom) the bonding energy between the electron and
the nucleus is simply £, [see (23.37)]. The ionization energy of a
helium-like atom is therefore
£ ‘ioiw ,£1_ £ = ^ -J z 2 — - Z ) , (23.42)
for r» < r 2 .
r2
0 for r| > r 2 •
T h i s e x p r e s s i o n w a s u s e d in e v a l u a t i n g t h e i n t e g r a l ( 2 3 . 3 8 ) .
T HEORY OF T HE HELIUM AT O M NEGLECTING SPIN STATES 367
E= (23.44)
* = .£Qpn, (23.44a)
£ = 2 l C"l2£«- (23.45)
n
Replacing each eigenvalue En in the summation by the lowest
eigenvalue £ min and using the fact that for normalized functions
2 |C J 2 = 1 ,
n
we find that
£ min sg ^ (23.46)
that is, the lowest value of the integral ^ ty*K''?d3x can be used to
determine the upper limit of the ground-state energy of the system.
It was found that the variational method gives very good results
when the perturbation energy £' is of the same order as the energy
£ 0 of the zero-order approximation. The variational method can
therefore be used in cases where perturbation theory gives poor
results (for example, in calculation of the ground-state energy of
the helium atom). When a problem is solved by the variational
method, both the additional interaction V’ and the main interaction
are treated equally in the Hamiltonian H of Eq. (23.7). A test
function <Ji then depends on several parameters and is selected
in such a way that the integral can be calculated exactly. The
energy £ will then be a function of these parameters and it is
obvious that the minimum value of this function will be close to
the true value if the test function resem bles the true function.
368 RELATIVISTIC QUANTUM MECHANICS
The most difficult part of the method lies in the choice of the
best test function. All the available information on the properties
of the system must be used in making this choice. It is impossible
in the general case to indicate the form of the test function, and
therefore it is frequently necessary to rely on physical and
mathematical intuition. Very often, a test function is sim ilar in
form to the solution of the unperturbed equation.
We shall now use the variational method to calculate the ground-
state energy of the helium atom. Our procedure will be based on
that of Hylleraas (1927). At the end of the discussion we shall
compare both perturbation and variational methods.
For the test function, Hylleraas chose the ground-state function
(23.27) of the hydrogen atom, replacing the charge Z by a certain
effective charge Z'. The quantity Z is the unknown parameter which
has to be determined from the variational principle. The test
function
<23-4?>
H = T1 + l/ 1 + T2 + F 2 + F', (23.48)
where
(23.51)
8 a0 ’
T\ = — El = ^ ^ - . (23.52)
7, = -£ 2 £ 1 = — a0 (23. 53)
E (Z') = O
—q
(Z,a — 2ZZ' —
o
Z'), (23.54)
£ min= _ ( Z _ 5 ) * l . (23.55)
el (23.56)
00
370 RELATIVISTIC QUANTUM MECHANICS
£ = 2 En\Cn\\ (23.57)
n=n0
We shall consider one of the most general forms of the variational problem, when
the choice of the test wave function 4' that is used to find the average value of the
Hamiltonian
E= (23.58)
describing the motion of a single particle is restricted only by the normalization con
dition
^ d3x = 1. (23.59)
Upon varying E with respect to ^ and using the hermitlcity of the operator H, we ob
tain
Here the variations H and cannot be regarded as Independent since they are con
nected by the normalization condition (23.59). These variations can be made Independent
by varying the condition (23.59)
^ ^ d‘x = 0.
7T h i s i n t e g r a l c a n b e g e n e r a l i z e d in a s i mi l a r f a s h i o n to t he c a s e of t h r e e or more
p ar ti cl es.
372 RELATIVISTIC QUANTUM MECHANICS
$ (23.64)
(i J I, + 9 .H 4 A + J 9? — 9 - e) 9i= 0,
(23.66)
( h 4 4~ ^ 'PiHrPi^i “I- ^ 9? 9irf,1*i — 9-2= 0-
C = V j- 9 ;iI/P/i%. + i y V 5 9W ^ - 9 ,9 (23.66a)
i
THEORY OF THE HELIUM ATOM NEGLECTING SPIN STATES 373
and
E j = J
(H; Ej) = 0
where the plus sign corresponds to <]js , and the minus sign to i|»a.
In order to analyze the exchange energy in greater detail, we shall
examine the behavior of the system in time, taking into account the
exchange energy. The wave functions of the symmetric and anti
symmetric states may be written as
(23.70)
that is.
if (l) = e~‘wC{u cos ot — iii sin o^} = e" 1*' {Cuu -|- Cuv], (23.74)
where
~ (23.77)
1 Z_ 3/2 .
^ (r) I e (23.79)
2 f/2n (to
Carrying out the integration in the same way as was done for the
evaluation of energy, we obtain, instead of (23.38a),
00
A. 2 *Zel (23.81)
3°a0
3
0 .8 - 1 0 ' 1 5 s e c .
w
The exchange time in the case where one of the electrons is in
the Is state and the other in the 1 0 s state is of the order of several
years, and there is practically no exchange. Consequently, it can
be seen that the exchange energy plays a significant part only
when the probability densities | ^ n ; | 2 of different states mutually
overlap to a considerable extent. When the overlapping of the
wave functions is insignificant, the exchange energy A is very
small. This situation somewhat resem bles the transfer of energy
from one coupled pendulum to another. It is well known that an
oscillating pendulum transfers its energy to another pendulum,
X= =bl. (24.6)
This result means that interchanging the particles leaves the wave
function unchanged (symmetric wave functions, with k = 1 )
(that is , particles with coordinates /"i and r* are in the states n2 s.2
and 'h^i , respectively).
Both wave functions (24.9) and (24.10) describe the same
physical state of the system . We note that any linear combination
of these functions will also describe the same physical state. One
of the possible linear combinations is the sum of the solutions
(24.9) and (24.10), namely,
From the form of lirs and T;‘ it follows directly that particles
obeying Fermi-Dirac statistics can be described only by anti
symmetric solutions. The Pauli exclusion principle (Fermi sta
tistics) is satisfied only by a solution of the form (24.13) since
when both particles are in the same state (that is, ni — tii — n and
s1 = s.i = s), we have
There are two different ways in which these four quantities can be
added. One of these involves adding the orbital and spin angular
momenta separately and then finding their total sum
L = L1 + L, 1 S = S, + S2) J = L + S. (24.21)
£s.o. _ L. S (24.24)
2 m ic* rJ
and of the operator for the square of the total angular momentum
Lsy l m (1>2) = {Lf + L? + (Li* — 'Liy) (L,x + lLly) + (Lljc iL^) (L2jf— iL2>1) +
+ 2 L „u ,} YLMo.2) = n n y LM(),2y, \ = L ( L + d (24.26)
As we shall show, the eigenfunction ylm 0 ,2) can be written as a linear combination of
products
where the wave functions K™1 (1) and Y™2 (2) refer to the first and second particles,
respectively, and
L* Y lm (1,2) = 2 + = (24.29)
77ij772<$
we find that M = fll| + '"sand = bm„.
Without loss of generality we may set 711= 0; that is, we make the z axis perpendic
ular to the total angular momentum. Then it follows from (24.27) that
K£0(l,2) = b ^ Y \ (1) Lr‘ (2) + M V (1) Y\ (2) + (1) K° (2). (24.30)
Taking into account the equation for spherical harmonics [see (11.88)]
This equation is equivalent to a system of three homogeneous equations for the unknown
Clebsch-Gordan coefficients
2f>0 + (2 — X) b, =0
2b0 + (2 — X) b_i = 0 (24.33)
(4 — X) bo + 2 bt +26_,=0.
384 RELATIVISTIC QUANTUM MECHANICS
Calculating this determinant and subjecting the wave function YLM to the normalization
condition
J rfas = i,
that is.
bi + b\ + bU = \,
1) X= 0, = = -$„= — •
V*
From the conditions X= L (L + 1) we find that L = 0; that Is, this solution corresponds to
the case in which the vectors A and fa are antiparallel
A f J A.
2) X= 2, bt = 0, - b l = b_l = y = , L = \:
This case corresponds to the addition of the angular momentum vectors at an angle
of 60°
A f t l».
We conclude, therefore, that two vectors A and A , whose absolute values are Integral
quantum numbers, can be oriented at angles such that the vector representing their sum
L is also characterized by integers. For A 55 l3 these values are
L = A + A, A + A — 1........A — A + 1, A — A (24.35)
(a total of 2A + 1 values).
This method of adding the angular momenta is based on a vector model of the atom.
We can obtain our previous results by setting A =A = 1 andM = 0. It is interesting to
note that the same values of the Clebsch-Gordan coefficients would also be obtained in
adding the orbital angular momentum to a spin angular momentum equal to unity (for
example, for photons). In this case, however, one would have to use the normalized spin
THEORY OF M ULTI ELEC TR O N A TO M S INCLUDING S P IN STATES 305
function Instead of the spherical harmonics I7™3. It Is easy lo show that the Clebsch-
Gordan coefficients will satisfy the following conditions:
V h L’M _ , V b L M b I. M _ ,
tn$ L
In general the Clebsch-Gordan coefficients bm^ will depend not only on the quantum
number m, = M — mlt but also on L and M. If the orbital angular momentum of the
second particle Is h = 1, then adding both orbital angular moments, we obtain
l
yLU= 2 i (2 ).
mo ——1
We recall that the spatial part of the wave function has already
been obtained (see Chapter 23). For n i ^ n it
tfmAri, n) = — (u — v).
(24.40)
366
where
« =% .(''! ) ^ 3 (/"a).
v = tynl (ri)%1(r |)- (24.41)
Let us now investigate the spin part of the wave function. It has
already been mentioned that in Russell-Saunders coupling the spin
angular momenta are added independently of the orbital angular
momenta. The law of conservation of spin, therefore, provides a
basis for the construction of the spin wave function for the two-
electron system. We shall take the spin function of each electron
to be an eigenfunction of the operator for the z component of the
spin
q Tl
2 °3- (24.42)
(24.44)
s , c = i .,c = |.
Since c | = l , and so on, we obtain from Eq. (24.45) X.2 3/4. The
matrix equation (24.44) for X, is equivalent to a system of two
homogeneous algebraic equations
ci(‘A - M = o ,
c-2 ('A + xi) = 0, (24.46)
1 ) X, — i/i, C\ — 1 , c2 — 0 . (24.47)
380 RELATIVISTIC QUANTUM MECHANICS
Here the spin is parallel to the z axis. The wave function corre
sponding to the eigenvalues l/s is
(24.48)
2) (24.49)
(24.50)
C+ = 1,
and
= 0.
“C ± 7 = c - 7 ; =*=7 *
=:,C ± - = ± C . (24.51)
We shall look for the spin function C of the two electrons of the
helium atom in the form of a superposition
C=a,
-r-a.iC, (24.52)
THE MULTIELECTRON ATOMS INCLUDING S P IN STATES 389
where C, [i!r and are the spin functions of the first and
second electrons, respectively, while ax, a,, a, and are the
Clebsch-Gordan coefficients that have to be determined. Let us now
attempt to select the coefficients a^ of the function (24.52) in such
a way that the latter will be an eigenfunction of the operator for the
z component of the total spin
and also of the operator for the square of the total spin
= + i *'■*")• (24.54)
Here the primes and the double primes on the Pauli matrices
indicate that these matrices act on the spin functions of the
first and second electrons, respectively. We thus have two
matrix equations
4 (° ;+ ° i ) c = n i tc. (24.55)
n-''-y^a'.a")c=n% c. (24.56)
Substituting the total spin function (24.52) and taking into account
the effect of the Pauli matrices on the spin functions of the in
dividual electrons [see (24.51)], we find from (24.55)
which can be obtained with the aid of (24.51), we obtain from (24.56)
2a, = Xsa,, 2 a4 = X2 a4,
aj = (^s— l ) ai. a 2 = (X2 — l)aj. (24.59)
As usual we set X2 = i ’(6’+ 1 ) , where S is the eigenvalue of the
total spin. We note that the system s of equations (24.57) and
(24.59) must be satisfied simultaneously. From these equations
we find that the possible nonvanishing solutions have eigenvalues
X2 = 2,with X, = + 1, 0, — 1 (triplet),
X2 = 0, with Xj = 0 (singlet).
Consider now four cases. Case 1. Total spin is equal to unity and
is directed along the z axis. The spins of both particles are par
allel. The corresponding solution, given by (24.52), is symmetric:
c* = c 1( - I ) c 2 ( - i ) , (m ^ ) , (2 4 -6 1 )
X.2 = 2, a4 — 1,
X, = — 1, al = a .1= ai = 0.
X, = 0. ai = a i = Q.
THE MULTIELECTRON ATOMS INCLUDING S P IN STATES 39
Case 4. Total spin is equal to zero. The spins of the particles are
antiparallel. The solution is antisymmetric:
(24.63)
c' =7f-(c' ( R H ) - c' R ) R ) f
k2= 0, a4 = —a;t = -— ,
X1= 0, a1= a i = 0.
r= y= (u ±v). (24.66)
In the case where both electrons are in the same state («i — n2)»
there is only one solution with a symmetrical spatial part:
It has already been noted that the total orbital angular momentum
L, which is a result of the addition of the orbital angular mo
menta li and of two electrons, assum es integral values (R ussell-
Saunders coupling). In the particular case where f, = f4 = 1 (both
electrons are in the p state), the total orbital angular momentum
takes the values L = 2, 1, 0. In terms of the vector model, these
values correspond to the following situations:
1. L = 2. The angular momenta are parallel;
fit t fa L = l i - \ - l - i = 2.
/ifj/i L = ll —li = 0.
In the general case where /, 5 s /2, the number L takes all possible
integral values
I = /, + /* 1, + — 2.........l y - h - (24.70)
In denoting the energy levels for multielectron atoms, we follow
the same rules as for hydrogen-like atoms, except that the states
with a definite value of the orbital angular momentum L are denoted
by capital letters
L= 0 S state
L= 1 P state
L= 2 D state
L= 3 F state, and so on,
Let us enumerate the energy levels of the helium atom. The lowest
level corresponds to the total orbital angular momentum L = 0 with
both electrons in the s state. This singlet term of parahelium is
denoted by
(lsls)'So
and corresponds to an antiparallel spin orientation. The super
script at the upper left-hand side of the letter characterizing a
particular term indicates the number of states (the multiplicity
of states).
The next term corresponds to the case when one of the electrons
is in the Is state and the other is in the 2s state. In this case, both
parahelium
(ls2s) *S0
and orthohelium
(ls2s) 3 S,
^cc, f o r e x a m p l e , D. I. B l o k h i n l s e v , I ^ r i n c i p l c s o f Q u a n t u m M e c h a n i c s ( I r a n s .) , A llyn
and Bacon, 1964.
2 T h c s c p r o b l e m s r e f e r to C h a p t e r s 2 3 a n d 24.
THE MULTIELECTRON ATOMS INCLUDING SPIN STATES 395
Choose a test function In the form r and determine the upper lim it of the
values of z at which at least one discrete level will exist. Using the obtained solution,
consider the special case of a Coulomb potential (g‘ = ef„ z = 0). Show that discrete
levels always exist for the Coulomb potential. Determine the lowest energy level and the
corresponding wave function.
Solution. The normalized wave function Is of the form [see (23.47)]:
7i — —Yr.
!— . 0 a / 2 e. _p‘
1
The average values of the kinetic and potential energies are [see (23.52), (23.53)]
T= lX,il
2Wo ’ (2Pi + *)*
The appearance of the first discrete level Is possible under the conditions
dE i
E, = T + V = 0 , =0 .
from which we find
max jfs •
D iscrete levels exist for E <C 0. This happens when z < zmax. For a Coulomb field
z = 0 , and therefore this condition is always satisfied. The average energy in the Coulomb
case is
E, 2 m0 Pl*0-
Using the condition that must be minimum, we find
where o0 = — - r ls the radius of the firs t Bohr orbit. The expressions for the wave func-
m0<?5
tlon and the average energy will be exactly the same as those yielded by the Schrodlnger
equation for the Is level of the hydrogen atom (see Chapter 13).
Problem 24.2. Calculate the diamagnetic susceptibility of the ground state of one
gram -atom of parahelium , using the expression (23.47) obtained for the wave function by
the variational method.
Solution. In the ground state of parahelium , the orbital and spin angular momenta are
equal to zero (j = 0), and therefore parahelium must be diamagnetic (see Problem 20.1).
In accordance with (20.48), the diamagnetic susceptibility per gram -atom is given by
the expression
fi]N
6h/oCs Ol +rp.
(24.74)
Z'
Evaluating the integral (24.75) with the above expression for 4,» we obtain
—. — 2al
“1“^*2 £12 ■
The diamagnetic susceptibility Is therefore
ytheor — ] .67 • 10-“,
E= - ™ 1 I
r i-
n Shell symbol
1 K
2 L
3 M
4 N
5 0
6 P
7 Q
1T h e s a m e v a l u e fo r N [ i s f o u n d fo r j j c o u p l i n g . F o r g i v e n v a l u e s o f t h e t h r e e q u a n t u m
n u m b e rs n , I, j , t h e f o u r t h q u a n t u m n u m b e r n i j m a y t a k e t h e v a l u e s —j ............. —Vi , V i j,
c o r r e s p o n d i n g lo 2j + 1 s t a t e s . H e n c e w c o b t a i n t h e s a m e v a l u e fo r N /
N t f b = 2( 2/ f 1) .
w here
O P T IC A L SPECTRA OF ALKALI METALS 399
2
F o r m ore d e t a i l s s e e H a r t r e e , D. R. The Calculation o f Atom ic Stru c tu r e s, N e w York:
J o h n Wiley & S o n s , I n c . , 1957.
400 R E L A T IV IS T IC QUANTUM M EC H A N IC S
(Z — N) e„ (25.3)
where Z is the atomic number, and N is the number of electrons. F or a neutral atom
.V = Z and, therefore, <J>oo= 0 ; that is, the electrons completely screen the charge
of the nucleus.
In constructing a statistical theory, three forms of energy should be taken into
account.
1. The electrostatic energy of attraction between the electrons and the nucleus.
This energy is related to the electron density p0 (the number of electrons per unit
volume) by the expression
where e = — e0 is the charge of the electron, and >hn = ^?'- is the potential produced by
the nucleus.
O P T IC A L SPECTRA OF ALKALI METALS 401
where
*]*e (r) = — e0
i r r'
A’1.
3. The kinetic energy of the electrons in the atom. Just as In the theory of solids at
absolute zero, the average kinetic energy of an Individual electron Is related to the
electron density p0 by the following expression. In accordance with Eqs. (6.33) and
(6.33a)’:
7'av = xPo2/B> (25.4b)
where
x (25.5)
The total energy of an electron gas in the field of a nucleus Is equal to the sum of the
potential energy, which consists of two parts [see (25.4) and (25.4a)], and the kinetic
energy [see (25.6)]. Thus the total energy Is
E = T + V n_e + V e _e =
Po (r) po (/•') d3x d3x' (25.7)
= x ^ Po!s/i d3x — e0 ^ p0fI>n d3x + y e% ^
If —r' I
The density of the electron gas must satisfy the condition
^ p0 d3x = N, (25.8)
5 [E + e0<W } = 0. (25.9)
Using this principle, we can find a relationship between the total potential 3>= <J>n -(- $ e
and the density of electrons p0
P o - ^ - ( 2 / n o f o (4>-<I>o))V2, (25.10)
T h e s e f o r m u l a s w e r e o b t a i n e d o n t h e a s s u m p t i o n th a t a t m o s t two e l e c t r o n s c a n o c c u p y
e a c h quantum s t a t e c h a r a c te r iz e d by th re e quantum n u m b ers. T h u s th e T h o m a s -F e rm i
s t a t i s t i c a l th e o ry a u to m a tic a ll y in c o r p o r a te s th e P a u l i e x c l u s i o n p rin c ip le , w h ic h is of
f u n d a m e n t a l i m p o r t a n c e in t h e t h e o r y o f c o m p l e x a t o m s .
402 RELATIVISTIC QUANTUM MECHANICS
where the Lagrangian m ultiplier 'b0, which plays the role of a constant potential, must
be found from the boundary conditions. In the derivation of (25.10), we have used the
relations
t ^ ^ pos/«5p0 d ’x,
(25.11)
» £l f Po(r) PoO <Pxd}x' =
2 1 \r — r'
[6po (r) po (r’)+po (r) 6p0 (r1)] 3* '
Ir-r'l
d9*; sj f e d‘x = lN.
Substituting Eq. (25.10) for the electron density into Poisson’s equation for a spheri
cally sym m etric electron distribution
and recalling that 4>„ = const, we obtain the T hom as-F erm l equation which form s the
basis of the statistical model of the atom
*4
When a specific problem is investigated, the solution of Eq. (25.13) must satisfy
certaly definite boundary conditions. For an atom the boundary conditions can be given
in the form
- (Z — N ) e , t (25.15)
for r = r 0.
r0
H ere r 0 Is determ ined from the condition that the electron density should be equal to
zero at r = r 0 , that Is, po (r0) = 0. Hence, In accordance with (25.10), we find
N ) e °-. (25.16)
fo
W'ith the help of Poisson's equation (25.12) and (25.13), the condition (25.8) may be
w ritten as
We note that the Thom as-Ferm i equation (25.13) has one exact solution
8l7t!/)' 1
<J>—<I'o r* ’
(25.19)
The solution (25.19) for a neutral atom gives too high a value for <I> as r — co ,
The m ore exact H artree-Fock method shows that the electron density will decrease
exponentially as r — oo .
Since we are Interested only In the basic principles of the statistical method, we shall
construct an approximate statistical theory of the atom with the help of the variational
method. This will enable us to formulate a solution of the problem In analytic form with
only sm all quantitative deviations which are of no Importance to us.
Solving a problem by the Rltz variational method, one can propose any number of
te st functions which depend on the different values of the variational param eter X.
We shall choose the test function on the basis of the following considerations. The
function should agree roughly with the solution of the Thom as-Ferm l equation for
We m a y n o t e t h a t t h e n u m e r i c a l i n t e g r a t i o n o f t h i s e q u a t i o n p r e s e n t s t w o a d v a n t a g e s
o v er th e n u m e ric a l in te g ra tio n of th e H a r tr e e - F o c k e q u a tio n s . F ir s t, th e T h o m a s -F e rm i
eq u atio n i s m uch s im p le r th an th e H a r tr e e - F o c k e q u a t io n s . S eco n d , th is e q u a t io n a n d the
b o u n d a r y c o n d i t i o n s ( f o r e x a m p l e , <I> = 0 f o r a n e u t r a l a t o m w i t h Z = N) c a n b e t r a n s f o r m e d
i n t o a u n i v e r s a l for m w h i c h i s i n d e p e n d e n t o f Z . T o d o t h i s , w e m u s t r e p l a c e O ( r ) b y a
new fu nction
With t h e f u n c t i o n f ( x ) , E q . ( 2 5 . 1 3 ) b e c o m e s
\/* - ~ Y = , (25.13a)
ax
T h e s e e q u a t i o n s a r e o f a u n i v e r s a l c h a r a c t e r , th a t i s , th e y do n o t n e c e s s a r i l y d e p e n d on
the q u a n tity Z . T h e r e f o r e , a f t e r n u m e r ic a lly i n t e g r a t i n g th e T h o m a s - F e r m i e q u a t io n , w e
c a n c h a n g e t h e v a r i a b l e ( w h i c h d e p e n d s o n Z ) a n d u s e t h e e q u a t i o n to i n v e s t i g a t e a n y
h e a v y atom . T h i s c a n n o t b e d o n e w ith th e H a r t r e e - F o c k e q u a t io n .
404 R E L A T IV IS T IC QUANTUM M EC H A N IC S
r —» 0 (this region Is the m ost im portant with reg ard to the solution of the problem as a
whole), and It should have a comparatively simple form , so that It can be exactly in
tegrated in the calculation of the total energy. One tes.t function which satisfied these
requirem ents is
Po =
c- / g (25.21)
1Gw 3/3
This can be easily shown by substituting Eqs. (25.21) and (25.22) for p0 and <J>e into the
equation
V 2O e = 4jce0p0.
Ze 0
Using also the expression 4>n we can see that the total potential satisfies the
r '
boundary condition (25.15) for r = r0—>oo, when the charge density vanishes together
g_5/aYlr
T = 4 t . v. (25.23)
■s V~r
We have the following expressions for the potential energy due to the interaction
between the nucleus and the electrons [see (25.4)] and fo r the potential due to the mutual
interaction of the electrons [see (25.4a)]:
' e- Y T r ZNe-k
ZNe- X’Va (25.24)
Vn - e
8
—7 =- dr ’
Vr 2
Adding Eqs. (25.23)-(25.25), we find that the total energy (25.7) of the electron cloud Is
/; = /U 2— UK, (25.26)
where
— A W * * * (*-?-> l&27)
The variational param eter X, corresponding to the reciprocal of the effective radius
of the atom, can be found from the condition for the minimum of the total energy E of
9 (S rA V a _ A ^ >
R e([ ~ X 100 \ 2 J (Z'_N\ (25.28)
Jl l 25
9 '31 r.JV/ 3 a0
^ 7^ Z
E= (25.29)
4/1
* e ff
(25.30)
E 2515 (2l \ 2 /3 £ i z 7/3= —0.758 ... —z 7 /a .
9 64 \3ji/ flo a«
It Is worth noting that numerical integration of the Thom as-Ferm i equation leads to a
value for the energy of the atom which Is very close to that given by (25.30)
Table 25.1
H 0.769 0.5
Li 9.982 7.5
Na 206.9 162
Hg 21207 18130
406 R E L A T IV IST IC Q U ANT UM M E C H A N IC S
Concluding this section, let us compare the curves which are obtained for the charge
distribution In neutral argon ( Z = 18) on the basis of the T hom as-Ferm l statistical
theory and on the basis of H artree’s method of the self-consistent field (see Fig, 25,1).
It can be seen from the graph that the curve of p0 computed according to the H artree
method (Curve B) has characteristic maxima and minima corresponding to the electron
shells, whereas the curve calculated according to the Thom as-Ferm l statistical theory
(Curve A) describes only the average behavior of the electron density, and therefore has
no relative maxima. F or large values of r there Is also a marked lack of agreement
between the two curves: the H artree-F ock method gives functions which decrease
exponentially with Increasing r, whereas In the statistical theory the decrease Is pro
portional to r~ 4 [see (25.19)].5
F i g . 2 5 .1 . C o m p a r i s o n o f th e e l e c t r o n d e n s i t y d i s t r i
b u t i o n s in a r g o n ( Z = 18) o b t a i n e d on t h e b a s i s of t h e
T h o m a s - F e r m i m e t h o d ( C u r v e A) a n d t h e H a r t r e e - F o c k
m e th o d ( C u r v e B). A p s e u d o l o g a r i t h m i c s c a l e i s u s e d
for t h e o r d i n a t e a x i s (w e h a v e p l o t t e d t h e q u a n t i t y
Z n ( 1 + Dag), w h e r e D = 4 r rr 2 Pg). T h e r e f o r e , t h e g rap h
w i l l b e l i n e a r for s m a l l Da0, a n d l o g a r i t h m i c for l a r g e
Da0. T h e c h o i c e o f t h i s s c a l e e n a b l e s u s to fo llo w
th e v a r i a t i o n o f Pg a t l a r g e v a l u e s o f r, a s w e l l a s for
r < a Q.
5 A d e t a i l e d e x p o s i t i o n o f th e s t a t i s t i c a l t h e o r y o f an ato m i s g i v e n in P . G o m b as , Die
sLnt.ist7sch(> T/ieorit* des A to m s und Hire A nw vndungvn, V i e n n a : S p r i n g e r - V e r l a g , 1949.
O PT IC A L SPECTRA OF ALKALI METALS 407
filled, but there is one electron in the outer M shell. The filling of
the shells in these atoms is illustrated in Fig. 25.2.
It should be noted that the bonding energy of an electron in an
inner shell is much greater that that of an electron in an outer
shell. An indication of this is provided by the ionization energy,
which is more than 2 0 ev for inertgases, whereas for alkali metals
it is only slightly more than 5 ev. The removal of the first valence
electron in lithium requires the expenditure of 5.39 ev, but re
moval of the second and third electrons from the inner shells re
quires the expenditure of 76 and 122 ev, respectively.
The atoms in Group I of the periodic table (Li, Na, K, Rb, Cs,
etc.) are known as alkali metals; each has an outer shell containing
one electron, just like the hydrogen atom. Their optical and
chemical properties, therefore, should be essentially the same as
those of the hydrogen atom (for example, all of these elements are
monovalent and they all exhibit doublet splitting of the spectral
term s).
The optical spectrum is caused by the transition of a valence
electron (that is, an electron in the outer shell) from an excited
state to a lower state. The excitation of electrons in the inner
shells generally requires considerably more energy than does that
of electrons in the outer shells; therefore, downward transitions from
excited states to the ground states of the inner shell are accompanied
408 R E L A T IV IS T IC quantum m e c h a n ic s
V, = - - Z a = - - . (25.31)
£ • = — ££. (25.32)
(25.33)
<25-34)
&= p (25.37)
F ig . 2 5 .3 . “N o n p e n e tr a t in g * a n d " p e n e
t r a t i n g ” o r b i t s in a l k a l i m e t a l a t o m s .
is the electric field produced by the
outer electron at the center of the
atomic core. Thus we can obtain the following expression for the
potential energy associated with the polarization6:
ou ou
T he co efficien t of p o larizatio n i s u s u a l l y d e t e r m i n e d fr om s e m i e m p i r i c a l f o r m u l a s .
I t s n u m e r i c a l v a l u e s f o r a l k a l i m e t a l i o n s ( c o r e s ) a r e a s f o l l o w s ( i n u n i t s o f 10
■24 3s
c m ):
0 . 0 3 ( L i + ); 0 . 1 9 ( N a + ); 0 . 8 9 ( K + ); 1 . 5 0 ( R b + ); 2 .6 0 (C s +) .
410 R E L A T IV IS T IC QUANTUM M EC H A N IC S
«' (/-*/.)/(/+«/.)(/ + D ( / + |
AEP = __2a a n—
3' (25.40)
where
1 n*’
3P
(25.41)
S - W + U b
64 — 3 n* 6l’
We can, therefore, find the total energy, which in this case depends
upon both I and n (for the time being, we are neglecting spin
corrections)
— __ * ! L A - \ p
cP n/-- „2 ^ a cp-
Rh_ el
n2 2a0n2 ’
we obtain
since
l _ 1 1 .2 6
(„ —6 )a n* ii J ~ n2 "T" n 3‘
+ (25.45)
Replacing the wave function for the s states by its value at the origin
[see (25.34)], we find the following approximate expression for the
additional energy of the s term s7:
2 ZelR- e% 28
A£vol ^ 5 ajjrc3 2 n8 ’ (25.46)
where
2 Z/?2
5 ai (25.47)
This expression no longer diverges.
7A s a r u l e , t h e c o r r e c t i o n S f o r p e n e t r a t i n g o r b i t s i s t a k e n i n t o a c c o u n t in t h e f o l l o w i n g
m a n n e r . It i s a s s u m e d t h a t t h e o r b i t c o n s i s t s o f t w o p a r t s : a n o u t e r p a r t a n d a n i n n e r o n e .
In t h e o u t e r p a r t o f t h e o r b i t t h e e l e c t r o n i s a c t e d u p o n b y a p o i n t c h a r g e c q Z b ( i n a l k a l i
m e t a l s Z a = 1); i n t h e i n n e r o r b i t , t h e e l e c t r o n e x p e r i e n c e s t h e e f f e c t o f p o i n t c h a r g e CqZ,-
w hich s h o u ld be g re a te r th a n C q Z b b e c a u s e th e s c r e e n i n g e ff e c t of th e in n e r e l e c tr o n s is
r e d u c e d . It is d if f ic u lt to d e te rm in e th e c h a r g e Z j , th e o r e tic a l ly , a n d it is u s u a l l y r e g a rd e d
a s an e m p iric a l p aram eter.
We t n u s o b t a i n t h e f o l l o w i n g e q u a t i o n f o r 8:
1
— Z; (/ + 1 ), (2 5 .4 3 )
a° ' V2^ Zf-'(/+ ) 1
w h e r e a i s t h e m a x i m u m d i s t a n c e fr om t h e n u c l e u s to t h e e l e c t r o n i n t h e i n n e r e l l i p s o i d
( t h a t i s , t h e r a d i u s o f t h e a t o m i c c o r e ) . I t i s e a s i l y s h o w n t h a t t h e q u a n t i t y 8; d e c r e a s e s
r a p i d l y w i t h i n c r e a s i n g I. A p a r t i c u l a r l y g o o d v a l u e o f i s o b t a i n e d for $ te rm s .
412 R E L A T IV IS T IC QUANTUM M EC H A N IC S
R= ^ 7 ’ (25.48)
where the coefficient 7 , which characterizes the distribution of the
charge inside the atom, is of the order of unity.
Consequently, for the total energy of an electron in the case of
“ penetrating” s orbits, we again obtain an equation of the same
form as (25.42)
Rti ____ el
(n— h f ~ 2ao” eff (25.49)
Table 25.2
Element K
1 11 0 .0 0 0 0 .0 0 0 0 .0 0 0 0,000
3 Li 0.412* 0 .011 0.002 0 .0 0 0
11 Na 1.373* 0.883* 0 .010 0 .001
19 K 2.230* 1.770* 0.140* 0.007
37 Rb 3.195* 2.711* 1.233* 0.012
55 Cs 4.131* 3.649* 2.448* 0.022
T h e v a l u e s o f 8 for p e n e t r a t i n g o r b i t s are i n d i c a t e d by an a s t e r i s k .
O P T IC A L SPECTRA OF ALKALI METALS 413
It follows from Eq. (25.49) and also from the Table 25.2 that,
for a given n , the largest downward shift (that is, in the direction
of decreasing energy) associated with the “ smearing” of the inner
electrons over a finite volume occurs in states with the lowest /.
In other words, the largest shift occurs for s term s . 9
The hydrogen atom alone has no penetrating orbits. For the Li
atom (Z = 3; that is, the next element after hydrogen in the first
group of the periodic table), the only penetrating orbit is the outer
s orbit. In the next alkali metal, namely, the Na atom (Z = ll), the
s and p orbits are penetrating.
E. FUNDAMENTAL SERIES
(1 s) = ^ = R,
^In C h a p t e r 21, w h e n t h e f i n i t e s i z e o f t h e n u c l e u s w a s t a k e n i n t o a c c o u n t , i t w a s a l s o
f o u n d t h a t t h e l a r g e s t s h i f t o c c u r s f o r s t e r m s . In t h a t c a s e , h o w e v e r , i t w a s t h e p o s i t i v e
c h arg e t h a t w a s s p r e a d o v e r a f in ite volum e, and th e re fo re th e te rm s w e re s h if te d u p w a rd s
and not dow nw ards.
414 R E L A T IV IS T IC QUANTUM M EC H A N IC S
1 0 If t h e p r i n c i p a l q u a n t u m n u m b e r n i n l i t h i u m a s s u m e s t h e v a l u e s n = 2, 3, 4 ( t h e t e r m
n = 1 i s o c c u p i e d by tw o e l e c t r o n s an d form s an in n e r s h e ll ) , th an th e q u an tu m n u m b e r n *
t a k e s t h e v a l u e s rt * = 1, 2, 3 - • • •
O PT IC A L SPECTRA OF ALKALI METALS 415
where
p = — = — 0.041,
d = — 8rf= —0.002.
m= (l*s) — (n*p).
which means
11 At p r e s e n t , s p e c t r o s c o p i s t s h a v e a l s o a d o p t e d a n o t a t i o n w h i c h f o l l o w s fr o m t h e o r e t i -
c a l c a l c u l a t i o n s . T h i s i s t h e n o t a t i o n u s e d in F i g . 2 5 . 4 .
416 R E L A T IV IS T IC QUANTUM M EC H A N ICS
which means
M = ± 1.
The names of the series partly reflect the character of their
multiplet structure.
R a n (25.60)
h i l ' Z'ct
n 7+ ‘ 4
OPTICAL SPECTRA OF AL K A L I METALS 417
Aujt zr ‘•'Pl/2 4 s
'•Put 3?
3 P 3 /2
2 s
?P,/2
?PV2 Aivjl ?Pyt
Aui, ~T~ A tv,] AiVt |
2 P;/t
2 Put
i
/ s
F i g . 2 5 .5 . S p e c t r a o f a l k a l i m e t a l s .
1 for i = o
2
and
R°?Z'ei{ (25.64)
Ao>„
2nz
12
S e v e r a l o t h e r fo r m u la s h a v e a l s o b e e n p r o p o s e d for th e d o u b l e t s p littin g . F o r e x a m p le ,
on th e b a s i s of t h e q u a s i - c l a s s i c a l p i c t u r e of p e n e t r a t i n g o r b it s , L a n d e p r o p o s e d a form ula
r e p l a c i n g in ( 2 5 . 6 3 ) ri b y n c ( f n ~ $ a n d ^ e f ( by Z,- , w h e r e Z a i s t h e t o t a l c h a r g e o f
t h e i o n a n d Z , i s t h e e f f e c t i v e c h a r g e o f t h e n u c l e u s in t h e i n n e r r e g i o n [ s e e E q . ( 2 5 . 4 3 ) 1
i n t o w h i c h t h e o r b i t p e n e t r a t e s . F o r f u r t h e r d e t a i l s w e r e f e r t h e r e a d e r to E . C o n d o n a n d
G. S h o r t l e y , T h e o r y o f A t o m i c S p e c t r a , N e w Y o r k : C a m b r i d g e U n i v e r s i t y P r e s s , 19 5 8 .
O PTIC A L SPEC TR A OF ALKALI METALS 419
account the selection rule for the total angular momentum quantum
number
A/ = 0, ± 1.
The spectral lines of the fundamental series are also split into
three components.
j n * d 5 /2
I n
I
I *
1 ' I ?P)/2
--------------- 1----------------fp,/2
F ig. 25.7. S p litting of the d iffu se
series.
From the last equation it is evident that X can vary from oo (as
in the case when an electron does not lose any energy during the
collision with the target^-/Huu'1= v ) to a certain minimum value
y ___ ch (26.1)
m in ~y >
as in the case when an electron loses all its energy in the collision
by the nuclear charge (Ze0) and the charge of the electrons in the
inner orbitals [— (Z — l)r0]. The total potential was equal to
®= (26.3)
as the main term in the expression for the potential, and to regard
the additional potential produced by the electron shell as a correc
tion. In this case, the presence of electron shells will result in a
screening (an effective decrease) of the nuclear charge Ze0 by the
amount S„e0, and the total potential will be
En = - (— (26.5)
(Z — S,)2 (Z — S2)2
n l2 22
(26.6)
-«[
424 R E L A T IV IS T IC QUANTUM M EC H A N IC S
= (26.7)
’\Vf —
r =1 ^ ■ Z - S n
(26.8)
Vf — rh
1S e e A. Som m erfeld, Alom huu und Spcclriillinicn I, V icw cg., H raunschw eip;, 1951,
C h a p t e r s 4 a n d 5.
M E N D E L E Y E V 'S P E R IO D IC S Y S T E M 425
(26.10)
S is - 3, S ip = 4.
|f f (Ll terms);
\ f = + (Z ~ 4)3 \ ( U \ t e r m s ) ; (26.12)
I /^ 2P 3/„ Z —4 . 1 ..3 1
]/ — = — 2“ + J 6 (2 — 4) t e r m s ) .
Mi (3si/s), M m(3pi/,,),
Mm (3/73/j), M[v(3d3/2), Mv(3ds/,,),
where the screening constants are S3s = 8.5, S3p= 10, S:w= 1 3 .
There are seven components for the N terms.
The parallel doublet L\ and Lu [see (26.12)], which is due to
the screening of the nucleus by the electrons is known as an irreg
ular, doublet, whereas the diverging doublet L\ and LM is known
as a regular doublet. The reasons for the adoption of this terminology
M E N D E L E Y E V 'S PER IO D IC SYSTEM OF ELEMENTS 427
T h e l a r g e s t n u m b e r o f t r a n s u r a n i u m e l e m e n t s h a s b e e n d i s c o v e r e d by G. S e a b o r g a n d
h is stu d en ts. The disco v ery of la w re n ciu m (L w ) w ith Z 10 3 w a s r e c e n t l y a n n o u n c e d .
M E N D E L E Y E V 'S P ER IO D IC SYSTEM OF ELEMENTS 429
T h e o r d e r in w h i c h t h e e l e c t r o n s u b s h e l l s a r e f o r m e d c a n b e r e m e m b e r e d m o s t s i m p l y
w i t h t h e h e l p o f t h e f o l l o w i n g e m p i r i c a l r u l e : t h e l e v e l s a r e f i l l e d in t h e o r d e r in w h i c h t h e
s u m o f t h e p r i n c i p a l a n d o r b i t a l q u a n t u m n u m b e r s , n + I, i n c r e a s e s , an d l e v e ls w ith th e
s a m e v a l u e o f t h i s s u m a r e f i l l e d i n o r d e r o f i n c r e a s i n g n. S i n c e I t a k e s t h e v a l u e s
0, 1, 2, n — 1, w e c a n f i n d t h e r u l e f o r f i l l i n g t h e t e r m s in a n y s h e l l . F o r e x a m p l e , t h e
fourth p e rio d ( s e e b e lo w ) w ill b e f i l l e d in th e o rd e r
a n d t h e s i x t h p e r i o d in t h e o r d e r
6 s ( n 4 / = 6); 4f ( n + I = 7);
5d { n 4 1 = 7); 6 p ( n + I = 7).
430 R E L A T IV IST IC QUANTUM M E C H A N IC S
112-118
-7p
104*-112*
rtTh■ i No,103
-6 a
oAc
—5f Q (32)
-6 a
i Fr - s»Pa
-7s
, T l -..Em
-6p
7?Nf -joHg F ig . 26.6. Diagram of th e fillin g of energy
-5a l e v e l s in t h e p e r i o d i c s y s t e m o f e l e m e n t s .
a CP 7# I—Li
-Uf(») P (32) O n ly s a n d p s u b s h e l l s c a n b e p r e s e n t in th e
7La
-5a o u t e r s h e l l of an a to m . A d s u b s h e l l c a n b e
f i l l e d o n ly b e g i n n i n g w ith t h e f i r s t i n n e r
•6s s h e l l . An f s u b s h e l l c a n b e f i l l e d o n l y b e
g in c.X 6
g i n n i n g w ith t h e s e c o n d i n n e r s h e l l . T h e
-5 p f i l l i n g o f t h e 3c? s u b s h e l l g i v e s t h e f e rr o
-Ua 0 (18) m a g n e t i c e l e m e n t s ( F e , C o , N i). T h e fil
l i n g o f t h e 4f s u b s h e l l g i v e s t h e l a n t h a
-5s n id e s or rare e a r th s (5 8 C e — 7 iL u ) . T h e
36^r f i l l i n g o f t h e 5f s u b s h e l l g i v e s t h e a c t i n i d e s
-4p ( 9 0T h — 103). T h e a s t e r i s k s d e n o t e t h e a to m
ic num bers of e le m e n ts w hich h a v e n o t y e t
-3a(t(/)\ N (18) b e e n d i s c o v e r e d . T h e m ax im u m n u m b e r of
-Us ) e l e c t r o n s in a g i v e n s h e l l o r s u b s h e l l i s i n
d i c a t e d in p a r e n t h e s e s . E m a n a t i o n (E m ) ( th e
,Al ... Ar
e l e m e n t w i t h Z = 8 6) i s a l s o c a l l e d r a d o n (Rn).
/iNo - ,?Mg ~3P ) M (8)
-3s J
56 - «7Ne
-77 V {s>
,n - 2He
-1(2)}K (2)
Rfi Rh
(3 — 0.146)- 2.854“"’
(26.13)
Rh Rh _
(4 - 2.23)- 1.77- •
It can be seen that Z;M^>E4s, and hence the 4s level will be filled
before the id level. Consequently, the third period will contain only
seven elements (,,Na — ,sAr), just like the second period.
MENDELEYEV'S PERIODIC SYSTEM OF ELEMENTS 4 31
We shall now attempt to treat the ground-state configuration of the elements m ore
rigorously.
In a paper devoted to the statistical theory of the atom (1928), Ferm i proposed a
method, now known as the Thom as-Ferm l method (see Chapter 25), to explain the periodic
system of elements. With this method, he obtained the minimum values of Z for which
s , p, d and / states are filled in atoms. He obtained these values by starting from the
following quasi-classical ideas.
In classical theory the angular momentum of a particle L is related to the momentum
p by the expression
L = r x p.
Consequently,
This relation represents a com prom ise between the Bohr relation L2 = 1i2 (I + 1)2 and
the quantum mechanical relation L2 = Ti2l (I + 1), B
The maximum momentum P is related to the density of the electron gas (electrons in
the atom) p0by the expression (6.32)
The electron density po may be found from the T hom as-Ferm i equation (see Chapter 25),
which, as we have already indicated, can be solved only by approximate or numerical
methods. A good approximation for p0, as follows from a solution of the Thom as-Ferm i
equation, is provided by the expression [see (25.21)]
(26.17)
16w3/2
where the coefficient X is found by the Ritz variational method.
Substituting these values for P 2and L2 into the inequality (26.14), we get
■t eZy/ i X , (l + lh ) 2
(26.18)
16 J r r2
3 * D (26.19)
e > —
x
where
1 16 \ a/a
D= (26.20)
faZ)
From the inequality (26.19) It Is evident that the right-hand side of (26.19) becomes
g re a te r than the left-hand side as x —- 0 (r — 0) and as x — co . The electrons in the atom
therefore can have a given value of / if x lies In the range Xi < x < x 2 for which the
Inequality (26.19) is satisified. Here x t and x» are roots of the equation
e- T /F = £ (26.21)
x ‘
The condition for the appearance of states with a given value of / is the equality of
both roots
X t
In this case we should equate not only the two functions them selves, but also their de
rivatives. Then, in addition to Eq. (26.21), we will have
_1_ O~ I« Vx D_
(26.22)
3 Vx x 2’
M E N D E L E Y E V 'S PERIO DIC SY STEM OF ELEM EN TS 433
Vx = 3,
that is, when
D = 9e-».
Substituting the value for D from (26.20), we find the value of Z at which electrons with a
given / will first appear
where e = 2.718... Is the base of the natural logarithms and the coefficient 7 Is equal to
0.158.
A numerical solution of the Thomas-Fermi equation gives a very similar value for 7
7 X-F — 0.155.
This again Is a convincing demonstration that the density (26.17) represents a good
approximation to the density which Is given by a numerical solution of the Thomas-
Fermi equation.
Equation (26.23) enables us to calculate the Z values at which the s, p, d, and / states
begin to be filled. The results of this calculation are given in Table 26.1. The first row
gives fractional values of Z computed from formula (26.23) with 7 p_p = 0 .155. The
values of Z, calculated with 7 ^. p = 0.158, for which s, p, d, a n d /sta tes first appear
are practically identical with those calculated with 7 -p_p = 0.155. The second row gives
the nearest greater integral value of Z. The last row of the table gives the empirical
values of Z at which the states first appear, and also the symbol of the corresponding
elements.
Table 26.1
Atomic numbers at which the s, p, d, and /sta te s first appear for a given I
I s P d /
0 1 2 3
From this table it can be seen that this approximate theory is In good agreement
with the experimental data. We may note that complete agreement Is obtained if the co
efficient 7 is taken to be 0.169 instead of 0.155.
It is well known that in light elements (Z = 1, 2, 3, 4) only s states are filled. The
filling of the p states begins with boron (Z = 5); this is in complete agreement with
theoretical data. Table 26.1 shows (in spite of the crudeness of the statistical model)
that the filling of the 3d subshell does not begin, as might be expected, in potassium
(Z = 19), but in scandium (Z = 21); that is to say, it does not begin until the 4s subshell
Is completely filled. Similarly, the Thomas-Fermi model explains the "delay” in the
filling of the 4/ subshell, which might be expected to begin in Ag (Z = 47). According to
the theory, however, the filling of the 4 / subshell should be shifted, and should begin only
in cerium (Z = 58). It follows from (26.3) that the filling of the 5g subshell (I = 4) would
begin in the element with Z = 124.
The Thomas-Fermi model accounts for a very important feature of the ground-state
configuration of atoms and explains the departure of the filling of the levels from the
ideal scheme (the hydrogen scheme) in terms of the "smearing” of the electron cloud.
434 R E L A T IV IS T IC QUANTUM M E C H A N IC S
STUc d i s c o v e r y a n d p r o p e r t i e s of th e n ew e l e m e n t s a r e d i s c u s s e d in G. T. S cab o rg ,
Thr Transuranium bllr m o n ts , R e a d i n g , M a s s . , A d d i s o n - W e s l e y , 1958.
Chapter 2 7
B. HETEROPOLAR MOLECULES
r '
However, inside the atomic core the charge will be Z,|>Zn; that
is, the screening of the nuclear charge will not be complete (see
Chapter 25). In exactly the same way, the outer electrons will
completely screen the remaining part of the nuclear charge (Zae0)
only in the region outside the outer shell (that is, only in the case
of excited states). While in this case there appear polarization
forces proportional to r*, they are not able to hold an additional
electron. In the outer shell itself the charge will be incompletely
compensated; for this reason (but provided there are unfilled
states in the outer shell), the incompletely screened part of the
nuclear charge will hold additional electrons in this shell, thus
forming a negative ion. The rule is that the le ss electrons there
are in the outer shell of a neutral atom, the larger the total
screening of the nuclear charge in this shell. Therefore, an alkali
metal will lose the one electron in its outer shell more readily
than it will acquire additional electrons.
A curve showing the dependence of the ionization potential on
Z is plotted in Fig. 27.1. It shows a minimum for alkali metals and
a maximum for inert gases. This curve reproduces rather faith
fully the periodicity exhibited by the number of electrons in the
outer shell.
In inert gases the ionization potential reaches its largest
value; the removal of an electron from the outer shell and its
transfer to another atom require a very large expenditure of
energy. In addition, no further electrons can be held in the outer
shell, which is completely filled. Therefore, inert gases do not
participate in ordinary heteropolar compounds (we shall also see
that they do not form homopolar bonds), and hence, as a rule,
they exist as unassociated atoms.
Atoms of alkali and alkaline earth metals readily give up their
valence electrons to another atom (the ionization potential is at
*F o r example, in sodium (Z 11), Ihe ten electrons in the inner shell completely
screen ten units of n u c le a r charge, which l e a v e s only th e e l e v e n th unit of n u c l e a r c h a rg e
to h e (partially) screened by the outer electron. In c h l o r i n e ( Z - 17) t h e t e n inner elec
trons completely screen only ten units of n u c l e a r c h a rg e , so that the seven e l e c t r o n s of
the outer s h e ll m u st s c r e e n the r em ain in g ch a rg e , w hich they c a n a c c o m p li s h only p a rtially .
Therefore, a c h l o r i n e at o m i s a b l e to h o l d an a d d i t i o n a l e l e c t r o n m o r e e a s i l y t h a n s o d i u m ,
and is thus converted into the n e g a tiv e i o n Cl * . O n the other hand, a sodium atom g iv e s
up i t s o u t e r e l e c t r o n mo re r e a d i l y , a n d in t h i s w ay f o rm s a p o s i t i v e ion N a .
THE THEORY OF S IM P L E MOLECULES 439
its minimum here), and thus convert to positive ions (for example,
a Na+ ion).
F ig . 27. 1. T h e d e p e n d e n c e of th e i o n i z a t i o n p o
t e n t i a l o f a n e u t r a l atom on t h e a t o m i c num ber.
Electron Affinity
Electron affinity
Element
(ev)
H 0.71
F 4.13
Cl 3.72
O 3.07
— F i? 1*.
This energy has been very carefully determ ined from experim
and found to be equal to: = 4-2 ev. Hence for the Coulo
F i g . 27 .3 . T h e f o rm atio n o f an N a C l m o l e c u l e
from Na"*” a n d C l i o n s . T h e i o n i z a t i o n p o t e n
t i a l o f s o d iu m (5.1 ev ) a n d th e af fi n i t y of the
c h l o r i n e ato m for an e l e c t r o n (3 .7 ev) a r e i n d i
c a t e d in p a r e n t h e s e s . T h e C o u lo m b b o n d i n g
e n e r g y b e t w e e n t h e i o n s in t h e m o l e c u l e i s
5 . 6 ev.
It should be noted that we have not considered here all the in
teractions that occur in a heteropolar molecule. In addition to the
Coulomb forces of attraction, there will also be repulsive forces;
these exceed the Coulomb forces at small distances and prevent
the two atoms from approaching closer than the distance R. In
any case, this elementary discussion explains the principal physical
processes involved in the formation of heteropolar molecules; it
also explains, however qualitatively, the ionic structure of their
crystal lattice and the dissociation of these m olecules into indi
vidual ions, a process which occurs in solutions.
H= T——
r----el r , H (27.2)
(£a - T + ^ = 0 ,
(27.5)
(* •- t + f )‘
THE THEORY O F S I M P L E MOLECULES 443
Ea = Ea, = Ex= - R h , /
(27.6)
to = tino (r). t o ' = tioo (/■'). r
r
where tn/m = tn»o is the wave function of
R a'
the ground state of the hydrogen atom Electron 1 Is near
[see (13.32)], namely, nucleus a i
(27.10)
For the solution | 0 (the electron near nucleus a), es represents the
main interaction, and - ^ the perturbation. On the contrary, for
the solution (the electron near a'), the main interaction i s - —,
and the perturbation is - —.
From Eq. (27.10) we can find the additional energy £', and
also a relationship between the coefficients C, and C., in the wave
function This can be done because the perturbation energy
removes the degeneracy (just as in the case of the helium atom).
444 RELATIVISTIC QUANTUM MECHANICS
Here we have used the fact that the wave function of the ground
state of the hydrogen atom is real.
In exactly the same way, assuming that the electron is near the
nucleus a', we may neglect the perturbation energy on the left-
hand side of Eq. (27.10) (in this case the perturbation energy is -^).
Since the solution of the homogeneous equation will now be the
function tya„ we obtain a second equation for the unknown quantities
-;-C, J = (27.12)
C* + +
+ C, J ' F - \ - ' ; - - f j * a.<Px = 0. (27.13)
J i]£<Fa: = 1. (27.14)
This follows from the fact that, although the expression S (R)
vanishes for R -*• oo (there will be no points where the two functions
<|i„ and both differ from zero), it becomes equal to unity for
and
(27.16)
T /'2(I+S)
('i'a + 'lV).
(27.18)
, A — SK
E ' s = V s {R) = K ' 1+S ’
^ tjja 6 s d'x = 0 .
e ~ k°r 1 f elkr d ’k
r '-2 7 1 s ) k* + *§ (27.20)
e i hr
'Woo ( r ) = — —
y no]
e
(k- + w d'k, (27.21)
where kn= —.
In accordance with (27.15), the expression for S becomes
ei>R
Ik'1 + k-y d*k
(27.24)
THE THEORY OF S I M P L E MOLECULES 447
The integral in this equation can be calculated with the aid of Eq.
(27.20), which should be differentiated three times with respect to
the parameter ArJ.
In order to calculate the additional Coulomb interaction, we
shall use the relation
(27.25)
In exactly the same way, we may represent the quantity l/R in the
form of the integral (27.25). Substituting these quantities into
formula (27.16) and integrating over the volume d'x with aid of
(27.23), we find
(27.26)
2
Hence, using (27.20) and setting k0= —,
Qq
we obtain an expression
for the additional Coulomb energy
(27.27)
(27.28)
where 6 0 = —. Hence
(27.29)
448 R E L A T IV IST IC QUANTUM M E C H A N IC S
ya,s el +
R I 1 \ - (27.30)
1 + ( l + E+
Here the upper signs (-) refer to the antisymmetric solution V3,
while the lower signs (+) refer to the symmetric solution. For
sm all values of R (/?<Ja0), we have
y a ,s _ - f- 2 el B-e~%
'"J Go G q
(27.32)
p: = ( r r = 2 (1 [_ S) (fa + r* - 2 * ,u - (27-3?)
state state
The first solution o(1(I. corresponds to the case when the first electron
is near the nucleus a (and the second electron near the nucleus a'),
while the second solution ']>a,a corresponds to the case when the
first electron is near the nucleus a', and the second electron is
near the nucleus a. Both these possibilities are depicted in Fig.
27.7, where the solid arrows show the atomic bonds, and the
dashed lines show the molecular bonds. When the distance between
the nuclei tends to infinity ( £ - > - 0 0 ) , all molecular bonds vanish.
F i g . 2 7 .7 . D ia g r a m o f t h e i n t e r a c t i o n s
in t h e H 2 m o l e c u l e . T h e s o l i d l i n e s
join the p a rtic le s w hose in teractio n is
t a k e n in to a c c o u n t in t h e z e r o t h a p
p r o x i m a ti o n . T h e d a s h e d l i n e s d e n o t e
i n t e r a c t i o n s w h i c h are r e g a r d e d a s
p ertu rb atio n s; a and a are the n u c le i
of t h e h y d r o g e n a to m s; 1 a n d 2 a r e
electrons.
SA b a r o v e r a sy m b o l d e n o t e s q u a n t i t i e s r e f e r r i n g to t h e n e u t r a l m o l e c u l e .
452 R E L A T IV IST IC QUANTUM M E C H A N IC S
K= J <]£„• { f + jrt - 7[ - 7 7 }■
d3*i -(27.40)
where the first and second parenthetic term s in the integrand cor
respond to the potential energy of repulsion between the two nuclei
and the two electrons, and the third and fourth terms correspond to
the potential energy of attraction between the first electron and the
nucleus a' and between the second electron and the nucleus a.
In exactly the same way, we obtain the exchange energy
$ a = ,, ■ •- (<v. - <?«■«).
V 2(1 - S )
(27.43)
K—A
F a (/?) =
1 —S '
The general form of the curves of V a~s for a neutral hydrogen mole
cule is approximately the same as for the H.2 molecular ion; there
fore, only symmetric solutions will give stable m olecules. For the
radius corresponding to the equilibrium position [that is, the
minimum of the potential energy Vs (R) for the symmetric solution],
we obtain Rlt — 1.51 a„ — 0.80 A. The corresponding value for the
dissociation energy is
In the 1I2 ion and the H atom, there is only one electron, and its
spin leads only to insignificant spin-orbit interactions. On the
other hand, there are two electrons in the H.2 configuration, and the
spin plays an important role in the theory of this molecule, even
though the spin-orbit and the spin-spin interactions give only
small corrections. In the hydrogen molecule, just as in the helium
atom, the mutual orientation of the spins of the two electrons
determines the type of symmetry of the spatial part of the wave
function; this is of primary significance in connection with the
stability of a molecule. We shall therefore consider more fully
the question of the relation of the spin to the symmetry properties
of the molecule.
The total wave function T must contain a spin part in addition
to the spatial part. When the potential energy of the spin-orbit
interaction can be neglected, then, just as in the case of R ussell-
Saunders coupling, the total wave function can be represented by
a product of a spatial part and a spin part. For electrons (which
obey Fermi statistics) the total wave function must change sign
when coordinates and spins are interchanged (that is, the solution
must be antisymmetric). We therefore have two possibilities
6 ]n th e c a s e of r e f l e c t i o n in t h e x z p l a n e , mirror r e f l e c t i o n a m o u n t s to a r e p l a c e m e n t of
y by - y.
7A s is w e l l k n o w n , th e a n g u l a r m om entum /, r x p is an a x i a l v e c t o r w h o s e d i r e c t i o n
is a m a t t e r of c o n v e n t i o n (it h a s o n e d i r e c t i o n in a r i g h t - h a n d e d s y s t e m of c o o r d i n a t e s ,
a n d th e o p p o s i t e d i r e c t i o n in a l e f t - h a n d e d s y s t e m ) . T h e d i r e c t i o n o f th e c o n t o u r b o u n d i n g
the a r e a a n d c o n s t r u c t e d from t h e v e c t o r s r a n d p r e m a i n s , h o w e v e r , u n a l t e r e d in b o th th e
rig h t- a n d l e f t - h a n d e d c o o r d i n a t e s y s t e m s .
THE THEORY OF S IM P L E MOLECULES 455
B B'
F i g . 27 .8 . C h a n g e o f t h e a n g u l a r m om entum on r e f l e c t i o n
in t h e A A ' B B p l a n e , w h i c h p a s s e s th r o u g h t h e a x i s o f s y m
m etry z. If t h e i n i t i a l r o t a t i o n c h a r a c t e r i z i n g t h e a n g u l a r
m om entum o c c u r s in a p l a n e p e r p e n d i c u l a r to A A B B ( s e e /),
the d irectio n of th is ro tatio n w ill be re v e r s e d a fte r reflec tio n
( s e e I'). If, h o w e v e r , t h e r o t a t i o n t a k e s p l a c e in t h e p l a n e o f
r e f l e c t i o n , it w i l l b e u n a l t e r e d by r e f l e c t i o n (// = / / ) .
and so forth.
The importance of symmetry with regard to the formation of a
molecule follows also from the fact that the ground state of most
diatomic molecules is a state in which the wave function is invariant
under all symmetry transformations . 8 Thus, is the main term
of the hydrogen molecule. The question of molecular symmetry,
however, lies outside the scope of this book.
It should be noted that in stable states of the hydrogen molecule
the spins of the two electrons are always oppositely directed. At
the same time, there are two types of hydrogen m olecules—para-
hydrogen and orthohydrogen. These names refer to the orientation
of the nuclear spins, and not to the orientation of electron spins.
In parahydrogen the spins of the nuclei are antiparallel, while in
orthohydrogen they are parallel. Since the number of possible
states for two particles with parallel spins is three tim es larger
than in the case of particles with antiparallel spins, ordinary hydro
gen at room temperature will consist of an equilibrium mixture of
25% parahydrogen and 75% orthohydrogen. As the temperature is
lowered in the presence of a catalyst (for example, charcoal), the
percentage of parahydrogen in the equilibrium mixture increases,
and is practically 100% at 0°K. Parahydrogen produced at low
temperatures is extremely stable and can be preserved in such
an equilibrium system for a period of several weeks at room
temperature. Orthohydrogen has not yet been obtained in pure
form. The difference in the thermal conductivities at low tempera
tures (the thermal conductivity of parahydrogen is larger) is used
for determining the composition of the mixture. Similarly, para
hydrogen and orthohydrogen have somewhat different dissociation
energies and optical properties.
Is 2s 2p
H [T J Us')
He n 1 Us2)
t I I Us22s22p<)
lP m (ls22s’2p2)
t t i (ls22s72p3)
m i l {ls22s22p3)
N 11 t t t i Us22s'2p3)
0 LI M i l ) I Us22s22p;‘)
F I I I I I I I Us22s22p5)
I I I I I I I I [ls22s22ps)
F i g . 2 7 .9 . D ia g r a m s h o w i n g t h e f i l l i n g
of the e le c tr o n s h e l ls o f s e v e r a l atom s
w i t h t h e s p i n t a k e n i n t o a c c o u n t . Homo-
p o l a r v a l e n c e o f a t o m s i s d e n o t e d by a
dot, a n d i o n i c v a l e n c e i s d e n o t e d by
a I ( p o s i t i v e ) o r —( n e g a t i v e ) sign.
Table 27.2
M ultiplicity and hom opolar valen ce
Group of the
periodic sys 1 II III IV V VI VII
tem
Multiplicity 2 1 1, 3 2, 4 1. 3, 5 2, 4, 6 1, 3, 5, 7 2, 4, G, 8
Valence* 1 j 0, 2 I, 3 0, 2, 4 1, 3, 5 0, 2, 4, 0 1, 3, 5, 7
|
* T h e b o ld -f a c e ty p e in d i c a t e s th e p rin c ip a l v a le n c e .
THE THEORY OF S IM P L E MOLECULES 459
Thus, the stability of even the best existing quartz oscillators is inadequate in a
number of cases (for example, high-stability oscillators are necessary for accurate
determination of distances by means of radar). For this reason, one of the great achieve
ments of recent times was the development of m asers9by Townes and coworkers, and
independently by Basov and Prokhorov. In this device, quantum transitions between
discrete energy levels in a molecule are used as microwave generators.
We note first of all that the process of spontaneous em ission, which depends on the
Einstein coefficient A, has no essential significance in the case of molecular emission,
since its intensity is proportional to u* and is extremely small in the radiofrequency
region (that is, in the range of frequencies that are low compared with light frequencies ji
As far as induced transitions, which are proportional to the Einstein coefficient B, are
concerned, the probability of upward transitions (resonance absorption) and the probability
of downward transitions (induced emission) are identical (see Chapter 9); they are pro
portional to the energy of the field. Therefore, a system (molecule) with two levels will
undergo a transition from one level to another under the Influence of a sufficiently strong
external field containing the resonant frequency, the transitions being accompanied by the
em ission and absorption of quanta.
F i g . 2 7 . 1 0 . S t r u c t u r e o f t h e am m o n ia m o l e c u l e
s h o w i n g tw o m i r r o r - s y m m e t r i c s t a t e s a a n d b
of th e sa m e energy.
If external radiation is passed through a substance, it will interact with its molecules
and cause resonant absorption as well as induced emission. In accordance with the
Boltzmann distribution (the number of particles in an equilibrium system with energy E
is proportional to e ~ E/kT), there must be fewer particles in the higher energy states
than in the lower states; therefore, absorption dominates induced emission in the thermal
equilibrium. The excess of absorbed energy is completely converted into the energy of
thermal motion of the molecules, raising the temperature of the gas.
In order for the system to amplify and not absorb the radiation incident on it, it is
necessary to disturb this thermal equilibrium in such a manner as to produce greater
occupancy of the higher energy levels than of the lower o n es.10 When this is done,
such a system will generate electromagnetic waves with extremely small line width
under the action of resonant radiation. The first masers were produced with ammonia
(NHa) molecules. The ammonia molecule consists of one nitrogen atom and three
hydrogen atoms and forms a right pyramid. However, from the laws of symmetry, the
stable state of the ammonia molecule corresponds not only to the case in which the
nitrogen atom is located above the triangle composed of the hydrogen atoms (Fig. 27.10a),
but also to the case in which the nitrogen atom is located at the same distance under the
triangle (Fig. 27.10b). The stable states a and Zihave equal energies and correspond to the
minima of the potential energy; they are thus separated by a potential barrier, so that
classical theory does not allow transitions from one to the other. In quantum theory,
however, the probability of penetration across the potential barrier Is different from
zero. Consequently, the nitrogen atom, which Is above the triangle, may reposition It
self below the triangle In the absence of any external action, and may then return to Its
original position. In molecular physics this phenomenon is called inversion. The process
of Inversion can be explained In terms of the existence of two perfectly Identical potential
wells, separated by a potential barrier of finite width. It is well known that this leads to
a splitting of the spectral lines, in spite of the fact that the energy states in the two po
tential wells are completely identical to one another. For the ammonia molecule this
splitting, expressed In wavelengths. Is 1.27 cm for the most intense line; this corre
sponds to a radiofrequency wave.
Another remarkable property of the ammonia molecule is that, with the help of an
electric field. It Is fairly easy to separate the molecules In the upper and lower energy
levels that have been formed as a result of Inversion. It Is found that when an ammonia
molecule is placed In an electric field, the two levels are shifted in different directions,
the upper Inversion level being shifted upwards (the energy increases) and the lower
inversion level downwards.
Since any system tends to a state In which its potential energy is minimum, we first
pass a beam of ammonia molecules through a carefully evacuated vessel containing an
electrostatic field which is produced by a quadrupole capacitor; this field decreases
towards the symmetry axis (see Fig. 27.11).
In this separating system the molecules in the upper level will tend to move to a
region where this field Is minimum (since the field increases their energy); that is,
they will be focused about the axis of the capacitor. The molecules that are in the lower
level, however, tend to move to the region of maximum field (that is, to the periphery,
where their energy will be minimum), and thus they will be ejected from the beam. After
passing through the above separating system, the beam of molecules, which contains a
predominance of excited molecules, enters a cavity resonator which, among its rather
wide range of radio frequencies (produced by conventional radio frequency methods) also
contains the resonant frequency of the molecular transitions.
This external radiation causes molecular transitions which are associated with a
line of small width (•%. 10 s e c -') atafrequency of 2.4 x 1010se c - ' (that is , with a relative
error no greater than 10~g), corresponding to the wavelength of 1.27 cm. This wavelength
is determined mainly by the lime of traverse of the molecules through the cavity resonator.
A molecular generator produced In such a way exhibits extremely stable frequency.
The frequency stability is so high that molecular clocks constructed on this principle
have an accuracy of approximately 1 sec per 300 years of continuous operation.11
On the basis of this principle, several types of radio receivers working in the micro-
wave region have been constructed. They all use paramagnetic crystals cooled to very
low temperatures. These devices, known as quantum paramagnetic amplifiers, greatly
increased the sensitivity of radio astronomical and radar equipment.
Recently, atoms and molecules have been used as generators and receivers in the
visible spectrum. In this case, we speak of lasers (light amplification by stimulated
emission of radiation).
What is the difference between an ordinary source of light and a laser? In an ordi
nary source of radiation, for example, the Sun, the spectrum consists of a broad fre
quency band and the individual incoherent quanta have arbitrary phases and directions.
Optical lasers enable us to obtain a monochromatic beam of light of high intensity (when
it is focused we obtain a radiation density a thousand times greater than that obtainable
1^ h e s t a b i l i t y o f t h e f r e q u e n c y in m a s e r s m a d e it p o s s i b l e to c o n s t r u c t tw o s t a n d a r d
i n d e p e n d e n t l y o p e r a t i n g , s y n c h r o n i z e d c l o c k s . T h e s e c l o c k s w e r e u s e d to m e a s u r e t h e
v e l o c i t y of l i g h t in o n e d i r e c t i o n u n d e r t e r r e s t r i a l c o n d i t i o n s a n d t h e r e f o r e to c h e c k the
fundam ental c o n c lu s io n s of the s p e c ia l theory of r e la tiv ity reg ard in g first-order e f fe c ts
( p ro p o r tio n a l to t h e v e l o c i t y v — cB).
We r e c a l l t h a t a il i n t e r f e r e n c e e x p e r i m e n t s s i m i l a r to t h e M i c h e l s o n e x p e r i m e n t i n v o l v e ,
in e f f e c t , o n ly o n e s t a n d a r d c l o c k a n d a r e u s e d to m e a s u r e th e v e l o c i t y of l i g h t o v e r a
c l o s e d p ath . T h u s , o n ly s e c o n d - o r d e r e f f e c t s ( p ro p o r t i o n a l to /32) c o u l d b e d e t e c t e d in
th e se experim ents.
462 R E L A T IV IS T IC QUANTUM M EC H A N ICS
by focusing sunlight), great coherence and extreme sharpness (with telescopic apparatus
a beam of light reaching the Moon would have a diameter of the order of 3 km). The
first laser produced by Maiman was a three-level ruby laser. Chromium atoms, which
are present in ruby (aluminum oxide) as a slight impurity, absorb light from a power
klystron over a wide band in the green and yellow regions. Initially, these atoms give
up (without emitting radiation) part of their energy to the crystal lattice and make a
transition to a metastable state from which dipole transitions are forbidden, and hence
they can be maintained in that state for a comparatively long time (of the order of
several milliseconds).
Fig- 2 7 . 1 1 . E l e c t r o s t a t i c f i e l d o f t h e
c a p a c i t o r in th e s e p a r a t i n g s y s t e m .
S o m e A p p l i c a t i o n s to N u c l e a r P h y s i c s
Chapter 2 8
~ T = [ H# + V"(0 ] m (28.1)
= (28.2)
where the wave function <Ji„ is a solu tion of the unperturbed tim e -
independent wave equation
E n<fn = W i f n, (28.3)
= l n.n. (28.4)
Substituting Eq. (28.2) into Eq. (28.1) and using Eq. (28.3), we
obtain the follow ing equation for the unknown c o efficien ts C„:
~ n En (28.5)
—-E it
M ultiplying Eq. (28.5) by type* , integrating the resu ltin g
e x p r e ssio n over a ll sp a c e , and usin g the orthonorm ality condition
(28.4), we obtain a sy ste m of equations for the unknown c o efficien ts
C n>
— 4 Cn‘
I
^ C„'-e"°Vn" V'n'n" (f), (28.6)
w here
(0n'n" — (28.7)
V'n'n" ( 0 = S W ( t ) y n " d * X .
Ch" = (28.11)
In quantum m echan ics one gen erally calcu lates the transition
probability w per unit tim e. Since the probability of finding a
p article in the state n’ is given by | C„< |3, we obtain the follow ing
ex p ressio n for the tran sition probability:
w = T t l i ^Cn' ^ (28.13)
n'
Equations (28.12) and (28.13) are the b a sis for the investigation
of many quantum -m echanical prob lem s in the fir s t-o r d e r tim e -
dependent perturbation theory.
*»-*?)•
At the instant of tim e q>= 0,
the p a rticle com es within the
range of interactions; that is ,
it m oves in a potential V ( r ) .
The p a rticle now has a finite
probability of m aking a tran
sitio n to the state with the
m omentum p ' = hk' and energy
E'= ctlK! — that is, F ig . 2 8 .1 . S c a tte r in g o f a p a r t ic l e by a c e n
ter of force: Hk is th e m o m en tu m o f th e
the p a rticle is sca ttered as in c id en t p article; ftk is th e m om entum of
a resu lt of interaction (Fig. th e scattered p article; t? is th e angle of
28.1). scatterin g ; 0 d e n o te s th e s c a t te r in g ce n ter.
468 F U N D A M E N T A L S OF N U C L E A R P H Y S I C S
w here L is the period, and the m om entum com ponents £,• and
k ’i (Z= 1 , 2, 3) are rela ted to the in teg ers m and n\ by m ean s of
the e x p r e ssio n s
2nn, 2-r.n',
. (28.15)
Substituting the wave functions (28.14) into Eq. (28.12), we find for
the co efficien t C„-
1 _ e M ( K' - K)
Cn'(t) = - p V H (28.16)
cn(K‘ — K) '
Vx — j e ‘xrV ( r ) d ' x , v. — k — k ’.
s in ct (K1— K)
ji (K‘ — K)
(28.18)
c(\K ' — K \ ~ 2 v .
Since the quantity t — t — t0= M is the tim e ela p sed from the
initial instant, and the quantity c/2 (/<"' — /<^) = AE is the energy
spread resu ltin g from sc a tte rin g , we find a relation sh ip between
th ese quantities
A/| AC |~ A . (28.19)
E L A STIC S C A T T E R IN G OF PA R T IC L E S 469
K' = K.
CO
ct ( K ' — K ) = t .
we get
eK.
71
— ctK
/ = F(/C)-^+J s^ d l F (IQ-
— oo
An e x a m p l e o f i n e l a s t i c s c a t t e r i n g i s p r o v i d e d by b r e m s s t r a h l u n g —a s c a t t e r i n g p r o c e s s
in w h i c h a n e l e c t r o n e m i t s a p h o t o n , s o t h a t K < K .
470 FUNDAM ENTALS OF NUCLEAR PH Y SIC S
w = l j k * y . \ V * \n (K ’ ~ *)• (28.22)
h'
N = h = i ' l <28-24>
With the aid of Eqs. (28.22) - (28.24), we find the follow ing
e x p r e ssio n for the sc a tte rin g c r o s s se ctio n :
o = J = | o ( » , <p)d2. (28.25)
o ( 0 , ?) = ( ^ T) V j 2. (28.26)
Vx = V(r)r*dr [ e ^d W,
0
w here d Q ’ is the so lid angle a sso cia ted with the vector r , w hereas
in Eq. (28.25) d l l is the so lid angle a sso cia ted with the vector A'.
EL A STIC S C A T T E R IN G OF PA R T IC L E S 471
« ( » ) = ! / ( » ) I*. (2 8 -2 7 )
where
x= \k — k'\ = 2 k s \ n \ , (28.27a)
R = - - ~
m T_c
10“ 13 cm . (28.31)' '
we obtain the follow ing e x p r e ssio n for the d ifferen tial c r o s s sectio n
of e la s tic sca tterin g :
/A\ _ 4/zi^2/?1 (28.33)
v ' n< p -V A + u
2A s h a s b e e n m e n t i o n e d in C h a p t e r s 2 5 a n d 2 7 , a m o r e a c c u r a t e a p p r o x i m a t i o n o f t h e
T h o m a s - K e r m i p o t e n t i a l i s g i v e n by t h e e x p r e s s i o n ( 2 5 . 2 2 ) . T h e r e s u l t s o f t h e tw o a p p r o x
i m a t i o n s , h o w e v e r , d o n o l d i f f e r g r e a t l y fr om o n e a n o t h e r ( w h i c h i s a c o n s e q u e n c e o f t h e
short-rJinge c h a r a c t e r of th e forces). The a p p ro x im atio n (2 8 .3 0 ) is m ore convenient in
c a lc u la tio n s of sca tte rin g .
EL A STIC S C A T T E R IN G OF PA R T IC L E S 473
x4 = 4£1sin s ~ 2 = ^ frj s i n
im-AOR*
«(») = n1 (28.34)
o(d) = i g f - . (28.35)
can alw ays find sm a ll angles & such that the follow ing inequality
is true:
sin - < 1. (28.37)
474 FU NDA M ENTALS OF NUCLEAR PH Y SICS
8nmlA‘R* (* sin
J [1 + 2k*R‘ (1 — cos 8)]2
o
_ 16tt/ngA2/?4 1
n* 4PR* + 1• (28.39)
F in ally, with the aid of Eq. (28.29), we can find the range of
applicab ility of the perturbation m ethod for our problem in the two
lim itin g c a s e s con sid ered above
Problem 28.1. Represent the cross section for scattering of particles by a spherically
symmetric potential as a sum of partial cross sections (the sum of the cross sections
for waves with a well-defined value of the orbital quantum number I ). Obtain the scatter
ing cross section for the general case and for the Born approximation.
Solution. Suppose the incident particle has a momentum p = fik and velocity
v = hh!m0, directed along the z axis.
The wave function of the incident particle is of the form of a plane wave
A plane wave can be expanded in spherical harmonics (see Problem 21.5). Then, using
the asymptotic expression for the Bessel function as r —'-co
T.I
t t sin kr —
(kry-
i+- Y \ Ykr
In the presence of the potential energy V(r), the asymptotic expression for the wave func
tion of the particle In a centrally symmetric field, In accordance with Eqs. (13,75) and
(13.78), should be chosen in the form
sin hr — +
CiPt (c os 3)
*as= 1 hr
where the phase shift 6, can be determined from the asymptotic solution of the Schrodinger
equation for the radial function in the presence of the potential V (r)
bh[*
Clearly the scattered wave is
i v i
^scat 't'as ^ inc:
/= 0
The unknown coefficients Cj can be determined from the condition that the function 'J'scat
must be a diverging spherical wave. Thus the coefficient of the converging wave
-iU r-4 )
e ' 2 ' must be equal to zero. Then
/(« ) £tkr
^scat
The function /(9 ) is the scattering amplitude [see (28.28)], which, according to the
exact theory, is equal to
= v l/<8) I2 dQ
to the number of particles incident per unit time on a unit surface perpendicular to their
velocity, that is, perpendicular to the z axis
^ i n c = u4'*inc4'in c = w-
From this we find the differential cross section
Substituting the value obtained for the scattering amplitude and using the orthonormality
property of Legendre polynomials, after integrating over the angles we get
Tt
j Pi ( cos ») Pr ( cos 9) sin 94 9 = )
which gives us the following expression for the total cross section:
oo
0
Next, using the expansion
oo
— = 2f r 2 (2^ ^ ; + ^ ) ^ (cos8).
IMJ ‘ 2
where
= 2li sin ,
the scattering amplitude (28.45a) can be reduced to the form (28.28) found In the Born
approximation.
Problem 28.2. Determine the cross section for scattering of particles by a spherically
symmetric potential barrier of height K0> 0 and radius a
V— I for r < a,
I0 for r > a,
when the radius a is much less than the de Broglie wavelength of the scattered particles,
that Is, when ka I.
Show that in this case the s wave (wave with / = 0) is the main contributor to the
scattering process.
Compare the exact solutions with those obtained by perturbation methods.
Solution. It may be seen from Eq. (28.46) that at ka I, the s wave (/ = 0) is the main
contributor.
Solving the problem by perturbation methods (Born approximation), that is, using
Eq. (28.28), we find the scattering amplitude
E L A STIC S C A T T E R IN G OF PA R T IC L E S 477
«U = 2* J s i n « | / ( 0 ) | * d # = - ? (28.47)
h'
o'
Let us determine the scattering phase shift in this simple problem. We restrict
ourselves to a determination of the phase shift for 1 = 0 and low energies E, when we can
set ka < I.
When I = 0, the wave equations have the form
7.J + **/„ = 0 for r > a,
(28.48)
7.J — *,2/.0 = 0 for r < a,
where
Using the boundary conditions 7.o(0) = 0, the solution of Eq. (28.48) can be represented
as
y _I A sinhx'r for r < a ,
0 — 1 sin (kr + 60) for r > a.
Equating the wave functions and their derivatives at the boundary of the region r = a , we
find In our case of small /; (E <<; K0)
where
2ntoVt
ft2
Hence, according to Eq, (28.45), we have
/tanhxa
o = 4na2 - - 1 (28.49a)
\ xa
In the case
xa < 1 (28.50)
we can set
tanh xa r
xa : 1- 3 (™)2
Then Eq. (28.49) gives an expression for o corresponding to the Born approximation
[see (28.47)].
When xa 1(that is, when V0 —■oo) the cross section reaches Its maximum value
o —: 4na2. (28.51)
2
&& <K rra ,
i s e q u i v a l e n t to t h e c o n d i t i o n ( 2 8 . 2 9 ) fo r t h e a p p l i c a b i l i t y o f p e r t u r b a t i o n m e t h o d .
470 FU N D A M E N TA LS OF NUCLEAR P H Y S IC S
The last expression Is four times greater than the corresponding cross section for
elastic scattering by an impenetrable sphere, calculated according to classical mechanics
when o is determined simply by the geometric cross'section of the sphere tea2. This
discrepancy is due to the appearance of wave properties (diffraction) of the scattered
particles.
Problem 28.3. Determine the cross section for scattering of slow particles (ka<^ 1)
by a spherically symmetric potential well:
V — V„ for r < a ,
0 for r >. a. (28.52)
Indicate what distinguishes the cross section for scattering of particles by a potential
well from the cross section for scattering by a potential barrier.
Solution. In the exact solution of the problem, Eq. (28.48) should be replaced by
k'1 = ^ ( E + V 0) = k2 + xK
The phase shift is given by the following expression [instead of Eq. (28.49)]:
k_
B0 = arc tan tan k ka. (28.54)
k' '*)
When k'a 1 and £ < Vo, we again obtain Eq. (28.47), which was derived in first-
order perturbation theory. In this case the cross sections for scattering by a potential
well and by a potential barrier are identical.
The difference appears when the quandtyxa lx 1 = ^ b e c o m e s comparable to or
• / tanh x a
greater than unity. Thus, in the case of a potential barrier, the quantity -------- - monoton-
x'a
lcally approaches zero as V0 increases, whereas in the case of a potential well the
.tank'd I
corresponding quantity varies periodically over the range from 0 to oo as
Vq increases.
In particular, if the quantity k'a approaches - y , we find the following expression for
k'a 1
the cross section in the region ------- <g|ka:
tan k'a
_a2
(28.55)
°res k2a2'
Because of the smallness of the quantity fc2a2 = ~ ^ a2, the expression for o is much
greater than the maximum value for the cross section In the case of scattering by a
potential barrier (°rnax = 4tta2 when Vo— oo [see (28.51)]). Since the relation k'a —
= Y'*-2a2 -j- k2a2 = — for ka 1 is actually equivalent to the condition of appearance
of the first level In the spherically symmetric potential w ell,x a £ s — , the cross section
(28.55) corresponds to the case of "resonant” scattering. Subsequent resonance maxima
of the cross section occur when xu : and so forth.
Note. Ih e scattering phase shift, together with the expression for the cross section,
can be accurately determined for a very limited class of problems (scattering by a
spherically symmetric potential barrier or potential well, scattering by a potential in
versely proportional to the square of the distance, and so forth). However, in the general
case we must use approximate methods. For intermediate values of the potential energy l/0
(for instance. In the case^of scattering of charged particles by a Coulomb field the constant
EL A STIC S C A T T E R IN G OF PA R T IC L E S 479
2e*
characterizing the potential energy must satisfy the condition —— 1) perturbation
nv
theory (that Is, the Bora approximation) gives good results. In cases when the Bora
approximation Is no longer applicable, the phase can be determined by other approximate
methods, for example, the WKB method [see Eq. (13.78)], the variational method and
so forth. In view of the special nature of these methods we shall not discuss them In this
general treatment.4 It should be noted that many qualitative features, which are char
acteristic of the scattering of particles by various potentials, are very well illustrated
in the scattering by a potential barrier or a potential well.
4
T h e t h e o r y o f s c a t t e r i n g i s g i v e n a m o r e e x t e n d e d t r e a t m e n t in N. M o tt a n d G. M a s s e y ,
The T h e o r y o f A t o m i c C o l l i s i o n s , N e w York: O x fo rd , 1949; L. Schiff, Q u a n t u m M e c h a n i c s ,
N e w Y o r k : M c G r a w - H i l l , 1955*
Chapter 2 9
Second Quantization
(29.1)
= 2 Cn(/)^ -
n n
C„(/) = C ne \ (29.2)
H ere the quantities E n are the energy eig en v a lu es, and the eig en
functions of the tim e-independent Schrodinger equation sa tisfy
the orthonorm ality condition
\ = 8 „„.. (29.3)
,v = J ^ (l).^ (()d'x,
(29.4)
p .= ^ r(t)P A (t)c P x .
SECOND Q U A N TIZ A TIO N 461
dx cl 0/1
dt dp ci ’
■t = (29.5)
H= + (29*6)
P_ (29.7)
W0
px — x p = y . (29.8)
= 2 E nC X ' ( t ) C n ( t )
rt, n'
H = 2 E nC * ( t ) C n (t). (29.10)
n
iEt
(29.11)
C n (t) = C ne C* (/) = C*e
that the tim e factor in the product C£ (/) • C„ (f) is sim p ly equal to
unity
C * ( t ) C n (t) = C nCn.
d- ^ i ! l = L ( H C n( ( ) - C n( t)H). (29.12)
C,'(/) = - y C , / ' ‘ ‘ ,
SECOND Q U A N TIZ A TIO N 403
and therefore from (29.12) we find the follow ing rela tio n , which is
a fundamental postulate of the second quantization:
_ Un€ n. = U C n. - C , , U . (29.13)
C„.C„-C„C„. = 0,
(29.15)
Cn'C% — C nC n>= onn
From Eq. (29.15), it follow s that the co e ffic ien ts Cn and C* are
op erators. Setting
C*„Cn = N, (29.17)
C nC > = \ + N . (29.18)
It follow s, ‘h erefo re, that th ese op erators do not vanish even in the
c a se where there are no p a r tic les p resen t. Indeed, even though
C'nCn = 0 when N = 0 , we s till have in that c a se CnC % = l . This
nonzero value for the com bination of co efficien ts C g iv es r is e to
a relationship between the vacuum (the field of virtual p a rticles)
and the rea l p a r tic le s.
C n'Cn + C nC n’ = 0 ,
r* r i c r * __*
“1“ K-'n'-'n' — °nn' »
v^i7«-LJ7/
C%Cn = N, (29.20)
we find
C nC*n = \ ~ N. (29.21)
Noting that C*Cn and C nC* cannot be n egative, we find that the number
of p a r tic le s in the sta te n can assu m e only two values: N = Q and
N = l. C onsequently, the P au li e x c lu sio n prin cip le is already in
cluded into th is solution and the p a r tic le s obey the F erm i s ta tis tic s .
In p articu lar, if there are no p a r tic le s p resen t in itia lly ( N = Q), then
ju st a s in the c a se of the B ose s ta tis tic s we have
It is w ell known that the photon field (electrom agn etic field) can
be d escrib ed by a vector potential which s a tis fie s the d’A lem berts
equation
= (29.23)
V- i 4 = 0. (29.24)
where
*= 1 * 1. and = («; = 0 , zb 1 , z b 2, z b 3 . . .).
y-a = 0. (29.26)
(29.13) for the field of photons p e r m its only a solu tion correspond ing
to the B ose s t a t is t ic s . 2 F u rtherm ore, taking into account the con
dition (29.24), we obtain the follow ing com m utation relation s:
C. SPONTANEOUS EMISSION
sin ce
\-A = 0,
Eq. (29.31) can be reduced to
<•=<>. (2 9 *3 3 )
e
V' { t )
cm,,
A(t) • p .
= p (29.34)
cm» L /2cm0 * \ li
should be s e t equal to unity. Such tran sition s are called the dipole
tr a n sitio n s .4
3 T a k i n g i n l o a c c o u n t t h e c o m m u t a t i o n r e l a t i o n s in ( 2 9 . 3 0 ) in t h e e x p r e s s i o n f o r t h e
v e c t o r p o t e n t i a l A in t h e c a s e w h e n t h e r e a r e n o p h o t o n s , w e s h o u l d r e t a i n o n l y a m p l i t u d e s
p r o p o r t i o n a l t o o * , t h a t i s , o p e r a t o r s fo r t h e c r e a t i o n o f p a r t i c l e s .
4 I f w e i n c l u d e t h e n e x t t e r m in t h e e x p a n s i o n w e o b t a i n q u a d r u p o l e r a d i a t i o n . Q u a d r u
p l e ra d ia tio n is ( r / A ) ^ lim e s w e a k e r th a n d ip o le r a d ia tio n an d i s of im p o r ta n c e only w hen
d ip o le tr a n s itio n s a re fo rbidden.
488 FUNDAM ENTALS OF NUCLEAR PH Y SIC S
o fi* = 8*.-------.
and include the fact that for su ffic ie n tly large valu es o f tim e t , we
have, in accord ance with (28.20),
s in t(u>-
= 8 (to — « )„ „ -). (29.39)
7t (!) -- (I)n n '
U) — i&nn' (29.40)
sin c e from Eq. (29.40) it follow s that the energy of the em itted
photon hio is equal to the energy ( E n — f v ) lo st by the atom as a
r e su lt of tran sition . For the probability of spontaneous e m is s io n ,
we obtain
* ' « n n ' I P n ' n I*
Ann' 2rAc3m’i
Ann' ---
nn
(29.41)
7lc<r ‘ r„'n
SECOND Q U A N TIZ A TIO N 489
D. BETA DECAY
We n o t e t h a t s i n c e t h e r e s t m a s s o f a n e u t r o n i s g r e a t e r t h a n t h e t o t a l r e s t m a s s o f t h e
proton, e l e c t r o n , a n d a n tin e u tr in o , it f o ll o w s th a t th e d e c a y o f a f r e e n e u tr o n s h o u l d a l s o
b e o b s e r v e d . T h e d e c a y o f a f r e e p r o t o n a p p e a r s i m p o s s i b l e from t h e e n e r g y s t a n d p o i n t a n d
th e refo re, p o s itro n d e c a y c a n b e o b s e r v e d o n ly in a b o und proton, w h en th e re q u ire d e n erg y
c a n b e t a k e n u p fr o m t h e n u c l e u s .
6We s h a l l e x p l a i n t h e d i f f e r e n c e b e t w e e n t h e n e u t r i n o a n d a n t i n e u t r i n o a t t h e e n d o f t h e
p r e s e n t c h a p t e r in t h e d i s c u s s i o n o f t h e n o n c o n s e r v a t i o n o f p a r i t y .
490 FUNDAMENTALS OF NUCLEAR PH Y SIC S
F u rth erm ore, the capture of a bound ele c tr o n is a lso p ossib le; as
a r e su lt of e le c tr o n capture a proton is changed into a neutron and
e m its a neutrino ( p -)- e~ — n v ) . As a rule; an e le c tr o n from the K
sh e ll is absorbed in th is p r o c e s s , and th erefore th is phenomenon is
c a lle d K capture, /(ca p tu re is sim ila r in nature to positron /3 decay,
sin ce in both c a s e s the charge of the nucleus is reduced by unity.
We sh a ll not con sid er h ere the d etails of j3 decay; our task
w ill be to d e sc rib e in gen eral term s the creation of an ele c tr o n
and antineutrino follow ing the F erm i theory.
The energy of in teraction o f a neutron with the e le c tr o n -
antineutrino field can be w ritten as
V e7 = m h (29.42)
w here / is a coupling constant introduced by F erm i. The m agnitude
of f is v ery sm a ll ( / ^ 1.4 x 10“49e r g x cm 3 ) so that this in ter
action is c a lle d a weak in teraction . The spontaneous decay of
p a r tic le s is caused m ainly by weak interaction s; th e re fo r e, the
life tim e o f elem en tary p a r tic le s or nuclei is com p aratively large
and v a r ie s from a fraction of a second to b illion s of y e a r s. N uclear
p r o c e s s e s , on the other hand, are cau sed by stron g in teraction s,
which so m e tim e s are a thousand tim e s la rg e r than the electro m a g
n etic in te r a c tio n s. The duration of the p r o c e s s e s caused by such
in tera ctio n s is v e r y short (of the ord er of 1 0 - 2 3 se c ).
N eglectin g spin e ffe c ts , the wave functions ipj and <p- can be
rep resen ted as
— L - a/ s a * e kK‘- ik ' r ,
(29.43)
(p i = z ,— a/a b * e ic-At-ix ■ r#
If there are no p a r tic le s p resen t at the in itial instant, the follow ing
rela tio n s hold for the am plitudes a* and b *: a * a = b * b = 0, a a * —
— bb* -- 1 .
Then, accord ing to Eq. (28.12), we have the follow ing e x p r e ssio n
for the c o efficien t C ( t ) :
(A + k)-/-~|(A : + x ) - / ? |< 1 ,
If the m atrix elem en t (29.45) does not vanish upon replacem ent
of the exponential by unity, the corresponding tran sition s are said
to be allow ed )3 tra n sitio n s.
We note that the allow ed tra n sitio n s, which correspond to the
dipole tra n sitio n s in the theory of photons, are a sso cia ted with
definite se lectio n r u le s . 7
7In th e c a s e o f v e c t o r i n t e r a c t i o n , t h e m a t r i x e l e m e n t
V p n = f X p X n d 3 x
2 H e 6 -> 3L i 6 + e ~ + V
in w h i c h th e s p i n o f t h e n u c l e u s c h a n g e s by u n i t y ( A J = 1). T h e s p i n o f t h e nucleus
i s e q u a l to z e r o (o n e a l p h a p a r t i c l e p l u s two n e u t r o n s w ith a n t i p a r a l l e l s p i n s ) , w h i l e t h e
s p i n o f t h e 3 L i 6 n u c l e u s i s e q u a l to u n i t y (one a l p h a p a r t i c l e p l u s o n e p ro to n a n d o n e
n e u t r o n w ith p a r a l l e l s p i n s , j u s t a s in d e u t e r i u m ) .
In o r d e r to e x p l a i n t h e s e s e l e c t i o n r u l e s , Gam ow a n d T e l l e r p o i n t e d o u t t h a t w h e n w e
form th e i n t e r a c t i o n e n e r g y o p e r a t o r , w h i c h i s a s c a l a r (V = Vp n V el/), o n e may t a k e a
p r o d u c t o f two v e c t o r s , a s w e l l a s o t h e r r e l a t i v i s t i c a l l y i n v a r i a n t c o m b i n a t i o n s o f t h e four
w a v e f u n c t i o n s . F o r e x a m p l e , w e may t a k e a p r o d u c t o f two p s e u d o v e c t o r s , w h i c h i s a l s o a
re la tiv is tic invariant. T he p se u d o v e c to r interaction
^pn ~ x
l e a d s to th e s e l e c t i o n r u l e s A J = 0, i l . It s h o u l d b e e m p h a s i z e d t h a t for a n u c l e o n at
r e s t , th e f i r s t t h r e e c o m p o n e n t s w ill b e d i f f e r e n t from zero .
In v iew of th e f a c t t h a t th e p s e u d o v e c t o r i n t e r a c t i o n f o r b i d s 0 -> 0 t r a n s i t i o n s , w h i c h
w ere n e v e r t h e l e s s o b s e r v e d e x p e r i m e n t a l l y , a s w e l l a s for s e v e r a l o t h e r r e a s o n s , p r e s e n t -
d ay t h e o r y u s e s a c o m b i n a t i o n o f v e c t o r a n d p s e u d o v e c l o r i n t e r a c t i o n s ( a s in th e F e y n m a n
a n d G ell-M ann v e r s i o n o f t h e th e o r y o f /3 d e c a y ) . T h i s v e r s i o n e n a b l e s u s to e x p l a i n the
b a s i c e x p e r i m e n t a l d a t a o b t a i n e d in t h e i n v e s t i g a t i o n of fi d e c a y .
492 FUNDAMENTALS OF NUCLEAR PHYSICS
F i g . 2 9 .1 . E n e r g y d i s t r i b u t i o n o f e l e c t r o n s in
fi d e c a y ( a c c o r d i n g t o t h e F e r m i t h e o r y ).
E q i s t h e m ax im u m e n e r g y o f t h e /3 sp e c t r u m .
T h e o r ig in of t h e c o o r d i n a t e s y s t e m c o r r e s p o n d s
2
to t h e e n e r g y m g c .
U sing Eq. (29.39) and changing the sum m ation over the m om enta
of e le c tr o n and neutrino to an integration [se e (29.38)], we obtain
an e x p r e ssio n for the probability of j3 decay
j w{E)dE, (29.48)
moc3
w here
One of the fundamental discoveries In the theory of weak Interactions was the dis
covery of the nonconservation of parity by Lee and Yang (1956). This phenomenon gives
ris e to a spatial asymmetry in the spontaneous decay of elem entary particles and, in
particu lar, in nuclear 0 decay.
The nonconservation of parity can be observed experimentally In the following two
phenomena.
1. The asymmetry of the angular distribution of electrons in the 0 decay of nuclei
with an oriented spin (the number of 0 electrons emerging along the direction of nuclear
spin does not equal the number of electrons emerging in the opposite direction).
2. The existence of circu lar polarization (helicity) in the particles formed during
decay (for example, electrons formed in 0 decay or p mesons formed in the decay of
it mesons), even in the case when the decaying system has zero spin.
The phenomenon of nonconservation of parity in 0 decay or in the decay of a n meson
was explained with the help of the theory that assigned a definite circu lar polarization
(helicity) to the neutrino. At one time it was thought that the neutrino (which is formed
in positron 0 decay) and the antineutrino (which is formed in electron 0 decay) were
identical particles (Majorana’s hypothesis). It was suggested that this hypothesis could
be tested in double 0 decay.
If the neutrino and antineutrino were identical, one would expect comparatively
large values for the probability of double 0 decay without the em ission of a neutrino (one
neutron of the nucleus em its an electron and a neutrino, while another neutron em its
an electron and absorbs this neutrino), that is, with the em ission of only two electrons.
If, however, double 0 decay consisted simply of two successive identical 0 decays with
the em ission of two electrons and two antineutrinos, the probability of decay should be
much less. Experiment has confirmed the correctness of the second hypothesis and
clearly demonstrated that the neutrino must be different from the antineutrino.
We note, incidentally, that both particles a re neutral and have a spin 1/2. Physicists,
however, were able to establish a difference between the p articles from phenomena
associated with nonconservation of parity. It has been dem onstrated experimentally that
the asymmetry observed during positron 0 decay of a nucleus with an oriented spin,
when a neutrino is emitted together with a positron, is the rev erse of the asymmetry
that is observed during electron 0 decay, in which an antineutrino is emitted together
with an electron. It was assumed, therefore, that the neutrino differs from the anti-
neutrino by the type of circular polarization. In order to explain the experimental data,
it was necessary to postulate that a neutrino resem bles a photon with left-hand circu lar
polarization, while an antineutrino resem bles a photon with right-hand polarization. The
It s h o u l d b e n o t e d t h a t t h e maxim um o f t h i s c u r v e i s s l i g h t l y s h i f t e d t o w a r d s sm a ll
e n e r g i e s . T h i s a s y m m e tr y i s d u e to t h e f a c t t h a t t h e a n t i n e u t r i n o m a s s i s e q u a l to zero ,
w h i l e th e e l e c t r o n m a s s i s d i f f e r e n t from z e r o . If t h e m a s s of t h e a n t i n e u t r i n o w e r e e q u a l
to t h e e l e c t r o n m a s s , t h i s c u r v e ( n e g l e c t i n g t h e C o u lo m b a t t r a c t i o n of t h e e l e c t r o n a n d th e
n u c l e u s ) w o u ld be s y m m e t r i c a l ; t h a t i s , t h e maxim um w o u ld o c c u r a t t h e p o i n t E / 2 .
g
F o r m o r e d e t a i l s , s e e H. B e th e a n d P . M orrison, E l e m e n t a r y N u c l e a r T h e o r y , N ew York:
J o h n Wiley & Sons, In c., 1958.
494 FUNDAMENTALS OF NUCLEAR PHYSICS
only difference is that the spin of the photon is 1 (in units of H), whereas the spin of
the neutrino is 1/2.
F i g . 29 .2 . H e l i c i t y o f t h e n e u F i g . 2 9 .3 . H e l i c i t y of a le f t-
tr in o and an tin eu trin o . The h a n d e d n e u t r i n o in r i g h t - h a n d
neutrino h a s a left-h an d circ u la r and left-hand coordinate
p o la r iz a tio n and the an tin eu trin o system s.
h a s a rig h t-h an d c ir c u la r p o la r i
zation.
The circu lar (longitudinal) polarization of the neutrino is generally called the helicity.
The neutrino has left-hand circ u lar polarization, or negative helicity. This meaiis
that if a left-handed screw rotates along the direction of polarization, it moves in the
direction of the momentum. The antineutrino, however, has right-hand circ u lar polari
zation, or positive helicity (see Fig. 29.2). In order to conserve helicity when changing
from one Lorentz fram e of reference to another, so that it can be adopted as a character
istic of the neutrino, it is necessary that the re s t m ass of the neutrino be exactly equal
to zero. 10
Several authors describe polarization with the aid of an axial vector, which is per
pendicular to the plane of rotation and has a different direction in the right-hand and
left-hand coordinate system s (see Fig. 29.3). It should be noted, however, that, although
the axial vector J and the polar vector p have different mutual orientations in the right-
hand and left-hand coordinate system s, the helicity is nevertheless conserved; that is,
the helicity of the neutrino is still negative, and only the method of description has
changed.11
Starting with the polarization properties of the neutrino, we shall make an attempt
to give a qualitative explanation of the nonconservation of parity during the spontaneous
decay of particles.
Let us consider, for example, the 0 decay of nuclei with oriented spin. We note that
the spin of the nucleus (its longitudinal polarization) is m ore naturally described by a
rotation, since the direction of the axial spin vector is arb itrary . The spatial asym
m etry which should be observed in the phenomena characterized by the nonconservation
of parity is associated with the fact that in electron 0 decay, right-handed antineutrinos
a re em itted upwards and downwards with spins oriented, respectively, parallel and
antiparallel to the spin of the nucleus. The electrons will be formed predominantly with
a helicity opposite to that of an antineutrino (that is, with a negative helicity). This pro
duces a spatial asym m etry due to which the number of electrons emitted in the direction
of nuclear spin does not equal the number of electrons emitted in the opposite direction
(see Fig. 29.4). In general, we obtain the following equation for the number of emitted
electrons as a function of the angle 0 between the direction of electron momentum and the
upward direction, which two directions form a right-handed system with the polarization
of a nucleus: cn,
wc_(9) = w 0_ ( l ■— a cos 9), (29.50)
where a is positive and is equal to approximately 0.4.
1 ^If th e r e s t m a s s of a p a r t i c l e d i f f e r s from z e r o , t h e p a r a l l e l s p i n a n d m o m entum v e c t o r s
may be d i r e c t e d a t an a n g l e a f t e r t r a n s i t i o n from o n e L o r e n t z fram e o f r e f e r e n c e to an o th e r .
^ S i n c e t h e d i r e c t i o n of t h e ( a x i a l ) s p i n v e c t o r * r e l a t i v e to t h e p o l a r v e c t o r o f t h e m o
m en tu m /> i s d i f f e r e n t in th e r i g h t - h a n d a n d l e f t - h a n d c o o r d i n a t e s y s t e m s , L e e , Y an g , L a n
d au \ Z h u r n a l E k s p e r i m c n t a V n o y i T e o r e t i c h c s k o y F i z i k i , 32, 405 (1957)1, a n d o t h e r s a s s u m e ,
on t h e c o n t r a r y , t h a t in t h i s c a s e t h e n e g a t i v e h e l i c i t y of a n e u t r i n o c h a n g e s to a p o s i t i v e
h e l i e r t y . In o t h e r w o r d s , t h e y a s s u m e t h a t t h e m e th o d of g e o m e t r i c d e s c r i p t i o n of t h e p a r t i
cle can c h a n g e its internal p ro p e rtie s (helicity).
SECOND QUANTIZ ATION 495
A sym m etry was detected In the elec tro n 0 decay of Co 60 nuclei with o rien ted spins
and also In the angular d istribution of ele c tro n s In the 0 decay of fre e , p o larized neutrons
(for which u «= 0.1).
Since a neutrino with a negative hellclty em erges during positron decay, the asym
m etry pattern will be opposite to the one described above. The number of em itted posi
trons Is related to the angle 9 by the equation
that Is, the positrons that are formed have mainly a positive hellclty and are emitted
prim arily upwards. An asymmetry opposite to that of electron decay was observed ex
perim entally In positron 0 decay of Co58 nuclei with oriented spins.
Electron Positron
0 decay 0 decay
F i g . 2 9 .4 . S c h e m a t i c d i a g r a m o f t h e /3 d e c a y
o f n u c l e i w ith o r i e n t e d s p i n s . T h e d i r e c t i o n
o f r o t a t i o n c h a r a c t e r i z e s t h e d i r e c t i o n of t h e
spin of th e p a r tic le s ; p is the p a r tic le
mom entum.
Longitudinal polarization was observed particularly clearly in the spontaneous decay
of pions into muons and a neutrino. Let us choose a coordinate system In which the pion
is at re st and consider the negative plon which decays Into a negative muon and an anti-
neutrino. Since the antineutrino has a positive helicity and the momenta of the muon and
the antineutrino m ust be equal and opposite, we find that the muon also m ust have a
positive helicity. Indeed, only In this case will the total spin of the muon-antineutrino sys
tem be equal to the Initial spin, that is, zero. In the decay of positive pions Into a neutrino
and positive muons, the muons, however, will obviously have a negative helicity (see
Fig. 29.5).
The total longitudinal polarization of the created muons has also been confirmed experi
mentally.
A m ore detailed discussion of the nonconservation of parity lies outside the scope
of this book, and we m ust re fe r the re a d e r to the special literatu re on this subject. 1 2
12S e e t h e p a p e r s in T. D. L e e a n d C. N. Y a n g , N o b e l L e c t u r e s , P h y s i c s , N ew York:
E l s e v i e r P u b l i s h i n g C o m p an y , 1964.
A|>|MMl(lix A
V ector S p a c e s
n
'Pa = E
i=1
a.-Vf
The c o e ffic ie n ts a,, i = 1, 2, . . . , n are com p lex num bers that char
a c te r iz e the v e c to r *PQ in the b a s is *Pi, ^ 2 , . . . , 'Pn. If a,- and 6 ,-,
i = 1, *2, . . . , n are the c o m p o n e n t s of the v e c to r s 'Pa = Sa,-*Pf and
*Pb = then c,- = a, + 6 ; are the com ponents of the v ecto r *PC =
S c /'P , = 'Pq + 'Pb and za; are the com ponents of the v e c to r z V a,
w here 2 is a com p lex num ber.
The g eo m etric concept which is the ^ -d im en sion al g e n e r a liza
tion of the notion of “ lin e s and plan es p assin g through the o r ig in ,”
can be developed with the follow ing definition:
A su b set 911 of a v e c to r sp ace is ca lled a l i n e a r m a n i f o l d if it
contains a ll the lin ea r com binations yi'Pi + >>2 ^ 2 + • • • + y ^ k along
with any fe(fe = 1 , 2 , . . . ) of its e le m en ts 'Pi, ¥ 2 , . . . , *P*. A lterna
tiv ely , if 21 is an arb itrary se t of v e c to r s containing the d istin ct
v e c to r s 'Pi, *P2 , . . . , 'Pft, then the s e t of a ll lin ea r com binations
xiV i + X2 ^ 2 + • • • + xft'Pfc (with arb itrary com p lex num bers xi, X2 ,
. . . , xk) is a lin ear m anifold 911. 2JI is c a lled the “ lin ear m anifold
spanned by 21.” If the v e c to r s 'Pi, ^ 2 , . . . . *P* are lin ea rly inde
pendent, the m anifold is fe-dim ensional.
The utility of the concept of a lin ea r m anifold is that it is the
dom ain of definition of a lin ea r op erator. In g en eral an operator
is a mapping o r co rresp on d en ce from a dom ain c o n sistin g of c e r
tain points in the v e c to r space into a range c o n sistin g of certa in
other points, denoted sy m b o lica lly by 'P -> R(*P). In quantum m ech
a n ics we are concern ed with l i n e a r o p e r a t o r s for which by defini
tion
-™ < x < then there are v e c to r s (functions) </» for which x</> or
dt/j/Ox are not square integrablc.
There is a “ projection property” a sso cia ted with a linear
manifold: any vector 'I' may be reso lv ed into the sum of two v e c
tors 'I* 'I'll i M'i, w here lI'|| is in the manifold ©2 (i.e ., there are
com plex num bers cq, c 2, • • • ><'k such that 'I' |) - c i ’l'i + c 2 'I' 2 * • • • i
*'*ll,*» where 'IV ll'2, . . . . M'* span the ©2 ) and'I'j is en tirely outside
©2. As a notational device we may define the lin ear operator
as follow s: *1' \\ = for all 'I'. The domain of definition of Pgjj is
the entire sp ace, while the range c o n s is ts of ©land the null vecto r.
If i lie s outside ©I, = Vnuii- If yV2 lie s wholy within©!, P ^ ^ =
'J'2. If *•' Is an arbitrary v ector, lie s en tirely within © 2 and
Pjjj2 V = PiW(Pgn'I') = Pgji1!'. Since this relation holds for all we may
a s s e r t the operator equation
O perators
if SI and © are sim ultan eou sly m easu rab le. T h ese definitions pro
vide a “ p h y sica l” construction of polynom ials of se v e r a l sim u l
taneously m easurable ob servab les SI, ©, e tc .
In quantum m echan ics the m athem atical idealization of an ob
servab le is a lin ear operator on a H ilbert sp a ce. Let us fir s t out
line the purely algeb raic a sp ects of op erators which can be defined
without r e fe r en ce to the space on which they act. An a b s t r a c t o p
e r a t o r a l g e b r a is a co llectio n of elem en ts called op erators which
together with com p lex num bers are endowed with the stru ctu re of
a v ecto r space (in a techn ical s e n s e , not to be confused with the
H ilbert space) and in addition an operator product. E xp licitly,
500 A PP E N D IX A
T iT j = £ C*Tk ,
k - 1
Inner P r o d u c t
V = E IW -.V )
i= 1
The projection operator onto the ray or one dim ensional manifold
defined by ¥; is P ,¥ = ¥ ,( ¥ ,,¥ ) . If ¥ i , ¥ 2 , . . . is an orthonorm al set
the projection operators are related to the v e c to r s by the equations
P,¥*. = S i k ^ k - The orthonorm ality property is P/P^ = S^P^ and the
co m p leten ess relation is 2 P,- = 1 , the unit operator.
Let ¥ j , ¥ 2 , . . . be the eigenfunctions of an operator M c o r r e
sponding to the eigenvalues , . . . . For sim p licity , let us
assum e the eigen valu es are d istin ct, M] j M'k for i -/ k; then (¥,-, ¥ ft) = 0
and the v e c to r s may be n orm alized so they form an orthonorm al
502 A P P E N D IX A
P g j ll ^ = E I % ><%• I'P>
i= 1
for an arb itrary ket | *P >. Since | *P > is arb itrary it is a p e r m is
sib le notational d ev ice to om it w riting the | *P >. Then the e x p r e s
sion for a p rojection operator in the new notation is
Pm = E l^ x ^ l
i= )
w here the sum runs over the v e c to r s *Pi, V2 , . . . , *P„ which span
the m anifold 912. An arbitrary operator A may be e x p r e sse d in
ter m s of its m atrix elem en ts A ik = (<!>,-,AO*) in a com p lete ortho-
norm al b a sis . . . by the notation
A = E
i,k
A ik I't’i X l ' t l
in the notation: the ket vector | <l>, > can be abbreviated to read | i >.
The Hilbert space for a sp ecific physical problem is built on the
cononical coordin ates q v q 2 , ■ ■ ■ Q>t f ° r a sYs tem of fc-degrees of
freedom . The abstract v e c to r s have no num erical sign ifican ce,
rather the q ’s se r v e to label what w ill be con sid ered a com plete
orthonorm al se t, yVqv q 2 .......... Qk • The ket w i l 1 be denoted by \<iv
q 2 , . . . , q k >. In the (im proper) 8 -function norm alization schem e
the orthonorm ality relation is
ao
then taking the inner product using the distributive rule under the
integral. The r e su lt is
504 A PPE N D IX A
ab — r \ v * > dx < I .
•'a
An arb itrary operator M is e x p r e sse d in term s of its x -sp a ce
m atrix e le m en ts <x | M| y > = by the form ula
y* CD 00
M = / / <x|M|y> l ^ x Vyldxtfy
• /-0 0 fc /-C 0
/
* 00 y * CD
y«00 00
The kinetic energy operator T has the m atrix elem en t < x | T | y > =
2
_ y )t which is a lso regarded as a lo ca l op erator. With
2m
the help of in teg ra tio n -b y -p a rts the expectation value of T can be
w ritten
2 r™
T / </>(x)* V ^(£(x)r/x ,
2m -m
q k )\ d q j • • • d q h
2
'A/?i • • • q k )* • • • ,q k > d qd1q h
The p rojection operator a sso cia ted with the (rectangular) volum e
V is f 3 1 (/ 1 ) P 2 (/2) • • • Pfe(/ft) and the in tegral sp ecifyin g the prob
ability of finding the p a rticle d escrib ed by 0 in V may be reduced
to the e x p r e ssio n
| | P 1 ( / 1) P 2 ( / 2 ) • • ■ P k < / * ) * II2
II PEn«>||2 .
II P U e )^ \\2 ,
w here P (/g) is the p rojection op erator a sso cia ted with the interval
P (/e) = E
E '< E n <E"
p En -
A sse r tio n s (1) and (2) may now be unified in what is c a lled “ the
m ost g en era l probability a sse r tio n p o s s ib le ” :
(e) The total probability W ranges over the valu es 0 < W < 1
for norm alized O.
A ssertio n (3) at the beginning of th is appendix, can be e x p r e sse d
in term s of the inner product. If we introduce the notation Exp 181,01
to stand for the statem ent “ the expectation value of the physical
observable 81 in the state 0 ,” a sse r tio n (3) may be stated
E(A) = E p Uj)
\j< X
(co n v ersely , P ( l j ) = E(AJ+i) - E(A;)) and le t the n -►~ and the m esh
s iz e vanish, the sum approaches the S tieltjes integral
/ •ao
E(A)(<ME(A)<D) ,
,O
D
which by definition of a function of an operator is (0, F(A)Q>). So
far we have indicated that a s s e r tio n s (3) and (3/ ) follow from the
S ta tistica l P ostu late. What is rem arkable is that the c o n v e r se is
a lso true: the S ta tistica l P ostu late fo llo w s from a s s e r tio n (3). The
proof of th is statem en t is obtained by a techn ical application of the
follow ing theorem : Let A \ , A 2 , . . . , A m be a s e t of m utually com
muting op erators; there e x is t s an operator R and functions E^A),
E2 (A), . . . ,E m(A) such that Ai = Ei(R ) , / \ 2 = E2 (R), . . . , F m = F m ( R ) .
In 'the p reced ing d isc u ssio n we have assu m ed that the state is
a pure sta te, that is , it is d e sc rib e d by sin g le v e c to r 0 . In gen era l,
in the p h ysical preparation of a state som e of the v a ria b les are
left uncontrolled and the state is not com p letely sp e c ifie d . This
situation can be form ulated in term s of a c la s s ic a l probability
distribution that is su p erim p osed on top of the quantum m echanical
uncertainty. L et us suppose that the sy ste m is in one of the sta te s
'Pa., a = 1,2,. . . , but we don’t know p r e c is e ly which one. L et Wa be
the c la s s ic a l probability that the sy ste m is in the state *i'a. The
b a sic sta tistic a l postulate can now be reform ulated as
The s ta te s M'a are an arb itrary s e t of sta tes; they need not be
m utually orthogonal. F or the Wa ’s it is required that Wa > 0 and
1.
The s ta tis tic a l a s s e r tio n s can a lso be form ulated in term s of
the tra c e . The trace of an operator A is
lr A E ( ' l ’n ^ > ,
and
P = £ wa l \ = E l ^ ^<¥*1
a a
The density m atrix is useful for making a gen eral statem en t about
the tim e evolution of the sy ste m . Expectation values change with
tim e according to the gen eral rule
— Expt a ; Wa , V J = tr \ p A + PA I.
dt
C hapter 1
C hapter 2
M 0 c 2 ► J /e x 2 .
C hapter 3
3.1 C alculate the d eB roglie wave lengths of the follow ing par
tic le s each with a kinetic energy 500 keV: photon, electron ,
proton, and alpha p a r tic le s. A lso of therm al oxygen atom s at
300°K.
3.2 What is the d isp ersion law = co(k) if the group v elo city is
in v ersely proportional to the phase v e lo city ? How is this
c a se r e a lize d physically?
3.3 Com pare the reflectio n of a p article and a wave from a
moving su rfa ce. Show that AE/Aco = A p / A k , w here AE(Ap) is
the change in the energy (momentum) of the p article and
Aai(Ak) is the change in frequency (wave number) of the w ave.
Show this independently of the d isp ersion law of the wave and
the energy-m om entum relation of the p a rticle. How can this
r e su lt be gen eralized and what is its sign ifican ce?
3.4 C alculate the d eflection of a charged p article by a thin slab of
m agnetic field , ex p ressin g the change of m omentum in term s
of the v ecto r potential. How is the difflection explained in
term s of the wave picture? What is the relation betw een mo
mentum and wave vecto r in the p resen ce of a m agnetic field?
A nsw er. SK = 0 a c r o ss the slab and p + Ae / c = £k.
3.5 C onsider a p article bouncing back and forth in a rigid box of
lin ear dim ension L. What is the m inim um m easurable ki
netic energy of the p article? How much energy is required
to con strain an electro n to rem ain within a volum e of nu
c le a r s iz e ?
3.6 D eterm ine the m axim um tim e a free p article w ill rem ain
within a volum e of radius R by con sid erin g the lim itation s on
the sp ecifica tio n of the initial data in the c la s s ic a l d escrip
tion of m otion.
3.7 Find an uncertainty relation connecting angular m omentum
and angular orientation. C onsider a rigid rotator. What is
the m inim um uncertainty in L?
3.8 One b illia rd ball bounces on another with a c e n te r -to -c e n te r
height of ten tim es the radius. What is the optim al horizontal
514 PROBLEM S
lo ca liza tio n to m axim ize the num ber of bounces? What is the
m axim um num ber of bounces?
A nsw er, n ~ 50.
3.9 What is the optim al lo ca liz a tio n of an ideal pendulum to
m axim ize the tim e it w ill rem ain balanced in an inverted
p osition and what is the m axim um tim e?
A n sw er. About s ix tim e s the period for sm a ll o sc illa tio n s.
C hapter 4
for any in terval a < x < b. Show that if ip 1 and \p2 are solu
tions for the sam e potential and if E 2 > E i, then \p2 has at
le a s t one node betw een each pair of con secu tive nodes of
Over the en tire in terval, then, tp2 has at le a s t one m ore node
than ipy. (The eigenfunction s are ordered in energy according
to the num ber of n od es. A s e t of eigenfunctions is com p lete
if th ere is one for each in tegral num ber of nodes.)
4.6 (C om parison potential for d is c r e te sta tes) C onsider the in
terval 0 < x < 00 with the boundary condition IKO) = 0 . Find
the potential for which U = x n is a zero energy solu tion of the
Schrodinger equation. Show that the potential obtained has no
d isc r e te eigenfunctions for n r e a l. If n is com p lex, n =i +
i f } , show that the potential has a rb itrarily many bound (i.e .,
- d iscr e te) sta te s . The solu tion may be taken to be rea l U =
Vx cos ft lnx.
PROBLEMS 51 5
C hapter 5
C hapter 6
6.1 Find the probability for a p a rticle of charge + Z'eo and speed
v to penetrate through the Coulomb b a r r ie r to a nucleus of
charge Zeo. (Gamow factor)
6.2 D erive (6.35), using in tegration by parts to pick out term s of
ord er T2 in e!T„.
dv
6.3 Find the p r e ssu r e exerted by the w a lls to contain a p article
in a v e r y deep potential w ell of volum e V at absolute zero
tem p era tu res.
6.4 Find a rela tio n betw een the volum e and p r e ssu r e of an e le c
tron g a s at 0 °K.
6.5 Show that (6 . 6 6 ) holds for an arbitrary period ic potential
(B lock ’s th eorem ).
C hapter 7
— = i- (Rx - xR)
dpx h
C hapter 8
8.1 Show that the com m utation rela tio n s betw een the op erators
for the v e lo c ity com ponents of a charged p a rticle in a m ag
n etic field are v x vy - vy v x = ^ - H z and c y c lic perm utations.
ml c
8.2 Show that if V,- and are energy eigenfunctions and if M does
not depend ex p lic itly on tim e, then the tim e dependence of o ff-
diagonal m atrix e le m en ts is given b y — ('Pi, M'F*) = faji*(^t-,MW*)
dl
w here fay,-* = E,- - E j .
8.3 Let U ( t ) be the operator that g e n e r a te s the solu tion *P(f) of the
tim e-depend en t Schrodinger equation (with H am iltonian H)
from arb itrary in itial data 'P(O) in the form WU) = U ( t ) ¥(0).
See problem 7.8.
(a) Show that 1/(0 s a tis fie s the operator equation i H U U ) =
H U it), with in itial data 1/( 0 ) = 1 .
(b) Deduce from the d ifferen tia l equation that U U ) is unitary.
(c) Specify under what conditions and in what s e n s e U U ) may
be w ritten as U U ) = exp ( - i H t / H ) .
8.4 In the Schrodinger pictu re o b serv a b les are rep resen ted by
op erators that do not depend e x p lic itly on tim e, the change of
dynam ic v a r ia b le s with tim e being d escrib ed by the change of
state v e c to r '{'(/). In the H eisenberg picture the tim e depend
ence is tra n sferred to the op erators th e m se lv e s by m eans of
the unitary operator in problem 8.2. The procedure is MU) =
(yi'(0 >M'l, (<)) = (Vq,)U(OV0), w here Vq = 'F(0 ) is the state vector
at / = 0 and w here 171(f) = U U ) ~ l M U U ) .
(a) Show that/7ilil= ftlW - Kft, w here U = U U ) ~ Y H U U ) . If H i s
independent of tim e H = H. T his is the H eisenberg equa
tion of m otion.
(b) If AB - BA = iC, th en QB - I3Q = iC.
(c) D erive an e x p r e ssio n for the tim e d erivative of the op
era to r product CL(/) B (/).
(d) Find the p osition operator in the H einsenberg picture for
„ the m otion of a fr e e p a r tic le. E x p ress the r e su lt in the
coordin ate rep resen tation .
PROBLEM S 519
(e) Using the position operator in part (d), find the tim e de
pendence of the variance of the position of a fre e ly moving
p a rticle.
8.5 Show that the mean value of the kinetic energy in a state be
longing to the d iscr e te spectrum is related to the mean value
of the potential energy by the relation 2 T = (r • W ). If V is a
hom ogeneous function of d egree a, V(Kr) = A“ V( r ) , then 2 T = a V
(virlal theorem ).
8 .6 V erify the follow ing “ sum r u le s .”
(a) E Ir, * | 2 = (?>* - (V*, r2 Vfc)
^
E
2
(e) E
I
V*k I Uk I2 = diverges
H ere Ticjik = E ; - E*. Note that &>,-* > 0 if k d esign ates the
ground sta te. Sum r u les are im portant becau se they provide
a m eans of testin g the p h ysical content of the theory even
though the m athem atical problem cannot be solved com
p letely.
C hapter 9
C hapter 10
a = \J(ma>/2~h) x + \J(Ti/2mtS) d / d x ,
a+ = V(ma)/2 ^) x - \J(1i/2mco) d / d x .
Ha - aH = - a
Ha + - a + H = a
Chapter 11
11.1 P rove the orthogonality and norm alization condition of L e
gendre polynom ials using the generating function (1 - 2 rx +
r2)~'h = £ rn Pn (x).
n= 0
11.2 D erive the com m utation relation s betw een the com ponents of
angular m om entum and the com ponents of (a) the position
operator, and (b) the momentum operator,
11.3 Let A x , A y and A z sa tisfy the com m utation relation s Lx A y -
Ay L x = i~HAz , L y A x - A x L y = - i H A z , LXA X - A x L x = 0, and c y c lic
perm utations, with r e sp ec t to the angular m om entum oper
a to rs. Such operators are ca lled “ vector o p e r a to r s” ; e x
am ples are r and p. P rove the follow ing relation s:
(a) L XA 2 - A 2 L x = 0
(b) L X(A • L) - (A • L) L x = 0
(c) L/2 A x — A X L 2 — i H ( A y L z + L/Z Ay — A z Liy — Ly A z )
(d) L2 (L 2 A x - A X L 2 ) - ( L 2 A* - AXL 2 ) L 2 =
2 ( L 2 A x + A X L 2 ) - 4 L X( A - L )
(e) M + M A )?Z = (L • * > - > ) £ ' .
11.4 Show that in a state ^ with a sharp value of L2, L z <p = m<p, the
m ean valu es of L x and L y are zero.
11.5 Suppose a sy ste m can be r e so lv e d into two weakly interacting
su b sy stem s 1 and 2 so that the total angular momentum L is
Li + L2 . If the su b sy stem s are in sta te s ch a ra cterized by
definite valu es of the quantum num bers / x, l l z and I2 , h x re
sp ectiv ely , what j^re the p o ssib le valu es of L2 and what is the
average value of L2?
11.6 E xp ress the sp h erical harm onics for 1 = 0 , 1 and 2 as poly
n om ials in x, y and 2 .
11.7 Find the transform ation rule of the sp h erical functions Yn>
Y1 0 a n d Y i- ifo r a rotation of the coordinate sy ste m through
Eulerian an gles a, fi and y.
522 PROBLEMS
+
+
+ 2
1
(c) L z L± - l ±l z = ±~HL±
(d) L+L- - L . L+ = 2 H L Z
(e) L2 L± " L ±L \ = 0
(f) l 2 l 2 - L Z L 2 = 0
11.9 Study the way in which the m ultiplication p ro p erties in prob
lem 1 1 . 8 can be rep resen ted in te r m s of m a tr ic e s, or what is
the sam e thing, lin e a r tran sform ation s on a fin ite dim ensional
v e c to r sp a c e . Let 'Pxm be a fin ite s e t of d egenerate eig en
functions of L2 with eigenvalue A. H ere n la b e ls the elem en ts
of the s e t. Show the follow ing:
(a) L z is an eigenfunction of L2.
(b) T here is som e lin ea r com bination of t h e n ’s which is an
eigenfunction of L z . Denote the eigenvalue by m t and the
p articu lar lin ea r com bination by ¥xm.
(c) L+^Xm is an eigenfunction of L2 with eigenvalue A.
(d) L±'i'Xm is an eigenfunction of L z with eigenvalue (m ± l ) t .
(e) F rom the condition that the norm of is nonnegative,
it fo llo w s (r e sp e c tiv e ly for +) that A > m( m ± l)7j2, the
equality holding if the v e c to r is null.
(f) The condition for the e x iste n c e of fin ite m u ltip letes is
that the s e r ie s gen erated by (L±)n term in a tes at both ends.
T h is im p lies that A has the form 1(1 + D U 2 , w here / may
have the v a lu es 0, 1 /2 , 1, 3 /2 , . . . , and that m runs over
range — /, — I + 1 , . . . , / — 1 , I.
(g) Find the m a tr ice s of order 2 1 + 1 that r e p r ese n t the action
of the op era to rs L z and L± on the v ecto r sp ace spanned by
, - I < m < I, for a fixed value of I, F or exam ple,
OP,/ m ,’ L,z V,Im ') = (L z )mm ' = m S mm S e lec t a rphase convention
com p atiable with ( 1 1 . 8 8 ).
(h) W rite out the e x p lic it m a tr ic e s for I = 1 /2 and v e r ify the
com m utation rela tio n s by d ir e c t m atrix m ultiplication.
Chapter 12
12.1 Find the en ergy le v e ls and eigenfunctions of an isotrop ic
th ree-d im en sio n a l harm onic o s c illa to r . Find energy eig en
functions that are a lso eigenfunctions of L2 and L z . What is
the d egen eracy of each lev e l? What part of the degeneracy
ste m s from isotrop y of the potential?
12.2 W rite out the e x p licit I = 1, m = 1, 0, - 1, eigenfunction s of
lo w est energy of a three dim ensional sp h e r ic a lly sym m etric
harm onic o s c illa to r . E xp ress th ese eigenfunction s a s lin ear
-co m b in a tio n s of the solu tion obtained by using sp earation of
v a r ia b le s in C artesian co o rd in a tes. If each trip let of wave
PROBLEMS 523
2 , / ,2 / ( / + 1)
nl + + I k - -----r — In = 0
h (p) ^ 7 s i n (p - y )
nz(p) ~ - j c o s (p - y )
(e) behavior at p = 0 ,
524 PROBLEMS
(2/)! -I- 1
n {(p) = - p
211\
1 / ' +1
i l j L(p) = — / P l ( x ) e l p x dx
2 y_!
12.6 Find the conditions under w hich the sp h erica l potential w ell
V = - |V 0 | fo r 0 < r < a, and V = 0 for r > a , can support N
s -w ave bound s ta te s , but not N 4 1,
12.7 In the lim it of zero kinetic en ergy, an s-w a v e phase shift
b ehaves a s, S0 (fe) = - k a , w here a is the “ sc a tte rin g len gth .”
Find an approxim ate relation betw een sca tterin g length and
the en ergy of a lo o se ly bound sta te, Eg = 0 .
Chapter 13
13.1 Find the m om entum d istribution of an e le c tr o n in the Is, 2s
and 2 p sta te s of hydrogen.
13.2 C alculate the life tim e of a hydrogen atom in the 2p sta te.
13.3 Give an e x p r e ssio n for the tran sition r a te s betw een con secu
tive c ir c u la r orb its in a hydrogen -lik e atom .
-1 ® 1
13.4 Show that (1 - t) expixi/(l - t )I = ^!) 1 LJ^(x)/fe is the gen-
ft= 0
eratin g function for L egu erre polynom ials L ^ ( x ) = Q®( x) .
13.5 At what quantum le v e l is a m uonic atom the sam e s iz e as a
norm al hydrogen atom in its ground state?
13.6 What is the probability of finding an e le c tr o n with quantum
num bers n , I inside a nucleus of radius R n ?
13.7 Show that if the energy is r e g a r d e d a sa com p lex v ariab le, the
Coulomb “ sca tterin g am plitude” for a definite value of /,
/ 2(§ ® \
' - l J / 2 ik, has a sim p le pole sin gu larity at the energy
value corresp on d in g to each le v e l of the d is c r e te spectrum .
Chapter 14
14.1 Evaluate the shift in energy le v e ls of a harm onic o sc illa to r
produced by a perturbing 5-function potential that is cen trally
located . State the lim it of valid ity of the approxim ation.
1 4 .2 , C alculate the lo w est order effect on the sp ectru m of a lin ear
harm onic o s c illa to r due to the r e la tiv istic in c r e a se in m a ss
PROBLEMS 525
w here
for I = 0 .
(a) Evaluate g Ei ( r , r ' ) for I > 0.
(b) What is the relation betw een the wave functions and b e
tw een the G reen’s functions of the th ree-d im en sio n a l
(problem 14.5) and the partial w ave form u lation s. It is
n e c e s s a r y to d istin gu ish ingoing and outgoing wave bound
ary conditions e ± l k r / r . The addition form ula for sp h erical
harm on ics is helpful,
Chapter 15
15.1 Identify the conditions under which the K lein-G ordan equation
red u ces to the Schrodinger. How does the in itial data prob
lem for the secon d order d ifferen tial equation reduce to that
of the fir s t order equation?
15.2 Show that the fu n ction -sp ace inner product
is independent of tim e if tfj and <£ are solu tion s of the K lein-
-Gordan equation. Find the form of the inner product in mo
m entum sp a ce. Is the Inner product p o sitiv e definite?
PROBLEMS 527
X fjx) f j x ' f
1 f i k i x -x ) k
a (2T7-)3 J
where co = o > ( k ) = c \ / f e 2 + m 2 c t y f t 2 and where the Minkowski
space inner product is k( x - x ' ) = a >(< - l') - k • ( x - x ' ) .
Chapter 16
16.1 What is the value of the g factor of a particle described by
the Hamiltonian (l/2mo) [V • (p - ( e / c ) A)J ?
16.2 A particle of spin 1/2 and magnetic moment p m oves in a
precessin g magnetic field
Hx = U sin0 cos cot , Hy = U sin0 sinca<, Hz = U cos 6 .
At time t = 0, the spin is parallel to the 2 axis. What is the
probability of the spin being antiparallel to the 2 axis at som e
later time?
16.3 Find the (time-dependent) position operators in the Heisen
berg representation of a particle of spin 1/2 and magnetic
moment p moving in a nonhomogeneous magnetic field
Hx = 0, Hy = - ky, Hz = H0 + kz .
16.4 Determine the energy spectrum and wave functions of a charged
particle moving in uniform electric and magnetic fields that
are perpendicular to one another.
Chapter 17
17.1 Show that the four m atrices d ' , a ' ) are Hermitian, linearly
independent and form a com plete b a sis for 2 x 2 m atrices.
17.2 How many m atrices are required to form a com plete set of
Hermitian, linearly independent N x N m atrices? How many
528 PROBLEMS
mutually commuting, Hermitian, linearly independent N x N
m atrices are there?
17.3 In the Dirac theory, a and p are mutually commuting linear
operators. Develop a notation for the linear vector space on
which the operators act. Find an expression for the inner
product in the com posite (direct product) space. Show that
the Dirac Hamiltonian is Hermitian.
17.4 U se the Heisenberg equation of motion to show
(a) .^-(x + n p3 a/2m c) = pg (p - (e/c) \)/m ,
(b) — (t + i h p 3/2 m c 2) = p 3 (H - e ^ l / m c 2 .
dt
17.5 What transform ation of the spinor wave function m ust be made
to resto re the form of the Dirac equation after a change of
gauge in the electrom agnetic potentials?
17.6 Show that the form of the Dirac equation rem ains unchanged
by Lorentz transform ation or spatial rotation.
Chapter 18
18.1 Suppose a system co n sists of two weakly interacting sub
sy stem s each of spin 1 /2 . The total spin is S = si + sg. What
are the possible eigenvalues of S2 and S2? Compute the value
of si • S2 in the triplet (spins parallel) and singlet (spins
antiparallel) states of the com posite system . Find eigenfunc
tions of S2 and S z as linear combinations of products of eigen
functions of the subsystem s.
18.2 An electron moving in a central field of force is in a state
specified by the quantum numbers I j m j . What are the possible
values of the 2 components of orbital and spin angular mo
mentum and what is the average value of each?
18.3 Is the parity operator / a linear operator? Is it Hermitian?
What are the commutation properties of I with the operators
r, p, L, S and J ?
18.4 An axial vector transform s as a vector under proper rota
tions (i.e ., rotations without space inversion) but does not
change sign under inversion. C lassify the following as being
either vector or axial vector: E, II, A, S, L, p and u x v
where u and v are v ecto rs.
18.5 Show that if a system is in a state characterized by a sharp
value m j for the 2 component of the total angular momentum,
the mean value of the total angular momentum about an axis
2 " making an angle 0 with the 2 axis is costf.
18.6 Evaluate the particle flux of positive energy and negative
energy plane wave solutions of the Dirac equation. Also ca l
culate the flux of the corresponding charge conjugate solutions.
18.7 .Can the charge conjugation transform ation be represented by
a linear operator?
PROOLEM S 529
Chapter 19
19.1 Evaluate p and ,j for a Dirac p article in an electrom agn etic
field. V erify that the Lorentz force is given c o r r e c tly .
19.2 C alculate the sp in -orb it and contact potentials for an e le c
tron outside a c lo se d atom ic core con sistin g of a hydrogen-
like atom .
Chapter 20
20.1 C alculate the energy lev e l splitting of a hydrogen atom in a
weak e le c tr ic field . N eglect the Lamb shift and assu m e the
Stark effe c t is sm a ll in com parison with the fine stru ctu re.
Account for the latter by using eigenfunctions of J2, J z andL 2
for the unperturbed s ta te s.
20.2 Evaluate the Stark e ffect for the n = 2, j = ^ le v e l of hydrogen
for the c a se w here the Stark effect is com parable with the
Lamb sh ift. R epresent the Lamb shift by a phenom enological
perturbation m atrix elem en t that touches only the s -s t a te .
20.3 Study the hydrogen Stark e ffect for the le v e l n = 2 in the
tran sition region where the Stark e ffect and the fine stru ctu re
are of the sam e order of m agnitude. P lot the energy le v e ls
as a function of S.
20.4 Compute the life tim e s of the 2pi, and 2pi, sta te s of hydrogen.
'2 '2
Chapter 21
21.1 D eterm ine the form of the contact interaction when the finite
nuclear s iz e is taken into account. C alculate the splitting of
the 2 s i - 2 p i le v e ls of hydrogen stem m ing from this effect.
It is about - 0.1 Me.
530 PROBLEMS
Chapter 22
22.1 Spfecify a com p lete se t of com m uting con stan ts of the m otion
for positronium .
22.2 C alculate the fine stru ctu re of positron iu m . Obtain the
H am iltonian by s e m ic la s s ic a l argum en ts. Account e sp e c ia lly
for “ hyp erfin e” sp littin g, knowing that the m agnetic m om ent
of the positron is equal in m agnitude and opposite in sign to
that of the e le c tr o n . P r e sen t the r e su lts in an energy le v e l
diagram for n = 1 , 2 .
22.3 Evaluate the Lamb sh ift for positronium .
22.4 C onstruct a theory for positronium using a D irac Ham iltonian
for each p a rticle and the Coulomb in teraction betw een them .
Separate the m otion of the cen ter of m a ss from the rela tiv e
m otion in the approxim ation of retaining only the low est order
r e la tiv is tic c o r r e c tio n s for the r e la tiv e m otion. D oes the
“ h yp erfin e” in teraction em e r g e autom atically? Hint: The
action of spin o p erators on a product wave function can be
e x p r e ss e d as follow s:
Chapter 23
23.1 C onstruct a com p lete s e t of orthonorm al tw o -p a rticle eigen
functions from a com p lete s e t of o n e -p a r tic le eigenfunction s.
23.2 D erive the H a rtree-F o ck equations for determ ining the b est
s in g le -p a r tic le functions to give an an tisym m etric (or sy m
m etric) tw o -p a rticle wave function of low est en ergy.
23.3 C alculate the n 1 and 2 le v e ls of a h yd rogen -lik e atom , in
c lu d in g fine stru ctu re and the Lamb shift, using a variational
method.
PROBLEMS 531
Chapter 24
24.1 Give the p o ssib le valu es of the total angular momentum for
the follow ing sta tes (term s): 1 S, 2 S, 3 S, 2 P, 3 P, 2D and 4 D.
24.2 Which ter m s are p o ssib le for the follow ing tw o -electro n con
figurations: (a) nsn's, (b) nsn'p, (c) nsn'd, and (d) np, n'p?
Which ter m s are c o n sisten t with the exclu sion principle if
n = n'?
24.3 Couple three unit angular m omenta /i = 1% = I3 = 1 to y ield a
resu ltan t eig en sta te L = 1, L2 = 0 of L2, w here L = 11 + I2 + >3 .
How many independent sta te s of this so r t are there?
24.4 E stim ate the low -lyin g excited state e n e r g ie s of helium in
the approxim ation of neglectin g exchange e ffe c ts . Do this
assum ing that one electro n is d escrib ed by the Is function
found in the ground state calcu lation and carryin g out the
variational procedure for a hydrogenic 2 s (and independently,
2p) wave function in the screen ed Coulomb field . The 2s state
m ust be taken orthogonal to som e appropriate Is sta te. The
root of the variational equation may be found by a rapidly
converging iteration m ethod.
24.5 C alculate the low -lyin g ex cited state e n e r g ie s of helium taking
into account exchange and spin e ffe c ts . U se appropriately
sy m m etrized product eigenfunctions for the configurations
(I s , 2 s) and ( I s , 2 p), w here the Is function is that found in
the ground state calcu lation . The required m atrix elem en ts
are given in problem 24.6 for r e fe r en ce .
24.6 C onsider the hydrogenic wave functions
Vls = 2 a 3 / 2 e ~ " Y 00
xV2s = Ml - Br)e~Pr Y 00
^ 2 p = cre'^ Y \ m
w here a = Z i s / a o , /3 = Z2 S/ 2 ao, and y = Z2 p/ 2 oo and w here fa, B
and c are determ ined by the orthogonality and norm alization
PROBLEMS 5 33
25.2 Using the v iria l theorem , show that in the T h om as-F erm i
m odel of a neutral atom the energy of e le c tr o sta tic repul
sion between the electro n s is 1 /7 the magnitude of the e le c
trostatic attraction between the electro n s and the nucleons.
25.3 E stim ate the order of magnitude of the follow ing quantities
in a neutral atom according to the T h o m a s-F erm i model:
(a) the s iz e of the atom,
(b) the average e le c tr o sta tic repulsion between two e le c tr o n s,
(c) the average kinetic energy of one electron ,
(d) the average speed of an electron ,
(e) the average angular m omentum of an electron ,
(f) the mean radial quantum num ber.
25.4 Show that the m ean perturbation of all sta te s of a given term
is zero for the sp in -o rb it in teraction.
25.5 C alculate the L • S splitting of the 3P term of helium .
25.6 Evaluate the L • S splitting of a sin g le n p e le ctr o n in a sp h eri
c a lly sym m etric potential. E xp ress the r e su lt in ter m s of an
arbitrary radial m atrix elem en t. E stim ate the radial m atrix
elem en t for the doublet splitting of sodium .
25.7 Two ele c tr o n s m ove in an ( np) 2 configuration in a sp h erically
sy m m etrica l potential. Regarding the e le c tr o sta tic repulsion
betw een the ele c tr o n s as a perturbation, evaluate the splitting
of term s in fir s t order approxim ation. N eglect the spin-
orbit interaction. U se qualitation con sid eration s to infer
ordering of the term s. Hint: To fa cilita te diagonalization of
the se cu la r equation, u se a rep resen tation in which M^and Ms
are diagonal and note that the sum of the roots of a se cu la r
equation is equal to the sum of the diagonal m atrix ele m en ts.
Chapter 26
26.3 U sing Hund’s r u les (se e problem 26.2), find the ground state
of the configuration n p x for x = 1, 2, . . . . 6 . F or each value
of x, state an elem en t for which this c a s e is r e a lize d physi
c a lly .
26.4 Do problem 26.3 for the configuration n d x for x = 1, 2 .............10.
Chapter 27
27.1 E stim ate the r ela tiv e freq u en cies and sep aration of energy
le v e ls for the e le c tr o n ic , vibrational and rotational m otions
of a diatom ic m o lecu le.
27.2 D erive the Schrodinger equation d escrib in g the m otion of the
nu clei of a diatom ic m olecu le in the approxim ation that the
n uclei m ove much m ore slo w ly than the e le c tr o n s and thus
e x p erien ce only an in teraction with the e le c tr o n s that is
averaged over many ele c tr o n revolu tion s. T his procedure
provides a sep aration of va ria b les betw een the e le c tr o n ic and
nhblear m otions. The approxim ation is c a lled the a d i a b a t i c
o r B orn-O ppenheim er approxim ation.
27.3 What are the p o ssib le sy m m etry sta te s of the diatom ic m ole
c u les D2, N2, LiH form ed from the bonding of the two atom s
in their ground sta te s?
27.4 What spin sy m m e tr ie s are p o ssib le for the rotational sta tes
of the deuterium m olecu le D2 in the ele c tr o n ic ground state?
The deuterium nucleus has spin 1.
27.5 C alculate the energy^of a rigid e le c tr ic dipole in a uniform
e le c tr ic fie ld . U se second order perturbation theory.
27.6 Show that the fo rc e betw een two hydrogen atom s in their
ground sta te s v a r ie s as 1/R7 if the atom s are separated by a
la rg e d istan ce R.
Chapter 28
28.1 The rate for making tra n sitio n s from an in itial state i to the
final state f is
w here p f is the num ber of final sta te s per unit energy interval
(“ Golden Rule No. 2 ” ).
(a) Find the e x p r e ssio n for U'if in fir s t order perturbation
theory. What n orm alization convention is im plied for the
wave function?
j(b) D erive an e x p r e ssio n for in second order perturbation
theory.
PROBLEMS 535
(c) Evaluate the density of sta te s p ( for a final state con sistin g
of two free p articles of definite total energy and m om entum .
(d) Evaluate p f for a final state con sistin g of three equal m ass
(free) p a r tic le s. E xp ress the r e su lts in the cen ter of m ass
sy ste m . How is the partition of energy among the three
p a rticles accounted for by a probability distribution?
(e) C alculate the dependence of p{ on energy near the energy
threshold for an /V-particle final state.
28.2 C alculate the rate for induced tran sition s from state m to
state n of an atom ic sy ste m in an e le c tr ic field with sp ectral
density £(&>) at tran sition frequency a> = — (Em - E n). Identify
n
the E in stein c o efficien t B and thereby infer the rate for
spontaneous e m issio n .
28.3 C onsider a p article of m a ss m bound in a th ree-d im en sion al
harm onic o sc illa to r potential -fer2. The particle is irradiated
with a m ild pulse U' = e x 2 e ~ u/ r)2 for the tim e i n t e r v a l <
t < ~. D eterm ine which tran sition s are p o ssib le and calculate
their p rob ab ilities. Identify the lim itin g c a s e s of sudden and
adiabatic perturbations. State the lim its of validity of the
perturbation calculation.
28.4 At t = 0, a hydrogen atom in its ground state is irradiated
with a uniform periodic e le c tr ic fie ld . D eterm ine the m ini
mum frequency of the field n e c e ssa r y to ionize the atom and
com pute the ionization probability per unit tim e. As an ap
proxim ation, the electro n in the final state may be regarded
as fr e e . State the lim its of validity of the approxim ation.
28.5 Show that the scatterin g of slow p a r tic les in a sh ort range
potential is ch aracterized b y d ^ k ) ~ fe2i+1. Find the proportion
ality constant in Born approxim ation and state the conditions
under which the approxim ation is valid.
28.7 Taking into account the sym m etry of the wave function, give
the d ifferen tial c r o s s se ctio n for e la s tic Coulomb scatterin g
of an electro n on an electro n and of an alpha p article on an
alpha p a rticle. D istinguish spin sta te s and a lso give the
form ula for the sca tterin g of unpolarized e le c tr o n s. Identify
quantum e ffe c ts and show how they disappear in the c la s s ic a l
lim it. F or r e fe r e n c e , the exact sca tterin g am plitude for a
fixed Coulomb potential is
j■ ( q) _____ y ^ . T d + iy) ' e ~2iyln sin 8/2
2 sin 2 | E(1 - iy)
(21) c + d -> a + b.
Chapter 29
29.1 V erify the anticom m utation rela tio n s
t p a ( r , t ) tftp ( r ' , t ) + tfra ( r , t ) = ta p S ( r - r)
29.3 C alculate the Born approxim ation m atrix elem en t for the
production of an e lectro n -p o sitro n pair in an extern al e le c tr ic
field
H' = g I ip+(x) p3 p l i p ( x ) M ( x ) d 3 x .